Download as pdf or txt
Download as pdf or txt
You are on page 1of 312

Iran’s Geometry Problems

Problems and Solutions from Contests

2014-2015
This booklet is prepared by Hirad Aalipanah, Iman Maghsoudi.
With special thanks to Morteza Saghafian, Mahdi Etesami Fard,
Davood Vakili, Erfan Salavati.
Copyright c Young Scholars Club 2014-2015. All rights reserved.
Ministry of education, Islamic Republic of Iran.
www.ysc.ac.ir - www.igo-official.ir
The first Iranian Geometry Olympiad was held simultaneously in Tehran and Is-
fahan on September 4th, 2014 with over 300 participants. This competition had two
levels, junior and senior which each level had 5 problems. The contestants solved
problems in 4 hours and 30 minutes.
In the end, the highest ranked participants in each level awarded with gold ruler,
silver ruler or bronze ruler respectively.
This booklet have the problems of this competition plus other geometry problems
used in other Iranian mathematical competition since summer of 2014 till spring of
2015.
This year the second Iranian Geometry Olympiad will be held in Tehran on Septem-
ber 3th, 2015. We tend to provide online presence for those who are interested from
other countries. Those who wish to participate can contact Mr. Salavati for more
information at erfan.salavati@gmail.com

Iranian Geometry Olympiads website: www.igo-official.ir


Problems
Problems 4

1.(Geometry Olympiad(Junior and Senior level)) In a right triangle ABC


we have ∠A = 90◦ , ∠C = 30◦ . Denot by C the circle passing through A which is
tangent to BC at the midpoint.Assume that C intersects AC and the circumcircle of
ABC at N and M respectively. Prove that M N ⊥BC.

Proposed by Mahdi Etesami Fard

2.(Geometry Olympiad(Junior Level)) The inscribed circle of 4ABC touches


BC, AC and AB at D, E and F respectively. Denote the perpendicular foots from
F , E to BC by K, L respectively. Let the second intersection of these perpendiculars
S
with the incircle be M , N respectively. Show that S4BM D
4CN D
= DK
DL

Proposed by Mahdi Etesami Fard

3.(Geometry Olympiad (Junior Level)) Each of Mahdi and Morteza has


drawn an inscribed 93-gon. Denote the first one by A1 A2 ...A93 and the second by
B1 B2 ...B93 . It is known that Ai Ai+1 k Bi Bi+1 for 1 6 i 6 93 (A93 = A1 , B93 = B1 ).
Ai Ai+1
Show that B i Bi+1
is a constant number independent of i.

Proposed by Morteza Saghafian

4.(Geometry Olympiad (Junior Level)) In a triangle ABC we have ∠C =


∠A + 90◦ . The point D on the continuation of BC is given such that AC = AD. A
point E in the side of BC in which A doesnt lie is chosen such that
1
∠EBC = ∠A, ∠EDC = ∠A
2
Prove that ∠CED = ∠ABC.

Proposed by Morteza Saghafian

5.(Geometry Olympiad (Junior Level)) Two points X, Y lie on the arc BC of


the circumcircle of 4ABC (this arc does not contain A) such that ∠BAX = ∠CAY .
Let M denotes the midpoint of the chord AX . Show that BM + CM > AY

Proposed by Mahan Tajrobekar


Problems 5

6.(Geometry Olympiad(Senior level)) In a quadrilateral ABCD we have


∠B = ∠D = 60◦ . Consider the line whice is drawn from M , the midpoint of AD,
parallel to CD. Assume this line intersects BC at P . A point X lies on CD such
that BX = CX. Prove that AB = BP ⇔ ∠M XB = 60◦

Proposed by Davood Vakili

7.(Geometry Olympiad(Senior level)) An acute-angled triangle ABC is given.


The circle with diameter BC intersects AB, AC at E, F respectively. Let M be the
midpoint of BC and P the intersection point of AM and EF . X is a point on the arc
EF and Y the second intersection point of XP with circle mentioned above. Show
that ∠XAY = ∠XY M .

Proposed by Ali Zooelm

8.(Geometry Olympiad(Senior level)) The tangent line to circumcircle of the


acute-angled triangle ABC (AC > AB) at A intersects the continuation of BC at P .
We denote by O the circumcenter of ABC. X is a point OP such that ∠AXP = 90◦ .
Two points E, F respectively on AB, AC at the same side of OP are chosen such
that
∠EXP = ∠ACX, ∠F XO = ∠ABX
If K, L denote the intersection points of EF with the circumcircle of 4ABC, show
that OP is tangent to the circumcircle of 4KLX.

Proposed by Mahdi Etesami Fard

9.(Geometry Olympiad(Senior level)) Two points P , Q lie on the side BC


of triangle ABC and have the same distance to the midpoint. The pependiculars
fromP , Q tp BC intesects AC, AB at E, F respectively. LEt M be the intersection
point of P F and EQ. If H1 and H2 denote the orthocenter of 4BF P and 4CEQ
recpectively, show that AM ⊥ H1 H2 .

Proposed by Mahdi Etesami Fard

10.(IGO Short list)Suppose that I is incenter of 4ABC and CI inresects AB


at D.In circumcircle of 4ABC, T is midpoint of arc BAC and BI intersect this circle
at M . If M D intersects AT at N , prove that: BM k CN .

Proposed by Ali Zooelm


Solutions
Solutions 11

1.(Geometry Olympiad(Junior and Senior Level)) In a right triangle ABC


we have ∠A = 90◦ , ∠C = 30◦ . Denot by C the circle passing through A which is
tangent to BC at the midpoint.Assume that C intersects AC and the circumcircle of
ABC at N and M respectively. Prove that M N ⊥BC.

Proposed by Mahdi Etesami Fard

−−−−−−−−−−−−−−−−−−−−−−−−−−−−−−−−−−−−−−−−−
solution.
Let K midpoint of side BC. Therefore:

AK = KC ⇒ ∠KAC = ∠N KC = 30◦

∠AN K = ∠N KC + ∠ACB = 60◦


A, K, N, M lie on circle (C). Therefore:

∠KAN = ∠KM N = 30◦ , ∠AM K = 60◦

We know that K is the circumcenter of 4ABC. So we can say KM = KC = AK.


Therefore 4AKM is equilateral.( because of ∠AM K = 60◦ ). So ∠AKM = 60◦ . We
know that ∠AKB = 60◦ , so we have ∠M KC = 60◦ . On the other hand:

∠KM N = 30◦ ⇒ M N ⊥BC


Solutions 12

2.(Geometry Olympiad(Junior Level)) The inscribed circle of 4ABC touches


BC, AC and AB at D, E and F respectively. Denote the perpendicular foots from
F , E to BC by K, L respectively. Let the second intersection of these perpendiculars
S
with the incircle be M , N respectively. Show that S4BM D
4CN D
= DK
DL

Proposed by Mahdi Etesami Fard

−−−−−−−−−−−−−−−−−−−−−−−−−−−−−−−−−−−−−−−−−
solution.
Let I be the incenter of 4ABC. We know that
)
∠BF K = 90◦ − ∠B 1
◦ 1 ⇒ ∠DF M = ∠B
∠BF D = 90 − 2 ∠B 2
But ∠DF M = ∠M DK. Therefore
1
∠M DK = ∠B
2
MK r
Hense 4M DK and 4BID are similar (same angles) and DK
= BD
. In the same
way we have N
DL
L r
= CD . Therefore

M K · BD N L · CD area of 4BM D M K · BD DK
r= = ⇒ = =
DK DL area of 4CN D N L · CD DL
Solutions 13

3.(Geometry Olympiad (Junior Level)) Each of Mahdi and Morteza has


drawn an inscribed 93-gon. Denote the first one by A1 A2 ...A93 and the second by
B1 B2 ...B93 . It is known that Ai Ai+1 k Bi Bi+1 for 1 6 i 6 93 (A93 = A1 , B93 = B1 ).
Ai Ai+1
Show that B i Bi+1
is a constant number independent of i.

Proposed by Morteza Saghafian

−−−−−−−−−−−−−−−−−−−−−−−−−−−−−−−−−−−−−−−−−
solution.
We draw a 93-gon similar with the second 93-gon in the circumcircle of the first
93-gon (so the sides of the second 93-gon would be multiplying by a constant number
c). Now we have two 93-gons witch are inscribed in the same circle and apply the
problem’s conditions. We name this 93-gons A1 A2 ...A93 and C1 C2 ...C93 .
_ _
We know that A1 A2 k C1 C2 . Therefore A1 C1 =A2 C2 but they lie on the opposite
_ _
side of each other. In fact, Ai Ci =Ai+1 Ci+1 and they lie on the opposite side of each
_ _ _ _
other for all 1 6 i 6 93 (A94 C94 =A1 C1 ). Therefore A1 C1 and A1 C1 lie on the opposite
_ ◦ ◦
side of each other. So A1 C1 = 0 or 180 . This means that the 93-gons are coincident
or reflections of each other across the center. So Ai Ai+1 = Ci Ci+1 for 1 6 i 6 93.
Ai Ai+1
Therefore, B i Bi+1
= c.
Solutions 14

4.(Geometry Olympiad (Junior Level)) In a triangle ABC we have ∠C =


∠A + 90◦ . The point D on the continuation of BC is given such that AC = AD. A
point E in the side of BC in which A doesnt lie is chosen such that
1
∠EBC = ∠A, ∠EDC = ∠A
2
Prove that ∠CED = ∠ABC.

Proposed by Morteza Saghafian

−−−−−−−−−−−−−−−−−−−−−−−−−−−−−−−−−−−−−−−−−
solution.
Suppose M is the midpoint of CD. Hense AM is the perpendicular bisector of
CD. AM intersects DE and BE at P, Q respectively. Therefore, P C = P D. We
have
∠EBA + ∠CAB = ∠A + ∠B + ∠A = 180◦ − ∠C + ∠A = 90◦
Hense AC ⊥ BE. Thus in 4ABQ, BC, AC are altitudes. This means C is the
orthocenter of this triangle and
1 1
∠CQE = ∠CQB = ∠A = ∠A + ∠A = ∠P DC + ∠P CD = ∠CP E
2 2
Hense CP QE is cyclic. Therefore

∠CED = ∠CEP = ∠CQP = ∠CQA = ∠CBA = ∠B.


Solutions 15

5.(Geometry Olympiad (Junior Level)) Two points X, Y lie on the arc BC of


the circumcircle of 4ABC (this arc does not contain A) such that ∠BAX = ∠CAY .
Let M denotes the midpoint of the chord AX . Show that

BM + CM > AY

Proposed by Mahan Tajrobekar

−−−−−−−−−−−−−−−−−−−−−−−−−−−−−−−−−−−−−−−−−
solution.
O is the circumcenter of 4ABC, so OM ⊥ AX. We draw a perpendicular line
from B to OM . This line intersects with the circumcircle at Z. Since OM ⊥ BZ,
OM is the perpendicular bisector of BZ. This means M Z = M B. By using triangle
inequality we have
BM + M C = ZM + M C > CZ
But BZ k AX, thus
_ _ _ _ _
AZ = BX = CY ⇒ ZAC = Y CA ⇒ CZ = AY

Hense BM + CM > AY.


Solutions 16

6.(Geometry Olympiad(Senior level)) In a quadrilateral ABCD we have


∠B = ∠D = 60◦ . Consider the line whice is drawn from M , the midpoint of AD,
parallel to CD. Assume this line intersects BC at P . A point X lies on CD such
that BX = CX. Prove that:

AB = BP ⇔ ∠M XB = 60◦

Proposed by Davood Vakili

−−−−−−−−−−−−−−−−−−−−−−−−−−−−−−−−−−−−−−−−−
solution.
Suppose X 0 is a point such that 4M BX 0 is equilateral.(X 0 and X lie on the same
side of M B) It’s enough to show that:

AB = BP ⇔ X 0 ≡ X

We want to prove that if AB = BP then ∠M XB = 60◦ .


AB = BP therefore 4ABP is equilateral. We know that ∠ABP = ∠M BX 0 = 60◦ ,
Therefore ∠ABM = ∠P BX 0 . On the other hand AB = BP, BM = BX 0 therefore
4BAM and 4BP X 0 are equal.

∠X 0 P M = 360◦ − ∠M P B − ∠BP X 0 = 360◦ − ∠DCB − ∠BAM 0 = 120◦


Solutions 17

M P k DC, so we can say ∠P M D = 120◦ . If we draw the line passing through


X 0 such that be parallel with CD and this line intersects AD in D0 , then quadri-
lateral M P X 0 D0 is isosceles trapezoid. Therefore P X 0 = M D0 . In the other hand
P X 0 = AM = M D ( becauese 4BAM and 4BP X 0 are equal.) According to the
statements we can say M D0 = M D. In other words, D0 ≡ D and X 0 lie on CD.
Therefore both of X and X 0 lie on intersection of DC and perpendicular bisector of
M B, so X 0 ≡ X.

Now we prove if ∠M XB = 60◦ then AB = BP .


Let P 0 such that 4M P 0 X be equilateral.(P 0 and X be on the same side of AB) It’s
enough to show that P 0 ≡ P .

Draw the line passing through P 0 such that be parallel with CD. Suppose that this
line intersects AD in M 0 .

∠XP 0 M 0 = 360◦ − ∠M 0 P 0 B − ∠BP 0 X = 360◦ − ∠DCA − ∠BAM = 120◦

Also ∠P 0 M 0 D = 120◦ . Therefore quadrilateral XP 0 M 0 D is isosceles trapezoid and


DM 0 = P 0 X = AM = DM . So we can say M 0 ≡ M ⇒ P 0 ≡ P .
Solutions 18

7.(Geometry Olympiad(Senior level)) An acute-angled triangle ABC is given.


The circle with diameter BC intersects AB, AC at E, F respectively. Let M be the
midpoint of BC and P the intersection point of AM and EF . X is a point on the arc
EF and Y the second intersection point of XP with circle mentioned above. Show
that ∠XAY = ∠XY M .

Proposed by Ali Zooelm

−−−−−−−−−−−−−−−−−−−−−−−−−−−−−−−−−−−−−−−−−
solution.
Suppose point K is intersection AM and circumcircle of 4AEF . M F tangent to
circumcircle of 4AEF at F .
( because of ∠M F C = ∠M CF = ∠AEF ). Therefore M F 2 = M K.M A . In the
other hand, M Y = M F so M Y 2 = M K.M A. It means

∠M Y K = ∠Y AM (1)

Also AP.P K = P E.P F = P X.P Y therefore AXKY is(...??) .Therefore

∠XAY = ∠XY K (2)

According to equation 1 and 2 we can say ∠XAY = ∠XY M .


Solutions 19

8.(Geometry Olympiad(Senior level)) The tangent line to circumcircle of the


acute-angled triangle ABC (AC > AB) at A intersects the continuation of BC at P .
We denote by O the circumcenter of ABC. X is a point OP such that ∠AXP = 90◦ .
Two points E, F respectively on AB, AC at the same side of OP are chosen such
that
∠EXP = ∠ACX, ∠F XO = ∠ABX
If K, L denote the intersection points of EF with the circumcircle of 4ABC, show
that OP is tangent to the circumcircle of 4KLX.

Proposed by Mahdi Etesami Fard

−−−−−−−−−−−−−−−−−−−−−−−−−−−−−−−−−−−−−−−−−
solution.

Let M and N on continuation of XF and XE such that M, L, X, N, K lie on


same circle. We have to prove ∠AM X = ∠ACX. In other hand, ∠ACX = ∠N XP
so we have to prove ∠ACX = ∠N M X.

We know that XF.F M = F L.F K = AF.F C. Therefore AM CX is cyclic and


∠AM X = ∠ACX. similarly we can say AN BX is cyclic. Now it’s enough to show
that ∠AM X = ∠N M X. In other words, we have to show that A, N , M lie on same
line. we know that AN BX is cyclic therefore:

∠N AM = ∠N AE + ∠A + ∠F AM = ∠EXB + ∠A + ∠CXF

= ∠A + 180◦ − ∠BXC + ∠ABX + ∠ACX

= ∠A + 180◦ − ∠BXC + ∠BXC − ∠A = 180◦


Solutions 20

9.(Geometry Olympiad(Senior level)) Two points P , Q lie on the side BC


of triangle ABC and have the same distance to the midpoint. The pependiculars
fromP , Q tp BC intesects AC, AB at E, F respectively. LEt M be the intersection
point of P F and EQ. If H1 and H2 denote the orthocenter of 4BF P and 4CEQ
recpectively, show that AM ⊥ H1 H2 .

Proposed by Mahdi Etesami Fard

−−−−−−−−−−−−−−−−−−−−−−−−−−−−−−−−−−−−−−−−−
solution.

First we show that if we move P and Q, the line AM doesn’t move. To show that
sin ∠A1
we calculate sin ∠A2
. By the law of sines in 4AF M and 4AEM we have

sin ∠A1 sin ∠F1 F M


= · (3)
sin ∠A2 sin ∠E1 EM

also, for 4F BP and 4CEQ we have

BP
)
sin ∠F1 = PF
· sin ∠B sin ∠F1 sin ∠B EQ
CQ ⇒ = · (4)
sin ∠E1 = EQ
· sin ∠C sin ∠E1 sin ∠C F P

from (3) and (4) we have

sin ∠A1 sin ∠B EQ F M


= · · (5)
sin ∠A2 sin ∠C F P EM

4F M Q and 4EM P are similar, thus


FM FQ EQ F Q + EP
= , =
FP F Q + EP EM EP
with putting this into (5) we have
Solutions 21

sin ∠A1 sin ∠B F Q


= · (6)
sin ∠A2 sin ∠C EP

on the other hand


FQ

tan ∠B = BQ 
 FQ tan ∠B
tan ∠C = EP ⇒ =
CP  EP tan ∠C
BQ = CP

if we put this in (6) we have


sin ∠A1 sin ∠B tan ∠B
= ·
sin ∠A2 sin ∠C tan ∠C
wich is constant.
now we show that H1 H2 s are parallel. consider α the angle between H1 H2 and
BC. Hense we have

H2 P − H1 Q
tan α = (7)
QP

H1 and H2 are the orthometers of 4BF P and 4CQE respectively. Thus we have
BQ · QP
QF · H1 Q = BQ · QP ⇒ H1 Q =
FQ
CP · P Q
EP · H2 P = CP · P Q ⇒ H2 P =
EP
but CP = BQ. Thus
P Q · BQ · (F Q − EP )
H2 P − H1 Q =
EP · F Q
by putting this in (7) :
BQ · (F Q − EP ) BQ BQ CP BQ
tan α = = − = −
EP · F Q EP FQ EP FQ

⇒ tan α = cot ∠B − cot ∠C (8)


Solutions 22

hense tan α is constant, thus H1 H2 s are parallel.


Soppuse θ is the angle between AM and BC. we have to show

tan α · tan θ = 1

let AM intersects with BC at X. We have


BX sin ∠A1 sin ∠C BX tan ∠B
= · ⇒ =
CX sin ∠A2 sin ∠B CX tan ∠C
let D be the foot of the altitude drawn from A. We have
AD
BX tan ∠B BD CD
= = AD
= ⇒ BD = CX
CX tan ∠C CD
BD

AD AD AD 1 1
tan θ = = = = CD BD
=
DX CD − CX CD − BD AD
− AD
cot ∠B − cot ∠C
this equality and (8) implies that AM ⊥ H1 H2 .
Problems of 2nd Iranian Geometry Olympiad 2015 (Elementary)

1. We have four wooden triangles with sides 3, 4, 5 centimeters. How many convex
polygons can we make by all of these triangles?(Just draw the polygons without any
proof)
A convex polygon is a polygon which all of it’s angles are less than 180◦ and there
isn’t any hole in it. For example:

This polygon isn’t convex This polygon is convex

P roposed by M ahdi Etesami F ard

2. Let ABC be a triangle with ∠A = 60◦ . The points M, N, K lie on BC, AC, AB
respectively such that BK = KM = M N = N C. If AN = 2AK, find the values of
∠B and ∠C.

P roposed by M ahdi Etesami F ard

3. In the figure below, we know that AB = CD and BC = 2AD. Prove that


∠BAD = 30◦ .

P roposed by M orteza Saghaf ian

1
4. In rectangle ABCD, the points M, N, P, Q lie on AB, BC, CD, DA respectively
such that the area of triangles AQM, BM N, CN P, DP Q are equal. Prove that the
quadrilateral M N P Q is parallelogram.

P roposed by M ahdi Etesami F ard

5. Do there exist 6 circles in the plane such that every circle passes through centers
of exactly 3 other circles?

P roposed by M orteza Saghaf ian

2
Problems of 2nd Iranian Geometry Olympiad 2015 (Medium)

1. In the figure below, the points P, A, B lie on a circle. The point Q lies inside the
circle such that ∠P AQ = 90◦ and P Q = BQ. Prove that the value of ∠AQB−∠P QA
is equal to the arc AB.

P roposed by Davood V akili

2. In acute-angled triangle ABC, BH is the altitude of the vertex B. The points D


and E are midpoints of AB and AC respectively. Suppose that F be the reflection of
H with respect to ED. Prove that the line BF passes through circumcenter of ABC.

P roposed by Davood V akili

3. In triangle ABC, the points M, N, K are the midpoints of BC, CA, AB respec-
tively. Let ωB and ωC be two semicircles with diameter AC and AB respectively,
outside the triangle. Suppose that M K and M N intersect ωC and ωB at X and Y
respectively. Let the tangents at X and Y to ωC and ωB respectively, intersect at Z.
prove that AZ⊥BC.

P roposed by M ahdi Etesami F ard

3
4. Let ABC be an equilateral triangle with circumcircle ω and circumcenter O. Let
P be the point on the arc BC( the arc which A doesn’t lie ). Tangent to ω at P in-
tersects extensions of AB and AC at K and L respectively. Show that ∠KOL > 90◦ .

P roposed by Iman M aghsoudi

5. a) Do there exist 5 circles in the plane such that every circle passes through centers
of exactly 3 circles?
b) Do there exist 6 circles in the plane such that every circle passes through centers
of exactly 3 circles?

P roposed by M orteza Saghaf ian

4
Problems of 2nd Iranian Geometry Olympiad 2015 (Advanced)

1. Two circles ω1 and ω2 (with centers O1 and O2 respectively) intersect at A and B.


The point X lies on ω2 . Let point Y be a point on ω1 such that ∠XBY = 90◦ . Let
X 0 be the second point of intersection of the line O1 X and ω2 and K be the second
point of intersection of X 0 Y and ω2 . Prove that X is the midpoint of arc AK.

P roposed by Davood V akili

2. Let ABC be an equilateral triangle with circumcircle ω and circumcenter O. Let


P be the point on the arc BC( the arc which A doesn’t lie ). Tangent to ω at P
intersects extensions of AB and AC at K and L respectively. Show that ∠KOL > 90◦ .

P roposed by Iman M aghsoudi

3. Let H be the orthocenter of the triangle ABC. Let l1 and l2 be two lines passing
through H and perpendicular to each other. l1 intersects BC and extension of AB
at D and Z respectively, and l2 intersects BC and extension of AC at E and X re-
spectively. Let Y be a point such that Y D k AC and Y E k AB. Prove that X, Y, Z
are collinear.

P roposed by Ali Golmakani

4. In triangle ABC, we draw the circle with center A and radius AB. This circle
intersects AC at two points. Also we draw the circle with center A and radius AC and
this circle intersects AB at two points. Denote these four points by A1 , A2 , A3 , A4 .
Find the points B1 , B2 , B3 , B4 and C1 , C2 , C3 , C4 similarly. Suppose that these 12
points lie on two circles. Prove that the triangle ABC is isosceles.

P roposed by M orteza Saghaf ian

5. Rectangles ABA1 B2 , BCB1 C2 , CAC1 A2 lie otside triangle ABC. Let C 0 be a


point such that C 0 A1 ⊥ A1 C2 and C 0 B2 ⊥ B2 C1 . Points A0 and B 0 are defined simi-
larly. Prove that lines AA0 , BB 0 , CC 0 concur.

P roposed by Alexey Zaslavsky (Russia)

5
Solutions of 2nd Iranian Geometry Olympiad 2015 (Elementary)

1. We have four wooden triangles with sides 3, 4, 5 centimeters. How many convex
polygons can we make by all of these triangles?(Just draw the polygons without any
proof)
A convex polygon is a polygon which all of it’s angles are less than 180◦ and there
isn’t any hole in it. For example:

This polygon isn’t convex This polygon is convex

P roposed by M ahdi Etesami F ard

−−−−−−−−−−−−−−−−−−−−−−−−−−−−−−−−−−−−−−−−−

Solution.

6
7
2. Let ABC be a triangle with ∠A = 60◦ . The points M, N, K lie on BC, AC, AB
respectively such that BK = KM = M N = N C. If AN = 2AK, find the values of
∠B and ∠C.

P roposed by M ahdi Etesami F ard

−−−−−−−−−−−−−−−−−−−−−−−−−−−−−−−−−−−−−−−−−

Solution.
Suppose the point P be the midpoint of AN . Therefore AK = AP = AN and
so we can say 4AP K is the equilateral triangle. So ∠AN K = ∠KP2
A
= 30◦ Let
∠ACB = ∠N M C = α. Therfore ∠ABC = ∠KM B = 120◦ − α. So ∠KM N = 60◦ .
Therefore 4KM N is the equilateral triangle. Now we know that ∠M N A = 90◦ .
Therefore α = 45◦ . So we have ∠C = 45◦ and ∠B = 75◦ .

8
3. In the figure below, we know that AB = CD and BC = 2AD. Prove that
∠BAD = 30◦ .

P roposed by M orteza Saghaf ian

−−−−−−−−−−−−−−−−−−−−−−−−−−−−−−−−−−−−−−−−−

Solution 1.
Let two points E and F on BC and AB respectively such that DF ⊥BC and
DE⊥AB. We can say DF = DC 2
= AB
2
.(because of ∠BCD = 30◦ and ∠DF C = 90◦ )
Also we know that DF = BE, therfore DE is the perpendicular bisector of AB. So
BD = AD.

Let H be a point on CD such that BH⊥CD. therefore BH = BC


2
= BD, so we
◦ ◦
can say D ≡ H and ∠BDC = 90 . Therefore ∠ABD = ∠BAD = 30 .

9
Solution 2.
Suppose that P is the point such that triangle DCP is Equilateral. We know that
P C⊥BC and P C = CD = AB, therfore quadrilateral ABCP is Rectangular.

⇒ ∠AP D = ∠AP C − ∠DP C = 90◦ − 60◦ = 30◦

In other hand, DP = DC and AP = BC. So 4ADP and 4BDC are congruent.


Therfore AD = BD.

Let the point H on CD such that BH⊥CD. therefore BH = BC


2
= BD, so we
◦ ◦
can say D ≡ H and ∠BDC = 90 . Therefore ∠ABD = ∠BAD = 30 .

10
4. In rectangle ABCD, the points M, N, P, Q lie on AB, BC, CD, DA respectively
such that the area of triangles AQM, BM N, CN P, DP Q are equal. Prove that the
quadrilateral M N P Q is parallelogram.

P roposed by M ahdi Etesami F ard

−−−−−−−−−−−−−−−−−−−−−−−−−−−−−−−−−−−−−−−−−

Solution.
Let AB = CD = a, AD = BC = b and AM = x, AQ = z, P C = y, N C = t. If
x 6= y, we can assume that x > y. We know that:

y<x ⇒ a−x<a−y (1)

SAQM = SCN P ⇒ zx = yt ⇒ z<t ⇒ b−t<b−z (2)


According to inequality 1, 2:

(a − x)(b − t) < (a − y)(b − z) ⇒ SBM N < SDP Q


it’s a contradiction. Therfore x = y, so z = t Now we can say two triangles AM Q
and CP N are congruent. Therefore M Q = N P and similarly M N = P Q. So the
quadrilateral M N P Q is parallelogram.

−−−−−−−−−−−−−−−−−−−−−−−−−−−−−−−−−−−−−−−−−

Comment.
If quadrilateral ABCD be the parallelogram, similarly we can show that quadri-
lateral M N P Q is parallelogram.

−−−−−−−−−−−−−−−−−−−−−−−−−−−−−−−−−−−−−−−−−

11
5. Do there exist 6 circles in the plane such that every circle passes through centers
of exactly 3 other circles?

P roposed by M orteza Saghaf ian

−−−−−−−−−−−−−−−−−−−−−−−−−−−−−−−−−−−−−−−−−

Solution.
In the picture below, we have 6 points in the plane such that for every point there
exists exactly 3 other points on a circle with radius 1 centimeter.

12
Solutions of 2nd Iranian Geometry Olympiad 2015 (Medium)

1. In the figure below, the points P, A, B lie on a circle. The point Q lies inside the
circle such that ∠P AQ = 90◦ and P Q = BQ. Prove that the value of ∠AQB−∠P QA
is equal to the arc AB.

P roposed by Davood V akili

−−−−−−−−−−−−−−−−−−−−−−−−−−−−−−−−−−−−−−−−−

Solution 1.
Let point M be the midpoint of P B. So we can say ∠P M Q = 90◦ and we know
that ∠P AQ = 90◦ , therefore quadrilateral P AM Q is cyclic. Therefore:

∠AP M = ∠AQM

In the other hand:

∠AQB − ∠AQP = ∠P QM + ∠AQM − ∠AQP = 2∠AQM

So we can say that the subtract ∠AQB from ∠P QA is equal to arc AB.

13
Solution 2.
Let the point K be the reflection of P to AQ. We have to show:

2∠AP B = ∠AQB − ∠AQP

Now we know that AQ is the perpendicular bisector of P K. So ∠AQP = ∠AQK


and P Q = KQ = BQ, therefore the point Q is the circumcenter of triangle P KB.
We know that:

2∠AP B = ∠KQB = ∠AQB − ∠AQK = ∠AQB − ∠AQP


Therefore the subtract ∠AQB from ∠P QA is equal to arc AB.

14
2. In acute-angled triangle ABC, BH is the altitude of the vertex B. The points D
and E are midpoints of AB and AC respectively. Suppose that F be the reflection of
H with respect to ED. Prove that the line BF passes through circumcenter of ABC.

P roposed by Davood V akili

−−−−−−−−−−−−−−−−−−−−−−−−−−−−−−−−−−−−−−−−−

Solution 1.
The circumcenter of 4ABC denote by O. We know that ∠OBA = 90◦ − ∠C,
therfore we have to show that ∠F BA = 90◦ − ∠C. We know that AD = BD = DH,
also DH = DF .

Therfore quadrilateral AHF B is cyclic (with circumcenter D)

⇒ ∠F BA = ∠F HE = 90◦ − ∠DEH , DE k BC ⇒ ∠DEH = ∠C

⇒ ∠F BA = 90◦ − ∠C

15
Solution 2.
The circumcenter of 4ABC denote by O. We know that quadrilateral ADOE is
cyclic. Also we know that AD = HD = DB, therefore:

∠A = ∠DHA = 180◦ − ∠DHE = 180◦ − ∠DF E ⇒ ADF E : cyclic

So we can say ADF OE is cyclic, therefore quadrilateral DF OE is cyclic.

∠C = ∠DEA = ∠DEF = ∠DOF

In the other hand: ∠C = ∠DOB so ∠DOF = ∠DOB, therefore B, F, O are collinear.

16
3. In triangle ABC, the points M, N, K are the midpoints of BC, CA, AB respec-
tively. Let ωB and ωC be two semicircles with diameter AC and AB respectively,
outside the triangle. Suppose that M K and M N intersect ωC and ωB at X and Y
respectively. Let the tangents at X and Y to ωC and ωB respectively, intersect at Z.
prove that AZ⊥BC.

P roposed by M ahdi Etesami F ard


−−−−−−−−−−−−−−−−−−−−−−−−−−−−−−−−−−−−−−−−−
Solution 1.
Let point H on BC such that AH⊥BC. Therefore quadrilaterals AXBH and
AY CH are cyclic. We know that KM and M N are parallel to AC and AB respec-
tively. So we can say ∠AKX = ∠AN Y = ∠A, therefore ∠ABX = ∠ACY = ∠A 2
and
∠XAB = ∠Y AC = 90◦ − ∠A 2
. So X, A, Y are collinear.

∠A ∠A
∠AHX = ∠ABX = , ∠AHY = ∠ACY = ⇒ ∠XHY = ∠XM Y = ∠A
2 2

Therefore quadrilateral XHM Y is cyclic. Also we know that ∠M XZ = ∠M Y Z =



90 , therefore quadrilateral M XZY is cyclic. So we can say ZXHM Y is cyclic. ther-
fore quadrilateral HXZY is cyclic.

17
∠A
In the other hand: ∠ZY X = ∠ACY = 2

∠A ∠A
∠ZHX = ∠ZY X = , ∠AHX = ⇒ ∠ZHX = ∠AHX
2 2
So the points Z, A, H are collinear, therefore AZ⊥BC.

−−−−−−−−−−−−−−−−−−−−−−−−−−−−−−−−−−−−−−−−−

Solution 2.
Let point H on BC such that AH⊥BC. We know that KM and M N are parallel
to AC and AB respectively. So we can say ∠AKX = ∠AN Y = ∠A, therefore
∠ABX = ∠ACY = ∠A 2
and ∠XAB = ∠Y AC = 90◦ − ∠A
2
. So X, A, Y are collinear.
∠A
⇒ ∠ZXY = ∠ZY X = ⇒ ZX = ZY
2
So the point Z lie on the radical axis of two these semicirculars. Also we know that
the line AH is the radical axis of two these semicirculars. Therefore the points Z, A, H
are collinear, therefore AZ⊥BC.

18
4. Let ABC be an equilateral triangle with circumcircle ω and circumcenter O. Let
P be the point on the arc BC( the arc which A doesn’t lie ). Tangent to ω at P in-
tersects extensions of AB and AC at K and L respectively. Show that ∠KOL > 90◦ .

P roposed by Iman M aghsoudi

−−−−−−−−−−−−−−−−−−−−−−−−−−−−−−−−−−−−−−−−−

Solution 1.
Suppose that M and N be the midpoints of AB and AC respectively. We know
that quadrilateral BM N C is cyclic. Also ∠BP C = 120◦ > 90◦ , so we can say
the point P is in the circumcircle of quadrilateral BM N C. Therefore: ∠M P N >
∠M BN = 30◦
In the other hand, quadrilaterals KM OP and N OP L are cyclic. Therefore:

∠M KO = ∠M P O , ∠N LO = ∠N P O ⇒ ∠AKO + ∠ALO = ∠M P N > 30◦

⇒ ∠KOL = ∠A + ∠AKO + ∠ALO > 90◦

19
Solution 2.
Suppose that ∠KOL ≤ 90◦ , therfore KL2 ≤ OK 2 + OL2 . Assume that R is the
radius of a circumcircle 4ABC. Let BK = x and LC = y and AB = AC = BC = a.
According to law of cosines in triangle AKL, we have:

KL2 = AK 2 + AL2 − AK.AL.cos(∠A) ⇒ KL2 = (a + x)2 + (a + y)2 − (a + x)(a + y)

In the other hand:

KB.KA = OK 2 − R2 ⇒ OK 2 = R2 + x(a + x)

LC.LA = OL2 − R2 ⇒OL2 = R2 + y(a + y)



We know that KL2 ≤ OK 2 + OL2 and a = R 3, therfore:

(a + x)2 + (a + y)2 − (a + x)(a + y) ≤ 2R2 + x(a + x) + y(a + y)

⇒ R2 ≤ xy (1)
KL is tangent to circumcircle of 4ABC at P . So we have:

KP 2 = KB.KA = x(a + x) > x2 ⇒ KP > x (2)

LP 2 = LC.LA = y(a + y) > y 2 ⇒ LP > y (3)


According to inequality 2, 3 we can say: xy < KP.LP (4)

Now According to inequality 1, 4 we have: R2 < KP.LP (5)

20
We know that ∠KOL ≤ 90◦ , therefore KOL is acute-triangle. Suppose that H is
orthocenter of 4KOL. So the point H lies on OP and we can say HP ≤ OP .

In other hand, ∠HKP = ∠P OL and ∠KHP = ∠OLP , therefore two triangles


T HP and OP L are similar. So we have:
KP OP
= ⇒ KP.LP = HP.OP ≤ OP 2 = R2
HP LP
But according to inequality 5, we have R2 < KP.LP and it’s a contradiction.
Therfore ∠KOL > 90◦ .

21
5. a) Do there exist 5 circles in the plane such that every circle passes through centers
of exactly 3 circles?
b) Do there exist 6 circles in the plane such that every circle passes through centers
of exactly 3 circles?

P roposed by M orteza Saghaf ian

−−−−−−−−−−−−−−−−−−−−−−−−−−−−−−−−−−−−−−−−−

a)Solution.
There aren’t such 5 circles. Suppose that these circles exists, therefore their centers
are 5 points that each point has same distance from 3 other points and has diffrent
distance from the remaining point. We draw an arrow from each point to it’s diffrent
distance point.
- lemma 1. We don’t have two points such Oi , Oj that each one is the diffrent
distance point of the other one.
proof. If we have such thing then Oi and Oj both have same distance to the
remaining points, therefore both of them are circumcenter of the remaining points,
which is wrong.
- lemma 2. We don’t have 4 points such Oi , Oj , Ok , Ol that Oi , Oj put their arrow
in Ok and OK puts it’s arrow in Ol .
proof. If we name the remaining point Om then the distances of Oi from Oj , Ol ,
Om are equal and the distances of Oj from Oi , Ol , Om are equal. Therefore each of
Ol , Om is the diffrent distance point of another which is wrong (according to lemma
1).
so each point sends an arrow and recives an arrow. Because of lemma 1 we don’t
have 3 or 4 points cycles. Therefore we only have one 5 points cycle. So each pair of
these 5 points should have equal distance. which is impossible.

−−−−−−−−−−−−−−−−−−−−−−−−−−−−−−−−−−−−−−−−−

b)Solution.
in the picture below, we have 6 points in the plane such that for every point there
exists exactly 3 other points on a circle with radius 1 centimeter.

22
Solutions of 2nd Iranian Geometry Olympiad 2015 (Advanced)

1. Two circles ω1 and ω2 (with centers O1 and O2 respectively) intersect at A and B.


The point X lies on ω2 . Let point Y be a point on ω1 such that ∠XBY = 90◦ . Let
X 0 be the second point of intersection of the line O1 X and ω2 and K be the second
point of intersection of X 0 Y and ω2 . Prove that X is the midpoint of arc AK.

P roposed by Davood V akili

−−−−−−−−−−−−−−−−−−−−−−−−−−−−−−−−−−−−−−−−−

Solution.
Suppose that the point Z be the intersection of BX and circle ω1 . We know that
∠Y BZ = 90◦ , therefore the points Y, O1 , Z are collinear.

∠O1 Y A = ∠ABX = ∠AX 0 X ⇒ Y AX 0 O1 : cyclic

In the other hand, we know that AO1 = Y O1 so ∠AX 0 X = ∠Y X 0 O1 = ∠XX 0 K.


Therefore the point X lies on the midpoint of arc AK.

23
2. Let ABC be an equilateral triangle with circumcircle ω and circumcenter O. Let
P be the point on the arc BC( the arc which A doesn’t lie ). Tangent to ω at P in-
tersects extensions of AB and AC at K and L respectively. Show that ∠KOL > 90◦ .

P roposed by Iman M aghsoudi

−−−−−−−−−−−−−−−−−−−−−−−−−−−−−−−−−−−−−−−−−

Solution 1.
Suppose that M and N be the midpoints of AB and AC respectively. We know
that quadrilateral BM N C is cyclic. Also ∠BP C = 120◦ > 90◦ , so we can say
the point P is in the circumcircle of quadrilateral BM N C. Therefore: ∠M P N >
∠M BN = 30◦
In the other hand, quadrilaterals KM OP and N OP L are cyclic. Therefore:

∠M KO = ∠M P O , ∠N LO = ∠N P O ⇒ ∠AKO + ∠ALO = ∠M P N > 30◦

⇒ ∠KOL = ∠A + ∠AKO + ∠ALO > 90◦

24
Solution 2.
Suppose that ∠KOL ≤ 90◦ , therfore KL2 ≤ OK 2 + OL2 . Assume that R is the
radius of a circumcircle 4ABC. Let BK = x and LC = y and AB = AC = BC = a.
According to law of cosines in triangle AKL, we have:

KL2 = AK 2 + AL2 − AK.AL.cos(∠A) ⇒ KL2 = (a + x)2 + (a + y)2 − (a + x)(a + y)

In the other hand:

KB.KA = OK 2 − R2 ⇒ OK 2 = R2 + x(a + x)

LC.LA = OL2 − R2 ⇒OL2 = R2 + y(a + y)



We know that KL2 ≤ OK 2 + OL2 and a = R 3, therfore:

(a + x)2 + (a + y)2 − (a + x)(a + y) ≤ 2R2 + x(a + x) + y(a + y)

⇒ R2 ≤ xy (1)
KL is tangent to circumcircle of 4ABC at P . So we have:

KP 2 = KB.KA = x(a + x) > x2 ⇒ KP > x (2)

LP 2 = LC.LA = y(a + y) > y 2 ⇒ LP > y (3)


According to inequality 2, 3 we can say: xy < KP.LP (4)

Now According to inequality 1, 4 we have: R2 < KP.LP (5)

25
We know that ∠KOL ≤ 90◦ , therefore KOL is acute-triangle. Suppose that H is
orthocenter of 4KOL. So the point H lies on OP and we can say HP ≤ OP .

In other hand, ∠HKP = ∠P OL and ∠KHP = ∠OLP , therefore two triangles


T HP and OP L are similar. So we have:
KP OP
= ⇒ KP.LP = HP.OP ≤ OP 2 = R2
HP LP
But according to inequality 5, we have R2 < KP.LP and it’s a contradiction.
Therfore ∠KOL > 90◦ .

26
3. Let H be the orthocenter of the triangle ABC. Let l1 and l2 be two lines passing
through H and perpendicular to each other. l1 intersects BC and extension of AB
at D and Z respectively, and l2 intersects BC and extension of AC at E and X re-
spectively. Let Y be a point such that Y D k AC and Y E k AB. Prove that X, Y, Z
are collinear.

P roposed by Ali Golmakani


−−−−−−−−−−−−−−−−−−−−−−−−−−−−−−−−−−−−−−−−−
Solution.
Suppose that HZ intersects AC at P and HX intersects AB at Q. According to
Menelaus’s theorem in two triangles AQX and AP Z we can say:
CX AB QE BZ AC P D
. . =1 (1) and . . =1 (2)
AC BQ EX AB P C DZ

In the other hand, H is the orthocenter of 4ABC. So BH⊥AC and we know that
∠DHE = 90◦ , therefore ∠HXA = ∠BHZ = α. Similarly we can say ∠HZA =
∠CHX = θ.

27
According to law of sines in 4HP C, 4HCX and 4HP X:

sin(90 − θ) sin(∠HCP ) sin(θ) sin(∠HCX) HP sin(α)


= , = , =
PC HP CX HX HX sin(90 − α)

PC tan(α)
⇒ =
CX tan(θ)
Similarly, according to law of sines in 4HBQ, 4HBZ and 4HQZ, we can show:

BZ tan(α) BZ PC PC CX
⇒ = ⇒ = ⇒ = (3)
BQ tan(θ) BQ CX BZ BQ

According to equality 1, 2 and 3, we can say:


XE PD
= (4)
EQ ZD

Suppose that the line which passes through E and parallel to AB, intersects ZX at
Y1 and the line which passes through D and parallel to AC, intersects ZX at Y2 .
According to Thales’s theorem we can say:
Y1 X XE Y2 X PD
= , =
ZY1 EQ ZY2 ZD

According to equality 4, we show that Y1 ≡ Y2 , therefore the point Y lies on ZX.

28
4. In triangle ABC, we draw the circle with center A and radius AB. This circle
intersects AC at two points. Also we draw the circle with center A and radius AC and
this circle intersects AB at two points. Denote these four points by A1 , A2 , A3 , A4 .
Find the points B1 , B2 , B3 , B4 and C1 , C2 , C3 , C4 similarly. Suppose that these 12
points lie on two circles. Prove that the triangle ABC is isosceles.

P roposed by M orteza Saghaf ian

−−−−−−−−−−−−−−−−−−−−−−−−−−−−−−−−−−−−−−−−−

Solution 1.
Suppose that triangle ABC isn’t isosceles and a > b > c. In this case, there
are four points (from these 12 points) on each side of 4ABC. Suppose that these
12 points lie on two circles ω1 and ω2 . Therefore each one of the circles ω1 and ω2
intersects each side of 4ABC exactly at two points. Suppose that P (A, ω1 ), P (A, ω2 )
are power of the point A with respect to circles ω1 , ω2 respectively. Now we know
that:
P (A, ω1 ).P (A, ω2 ) = b.b.(a − c).(a + c) = c.c.(a − b)(a + b)
⇒ b2 (a2 − c2 ) = c2 (a2 − b2 ) ⇒ a2 (b2 − c2 ) = 0 ⇒ b=c
But we know that b > c and it’s a contradiction. Therefore the triangle ABC is
isosceles.

−−−−−−−−−−−−−−−−−−−−−−−−−−−−−−−−−−−−−−−−−

Solution 2.
Suppose that triangle ABC isn’t isosceles. In this case, there are four points
(from these 12 points) on each side of 4ABC. Suppose that these 12 points lie on
two circles ω1 and ω2 . Therefore each one of the circles ω1 and ω2 intersects each side
of 4ABC exactly at two points (and each one of the circles ω1 and ω2 doesn’t pass
through A, B, C). We know that the intersections of ω1 and the sides of 4ABC is
even number. Also the intersections of ω2 and the sides of 4ABC is even number.
But Among the these 12 points, just 3 points lie on the sides of 4ABC and this is
odd number. So it’s a contradiction. Therefore the triangle ABC is isosceles.

29
5. Rectangles ABA1 B2 , BCB1 C2 , CAC1 A2 lie otside triangle ABC. Let C 0 be a
point such that C 0 A1 ⊥ A1 C2 and C 0 B2 ⊥ B2 C1 . Points A0 and B 0 are defined simi-
larly. Prove that lines AA0 , BB 0 , CC 0 concur.

P roposed by Alexey Zaslavsky (Russia)

−−−−−−−−−−−−−−−−−−−−−−−−−−−−−−−−−−−−−−−−−

Solution.
Suppose that lA is the line which passes through A and perpendicular to B2 C1 .
Let lB and lC similarly. Suppose that CB1 = BC2 = x and BA1 = AB2 = y and
AC1 = CA2 = z. According to angles equality, we can say:

sin(∠A1 ) y sin(∠B1 ) x sin(∠C1 ) z


= , = , =
sin(∠A2 ) z sin(∠B2 ) y sin(∠C2 ) x

According to sine form of Ceva’s theorem in 4ABC, lA , lB , lC are concur. Suppose


that lA , lB , lC pass through the point P . We know that 4P BC and 4A0 C2 B1 are
equal. ( because of BP k A0 C2 , CP k A0 B1 , BC k B1 C2 and BC = B1 C2 ). So we
have:

P A0 = x , P C 0 = y , P B 0 = z P A0 ⊥BC , P B 0 ⊥AC , P C 0 ⊥AB

30
Suppose that P A0 , P B 0 , P C 0 intersects BC, AC, AB at D, E, F respectively and:
P D = m , P E = n , P F = t. According to before figure, we have:
sin(∠A1 ) n y sin(∠B1 ) t x sin(∠C1 ) m z
= = , = = , = =
sin(∠A2 ) t z sin(∠B2 ) m y sin(∠C2 ) n x
kyz
If n = ky, then: t = kz , m = x
.

Now draw the line from A0 such that be parallel to BC. The intersection of this
line and extension AB and AC denote by B3 and C3 respectively. Let the point A00
be the intersection of AA0 and BC. According to Thales’s theorem, we have:
BA00 B3 A0
=
CA00 C 3 A0

31
Let ∠B3 P A0 = α and ∠C3 P A0 = θ. We know that the quadrilaterals P F B3 A0
and P EC3 A0 are cyclic. Therefore ∠B3 F A0 = α and ∠C3 EA0 = θ.

According to law of sines in 4P B3 A0 and 4P C3 A0 and 4P C3 B3 :

B3 A0 tan(α)
0
=
C3 A tan(θ)

Also according to law of sines in 4P F A0 :

t sin(∠B + α − 90) cos(∠B + α)


= = = cos(∠B) − tan(α).sin(∠B)
x cos(α) cos(α)
t
cos(∠B) − x
⇒ tan(α) =
sin(∠B)

Similarly we can say:


n
cos(∠C) − x B3 A0 BA00 x.cos(∠B) − t sin(∠C)
tan(θ) = ⇒ 0
= 00
= .
sin(∠C) C3 A CA x.cos(∠C) − n sin(∠B)

Similarly, two other fractions can be calculated.

32
According to Ceva’s theorem in 4ABC, we have to that:

x.cos(∠B) − t sin(∠C) z.cos(∠C) − m sin(∠A) y.cos(∠A) − n sin(∠B)


. . . . . =1
x.cos(∠C) − n sin(∠B) z.cos(∠A) − t sin(∠C) y.cos(∠B) − m sin(∠A)

x.cos(∠B) − t z.cos(∠C) − m y.cos(∠A) − n


⇐⇒ . . =1
x.cos(∠C) − n z.cos(∠A) − t y.cos(∠B) − m

In other hand, we know that:


kyz
n = ky , t = kz , m=
x

x.cos(∠B) − kz x.cos(∠C) − ky x.cos(∠A) − kx


⇐⇒ . . =1
x.cos(∠C) − ky x.cos(∠A) − kx x.cos(∠B) − kz

Therfore, we show that AA0 , BB 0 , CC 0 are concur.

33
1

Problems of 3rd Iranian Geometry Olympiad 2016 (Elementary)

1. Ali wants to move from point A to point B. He cannot walk inside the
black areas but he is free to move in any direction inside the white areas (not
only the grid lines but the whole plane). Help Ali to find the shortest path
between A and B. Only draw the path and write its length.

P roposed by M orteza Saghaf ian

2. Let ω be the circumcircle of triangle ABC with AC > AB. Let X be a


point on AC and Y be a point on the circle ω, such that CX = CY = AB.
(The points A and Y lie on different sides of the line BC). The line XY inter-
sects ω for the second time in point P . Show that P B = P C.

P roposed by Iman M aghsoudi

3. Suppose that ABCD is a convex quadrilateral with no parallel sides. Make


a parallelogram on each two consecutive sides. Show that among these 4 new
points, there is only one point inside the quadrilateral ABCD.

P roposed by M orteza Saghaf ian


2

4. In a right-angled triangle ABC (∠A = 90◦ ), the perpendicular bisector


of BC intersects the line AC in K and the perpendicular bisector of BK inter-
sects the line AB in L. If the line CL be the internal bisector of angle C, find
all possible values for angles B and C.

P roposed by M ahdi Etesami F ard

5. Let ABCD be a convex quadrilateral with these properties: √


∠ADC = 135◦ and ∠ADB−∠ABD = 2∠DAB = 4∠CBD. If BC = 2CD
prove that AB = BC + AD.

P roposed by M ahdi Etesami F ard


3

Problems of 3rd Iranian Geometry Olympiad 2016 (Medium)

1. In trapezoid ABCD with AB k CD, ω1 and ω2 are two circles with di-
ameters AD and BC, respectively. Let X and Y be two arbitrary points on ω1
and ω2 , respectively. Show that the length of segment XY is not more than
half of the perimeter of ABCD.

P roposed by M ahdi Etesami F ard

2. Let two circles C1 and C2 intersect in points A and B. The tangent to


C1 at A intersects C2 in P and the line P B intersects C1 for the second time
in Q (suppose that Q is outside C2 ). The tangent to C2 from Q intersects C1
and C2 in C and D, respectively (The points A and D lie on different sides of
the line P Q). Show that AD is bisector of the angle CAP .

P roposed by Iman M aghsoudi

3. Find all positive integers N such that there exists a triangle which can
be dissected into N similar quadrilaterals.

P roposed by N ikolai Beluhov (Bulgaria) and M orteza Saghaf ian

4. Let ω be the circumcircle of right-angled triangle ABC (∠A = 90◦ ). Tan-


gent to ω at point A intersects the line BC in point P . Suppose that M is the
midpoint of (the smaller) arc AB, and P M intersects ω for the second time in
Q. Tangent to ω at point Q intersects AC in K. Prove that ∠P KC = 90◦ .

P roposed by Davood V akili

5. Let the circles ω and ω 0 intersect in points A and B. Tangent to circle


ω at A intersects ω 0 in C and tangent to circle ω 0 at A intersects ω in D.
Suppose that the internal bisector of ∠CAD intersects ω and ω 0 at E and F ,
respectively, and the external bisector of ∠CAD intersects ω and ω 0 in X and
Y , respectively. Prove that the perpendicular bisector of XY is tangent to the
circumcircle of triangle BEF .

P roposed by M ahdi Etesami F ard


4

Problems of 3rd Iranian Geometry Olympiad 2016 (Advanced)

1. Let the circles ω and ω 0 intersect in A and B. Tangent to circle ω at A


intersects ω 0 in C and tangent to circle ω 0 at A intersects ω in D. Suppose that
the segment CD intersects ω and ω 0 in E and F , respectively (assume that E
is between F and C). The perpendicular to AC from E intersects ω 0 in point
P and perpendicular to AD from F intersects ω in point Q (The points A, P
and Q lie on the same side of the line CD). Prove that the points A, P and Q
are collinear.

P roposed by M ahdi Etesami F ard

2. In acute-angled triangle ABC, altitude of A meets BC at D, and M is


midpoint of AC. Suppose that X is a point such that ∠AXB = ∠DXM = 90◦
(assume that X and C lie on opposite sides of the line BM ). Show that
∠XM B = 2∠M BC.

P roposed by Davood V akili

3. Let P be the intersection point of sides AD and BC of a convex qualri-


lateral ABCD. Suppose that I1 and I2 are the incenters of triangles P AB and
P DC, respectively. Let O be the circumcenter of P AB, and H the orthocenter
of P DC. Show that the circumcircles of triangles AI1 B and DHC are tangent
together if and only if the circumcircles of triangles AOB and DI2 C are tangent
together.

P roposed by Hooman F attahimoghaddam

4. In a convex quadrilateral ABCD, the lines AB and CD meet at point


E and the lines AD and BC meet at point F . Let P be the intersection point
of diagonals AC and BD. Suppose that ω1 is a circle passing through D and
tangent to AC at P . Also suppose that ω2 is a circle passing through C and
tangent to BD at P . Let X be the intersection point of ω1 and AD, and Y
be the intersection point of ω2 and BC. Suppose that the circles ω1 and ω2
intersect each other in Q for the second time. Prove that the perpendicular
from P to the line EF passes through the circumcenter of triangle XQY .

P roposed by Iman M aghsoudi

5. Do there exist six points X1 , X2 , Y1 , Y2 , Z1 , Z2 in the plane such that all


of the triangles Xi Yj Zk are similar for 1 ≤ i, j, k ≤ 2?

P roposed by M orteza Saghaf ian


5

Solutions of 3nd Iranian Geometry Olympiad 2016 (Elementary)

1. Ali wants to move from point A to point B. He cannot walk inside the
black areas but he is free to move in any direction inside the white areas (not
only the grid lines but the whole plane). Help Ali to find the shortest path
between A and B. Only draw the path and write its length.

P roposed by M orteza Saghaf ian

−−−−−−−−−−−−−−−−−−−−−−−−−−−−−−−−−−−−−−−−−
Solution.

According to Pythagorean theorem, the length of the path AB is equal to:


p p p √
32 + 3 2 + 32 + 4 2 + 1 + 22 + 2 2 + 1 = 7 + 5 2
6

2.Let ω be the circumcircle of triangle ABC with AC > AB. Let X be a point
on AC and Y be a point on the circle ω, such that CX = CY = AB. (The
points A and Y lie on different sides of the line BC). The line XY intersects ω
for the second time in point P . Show that P B = P C.

P roposed by Iman M aghsoudi

−−−−−−−−−−−−−−−−−−−−−−−−−−−−−−−−−−−−−−−−−
Solution.
We know that CX = CY therefore:
_ _ _
∠Y XC = ∠XY C ⇒AP + CY =P C
_ _ _ _ _
Also we have AB = CY therefore AP + CY =AP + AB=P B, so P B = P C.
7

3. Suppose that ABCD is a convex quadrilateral with no parallel sides. Make


a parallelogram on each two consecutive sides. Show that among these 4 new
points, there is only one point inside the quadrilateral ABCD.

P roposed by M orteza Saghaf ian

−−−−−−−−−−−−−−−−−−−−−−−−−−−−−−−−−−−−−−−−−
Solution.
It’s clear that the ray from B parallel to AD passes through the quadrilateral
if and only if ∠DAB + ∠ABC > 180◦ .

We have to find a parallelogram such that both of it’s rays pass thorough
ABCD. Among A, B and C, D there is exactly one set with sum of angles
greater than 180◦ . Also among A, D and B, D there is exactly one set with sum
of angles greater than 180◦ . These two good sets have a vertex in common, say
A. So both of the rays from B parallel to AD, and from D parallel to AB, are
inside the quadlirateral.
8

4. In a right-angled triangle ABC (∠A = 90◦ ), the perpendicular bisector of


BC intersects the line AC in K and the perpendicular bisector of BK intersects
the line AB in L. If the line CL be the internal bisector of angle C, find all
possible values for angles B and C.

P roposed by M ahdi Etesami F ard

−−−−−−−−−−−−−−−−−−−−−−−−−−−−−−−−−−−−−−−−−
Solution.
We have three cases:
Case i. AC > AB. We know that:

∠LBK = ∠LKB = α ⇒ ∠KLA = 2α ⇒ ∠LKA = 90◦ − 2α


∠BKA α
BK = CK ⇒ ∠KBC = ∠KCB = = 45◦ −
2 2
Let T be a point on BC such that LT ⊥BC. We know that the line CL is the
internal bisector of angle C, so LT = LA also we have LB = LK therefore two
triangles BT L and KAL are congruent.
α
⇒ ∠LBT = ∠LKA ⇒ 45◦ + = 90◦ − 2α ⇒ α = 18◦
2
Therefore ∠B = 45◦ + α
2 = 54◦ and ∠C = 36◦
9

Case ii. AC < AB. We know that:

∠LBK = ∠LKB = α ⇒ ∠KLA = 2α ⇒ ∠LKA = 90◦ − 2α

Let T be a point on BC such that LT ⊥BC. We know that the line CL is the
internal bisector of angle C, so LT = LA also we have LB = LK therefore two
triangles BT L and KAL are equal.

⇒ ∠LBT = ∠LKA = 90◦ − 2α ⇒ ∠CBK = ∠BKC = 90◦ − α

On the other hand we have:

BK = CK ⇒ ∠CBK = ∠BKC = 60◦ ⇒ α = 30◦

Therefore ∠B = 90◦ − 2α = 30◦ and ∠C = 60◦

Case iii. AC = AB. In this case, K ≡ A and L is the midpoint of AB. Let T
be a point on BC such that LT ⊥BC. We know that the line CL is the internal
bisector of angle C, so LT = LA = LB which is impossible.
10

5. Let ABCD be a convex quadrilateral with these properties: √


∠ADC = 135◦ and ∠ADB−∠ABD = 2∠DAB = 4∠CBD. If BC = 2CD
prove that AB = BC + AD.

P roposed by M ahdi Etesami F ard

−−−−−−−−−−−−−−−−−−−−−−−−−−−−−−−−−−−−−−−−−
Solution.
Suppose that ∠CBD = α, so ∠DAB = 2α, therefore:
∠ADB − ∠ABD = 4α , ∠ADB + ∠ABD = 180◦ − 2α
⇒ ∠ADB = 90◦ + α , ∠ABD = 90◦ − 3α ⇒ ∠DAB + ∠CBA = 90◦
Let P be intersection point of AD and BC. So we have ∠AP

B = 90◦ . On the
other hand we know that ∠P DC = 45◦ , therefore P D = 22 CD = BC
2

Let the point Q be the reflection of point D in point P , Thus QD = 2P D = BC.


We know that two triangles DP B and QP B are congruent. So ∠CBD =
∠CBQ = α, therefore ∠ABQ = 90◦ − α. On the other hand ∠DAB = 2α, so
the triangle ABQ is isosceles.
⇒ AB = AQ ⇒ AB = DQ + AD = BC + AD
.
11

Solutions of 3nd Iranian Geometry Olympiad 2016 (Medium)

1. In trapezoid ABCD with AB k CD, ω1 and ω2 are two circles with di-
ameters AD and BC, respectively. Let X and Y be two arbitrary points on ω1
and ω2 , respectively. Show that the length of segment XY is not more than
half of the perimeter of ABCD.

P roposed by M ahdi Etesami F ard

−−−−−−−−−−−−−−−−−−−−−−−−−−−−−−−−−−−−−−−−−
First solution.
Let O1 and O2 be the centers of circles ω1 and ω2 , respectively. It’s clear that
O1 and O2 are the midpoints of AD and BC, respectively.
AD BC AB + CD
XO1 = , Y O2 = , O 1 O2 =
2 2 2
AB + BC + CD + DA
⇒ XY ≤ XO1 + O1 O2 + Y O2 =
2
12

Second solution.
The farthest points of two circles lie on their center line.

And it’s clear in the figure that:


AD AB + CD BC
XO1 = , O1 O2 = , Y O2 =
2 2 2
13

2. Let two circles C1 and C2 intersect in points A and B. The tangent to C1


at A intersects C2 in P and the line P B intersects C1 for the second time in Q
(suppose that Q is outside C2 ). The tangent to C2 from Q intersects C1 and
C2 in C and D, respectively (The points A and D lie on different sides of the
line P Q). Show that AD is bisector of the angle CAP .

P roposed by Iman M aghsoudi

−−−−−−−−−−−−−−−−−−−−−−−−−−−−−−−−−−−−−−−−−
Solution.
We know that:
∠CAB = ∠CQB , ∠DAB = ∠BDQ
⇒ ∠CAD = ∠CAB + ∠DAB = ∠CQB + ∠BDQ = ∠P BD = ∠P AD
Therefore AD is the bisector of ∠CAP .
14

3. Find all positive integers N such that there exists a triangle which can be
dissected into N similar quadrilaterals.

P roposed by N ikolai Beluhov (Bulgaria) and M orteza Saghaf ian

−−−−−−−−−−−−−−−−−−−−−−−−−−−−−−−−−−−−−−−−−
Solution.
For N = 1 it’s clear that this is impossible. Also for N = 2 this dissection is
impossible too, because one of the two quadrilaterals is convex and the other is
concave. For N ≥ 3 we can do this kind of dissection in equilateral triangle.
15

4. Let ω be the circumcircle of right-angled triangle ABC (∠A = 90◦ ). Tangent


to ω at point A intersects the line BC in point P . Suppose that M is the
midpoint of (the smaller) arc AB, and P M intersects ω for the second time in
Q. Tangent to ω at point Q intersects AC in K. Prove that ∠P KC = 90◦ .

P roposed by Davood V akili

−−−−−−−−−−−−−−−−−−−−−−−−−−−−−−−−−−−−−−−−−
Solution.
Suppose that AB < AC. It’s enough to show that P K k AB.
AQ PQ MB PB
4P M A ∼ 4P AQ ⇒ = , 4P M B ∼ 4P CQ ⇒ =
MA PA QC PQ
AC PA
4P BA ∼ 4P AC ⇒ =
BA PB
We know that M A = M B, so according to above three equations we can say
that:
AQ BA
= (1)
QC AC

KA KQ AQ KA AQ 2
4KAQ ∼ 4KQC ⇒ = = ⇒ =( ) (2)
KQ KC QC KC QC
PB PA BA PB BA 2
4P BA ∼ 4P AC ⇒ = = ⇒ =( ) (3)
PA PC AC PC AC
KA PB
(1), (2), (3) ⇒ = ⇒ P K k AB
KC PC
16

The solution is the same in case of AB > AC.


17

5. Let the circles ω and ω 0 intersect in points A and B. Tangent to circle ω at A


intersects ω 0 in C and tangent to circle ω 0 at A intersects ω in D. Suppose that
the internal bisector of ∠CAD intersects ω and ω 0 at E and F , respectively, and
the external bisector of ∠CAD intersects ω and ω 0 in X and Y , respectively.
Prove that the perpendicular bisector of XY is tangent to the circumcircle of
triangle BEF .

P roposed by M ahdi Etesami F ard

−−−−−−−−−−−−−−−−−−−−−−−−−−−−−−−−−−−−−−−−−
Solution.
Suppose that P is the intersection point of XE and Y F . We know that:
∠EXA = ∠EAC = ∠EAD = ∠F Y A = α ⇒ P X = P Y
∠ABE = ∠EXA = α , ∠ABF = 180◦ − ∠F Y A = 180◦ − α
⇒ ∠EBF = ∠XP Y = 180◦ − 2α ⇒ P EBF : cyclic
EF ⊥XY ⇒ ∠P EF = ∠AEX = ∠AF Y ⇒ PE = PF
We proved that P E = P F and the quadrilateral P EBF is cyclic. Therefore, P
is the midpoint of arc EF in the circumcircle of triangle BEF . Also we know
that the perpendicular bisector of XY is parallel to EF and passes through P .
So the perpendicular bisector of XY is tangent to the circumcircle of triangle
BEF at P .
18

Solutions of 3nd Iranian Geometry Olympiad 2016 (Advanced)

1. Let the circles ω and ω 0 intersect in A and B. Tangent to circle ω at A


intersects ω 0 in C and tangent to circle ω 0 at A intersects ω in D. Suppose that
CD intersects ω and ω 0 in E and F , respectively (assume that E is between F
and C). The perpendicular to AC from E intersects ω 0 in point P and perpen-
dicular to AD from F intersects ω in point Q (The points A, P and Q lie on
the same side of the line CD). Prove that the points A, P and Q are collinear.

P roposed by M ahdi Etesami F ard

−−−−−−−−−−−−−−−−−−−−−−−−−−−−−−−−−−−−−−−−−
Solution.
We know that:
∠AF C = ∠AED = 180◦ − ∠CAD , ∠AEF = 180◦ − ∠AQD
⇒ ∠AF D = ∠AQD

So the point Q is the reflection of the point F in the line AD. Similarly we can
say the point P is the reflection of the point E in the line AC. Therefore:
∠DAQ = ∠DAF = ∠ACD , ∠CAP = ∠CAE = ∠CDA
⇒ ∠DAQ + ∠CAD + ∠CAP = ∠ACD + ∠CAD + ∠CDA = 180◦
So the points A, P and Q are collinear.
19

2. In acute-angled triangle ABC, altitude of A meets BC at D, and M is


midpoint of AC. Suppose that X is a point such that ∠AXB = ∠DXM =
90◦ (assume that X and C lie on opposite sides of the line BM ). Show that
∠XM B = 2∠M BC.

P roposed by Davood V akili

−−−−−−−−−−−−−−−−−−−−−−−−−−−−−−−−−−−−−−−−−
First solution.
Let N be the midpoint of side AB. So M N k BC and ∠M BC = ∠N M B.
Therefore it’s enough to show that the line M N is the bisector of ∠XM B.

∠ADB = ∠AXB = 90◦ ⇒ AXDB : cyclic

⇒ ∠BXD = ∠BAD = 90◦ −∠ABC ⇒ ∠BXM = 180◦ −∠ABC = ∠BN M


⇒ BN XM ; cyclic , AN = N X = BN ⇒ ∠BM N = ∠XM N
20

Second solution.
Let P be the intersection point of XM and BC. Suppose that Q is the point
such that the quadrilateral ADBQ be a rectangle. We know that:

∠DXP = ∠ADP = 90◦ ⇒ ∠ADX = ∠XP D

Also we know that AXDBQ is cyclic, so:

∠ADX = ∠AQX ⇒ ∠AQX = ∠XP D

So Q, X and P are collinear because AQ k BP .

AM = M C and AQ k BP ⇒ QM = M P

Now we know that ∠QBC = 90◦ , thus:

QM = BM = M P ⇒ ∠XM B = 2∠M BC
21

3. In a convex qualrilateral ABCD, let P be the intersection point of AC and


BD. Suppose that I1 and I2 are the incenters of triangles P AB and P DC
respectively. Let O be the circumcenter of P AB, and H the orthocenter of
P DC. Show that the circumcircles of triangles AI1 B and DHC are tangent
together if and only if the circumcircles of triangles AOB and DI2 C are tangent
together.

P roposed by Hooman F attahimoghaddam

−−−−−−−−−−−−−−−−−−−−−−−−−−−−−−−−−−−−−−−−−
Solution.
Suppose that the circumcircles of triangles AI1 B and DHC is tangent together
at point K. Let Q be the second intersection point of circumcircles of triangles
AKD and BKC. we know that:

∠DHC = ∠DKC = 180◦ − ∠P

∠P + ∠P DK + ∠P CK = ∠DKC ⇒ ∠P DK + ∠P CK = 180◦ − 2∠P


AQKD : cyclic ⇒ ∠AQK = 180◦ − ∠P DK
BQKC : cyclic ⇒ ∠BQK = 180◦ − ∠P CK
⇒ ∠AQB = ∠P DK+∠P CK = 180◦ −2∠P = 180◦ −∠AOB ⇒ AOBQ : cyclic
Also we have ∠AKD = ∠AQD , ∠BKC = ∠BQC and ∠AQB = ∠DKC −∠P .
So ∠CQD = ∠AKB + ∠P = 180◦ − ∠AI1 B + ∠P = 90◦ + ∠P
2 = ∠CI2 D.
22

So the qudrilateral CDQI2 is cyclic. So we have to show that circumcircles of


triangles AOB and DI2 C is tangent together at the point Q. It’s enough to
show that:
∠ABQ + ∠DCQ = ∠AQD
We know that the circumcircles of triangles AI1 B and DHC are tangent to-
gether at the point K, so we have:

∠ABK + ∠DCK = ∠AKD

⇒ (∠ABQ + ∠KBQ) + (∠DCQ − ∠KCQ) = ∠AKD


We know that ∠KBQ = ∠KCQ and ∠AKD = ∠AQD, So:

∠ABQ + ∠DCQ = ∠AQD

Therefore the circumcircles of triangles AOB and DI2 C are tangent together
at point Q.
On the other side of the problem, Suppose that the circumcircles of triangles
CI2 D and AOB are tangent together at point Q. Let the point K be the second
intersection of circumcircles of triangles AQD and BQC. Similarly we can show
that the circumcircles of triangles AI1 B and DHC are tangent together at the
point K.

−−−−−−−−−−−−−−−−−−−−−−−−−−−−−−−−−−−−−−−−−

Comment.
Also there is another solution using inversion with respect to a circle with
Michel’s point of the quadrilateral as its center.

−−−−−−−−−−−−−−−−−−−−−−−−−−−−−−−−−−−−−−−−−
23

4. In a convex quadrilateral ABCD, the lines AB and CD meet at point E


and the lines AD and BC meet at point F . Let P be the intersection point
of diagonals AC and BD. Suppose that ω1 is a circle passing through D and
tangent to AC at P . Also suppose that ω2 is a circle passing through C and
tangent to BD at P . Let X be the intersection point of ω1 and AD, and Y
be the intersection point of ω2 and BC. Suppose that the circles ω1 and ω2
intersect each other in Q for the second time. Prove that the perpendicular
from P to the line EF passes through the circumcenter of triangle XQY .

P roposed by Iman M aghsoudi

−−−−−−−−−−−−−−−−−−−−−−−−−−−−−−−−−−−−−−−−−
First solution.

Lemma 1. In the convex quadrilateral ABCD, the lines AB and CD meet


at point E and the lines AD and BC meet at point F . Let point P be the
intersection of AC and BD. Suppose that X and Y be two arbitrary points on
AD and BC, respectively. If BC ∩ P X = U and AD ∩ P Y = V , then the lines
XY , U V and EF are concurrent.
proof.
Let point Z be the intersection of XY and U V . Suppose that P F ∩ U V = L
and P F ∩ CD = K. We know that:

(Z, L, V, U ) = −1 , (E, K, D, C) = −1

If ZF intersects CD in E 0 , so we can say that (E 0 , K, D, C) = −1. Therefore


E ≡ E 0 , so the lines XY , U V and EF are concurrent.
24

Lemma 2. In the cyclic quadrilateral ABCD with circumcenter O, the lines


AB and CD meet at point E and the lines AD and BC meet at point F . If
point P be the intersection of AC and BD, then P O⊥EF .
proof.
Let ω be the circumcircle of quadrilateral ABCD. Suppose that point R is the
intersection of tangents to circle ω at A and C, and point S is the intersection
of tangents to circle ω at B and D.
According to Pascal’s theorem in Hexagonal AABCCD and ABBCDD, we can
say that points R and S lie on line EF .

We know that polar of the point R with respect to circle ω passes through P .
So polar of the point P with respect to circle ω passes through R. Similarly,
we can say that polar of the point P with respect to circle ω passes through S.
Therefore polar of the point P with respect to circle ω is EF . So P O⊥EF .
25

Suppose that P X intersects BC in point U , and P Y intersects AD in point V .

∠XQP = α ⇒ ∠XDP = ∠XP A = ∠U P C = α

∠Y QP = θ ⇒ ∠Y CP = ∠Y P B = ∠V P D = θ

⇒ ∠XV Y = ∠XQY = ∠XU Y = α + θ ⇒ QV XY U : cyclic

Let point O be the circumcenter of QV XY U . According to lemma 1, we can


say that XY , U V and EF are concurrent at point Z. Now according to lamme
2, we can say that P O⊥EF . So the perpendicular from P to EF passes through
the circumcircle of triangle XQY .
26

Second solution.
Suppose that point O is the circumcenter of triangle XQY . The inversion with
respect to a circle with center P trasnforms the problem into this figure. Suppose
that X 0 is the inversion of point X wrt P . We have to show that the line P O0
is the diameter of circumcircle of triangle E 0 P F 0 . Let O00 be the circumcenter
of triangle X 0 Q0 Y 0 . We know that the points P , O0 and O00 are collinear. So
we have to show that he line P O00 passes through the circumcenter of triangle
E0P F 0.

Suppose that O1 , O2 , O3 and O4 are the centers of circles in the above figure
and K be the intersection point of O1 O3 and O2 O4 . We know that point K
lies on perpendicular bisector of P E 0 and P F 0 , thus K is the circumcenter of
triangle P E 0 F 0 . So we have to show that P , K and O00 are collinear. On the
other hand, we know that the quadrilateral D0 B 0 Y 0 Q0 is isosceles trapezoid. So
the point O00 lies on perpendicular bisector of B 0 D0 . Similarly, the point O00 lies
on perpendicular bisector of A0 C 0 . Therefore, the point O00 is the intersection
of A0 C 0 and B 0 D0 .
27

Suppose that:
A0 C 0 ∩ O1 O2 = M , A0 C 0 ∩ O3 O4 = T
B 0 D0 ∩ O2 O3 = N , B 0 D0 ∩ O1 O4 = L
Let points U and V be on A0 C 0 such that KU ⊥A0 C 0 and O00 V ⊥A0 C 0 . Also let
points R and S be on B 0 D0 such that KR⊥B 0 D0 and O00 S⊥B 0 D0 .

We know that O1 O2 and O3 O4 are perpendicular to A0 C 0 . So O1 O2 k O3 O4


Similarly O2 O3 k O1 O4 , therefore the quadrilateral O1 O2 O3 O4 is a parallelo-
gram. It means that the point K lies on the midpoint of the segments O1 O3
and O2 O4 . So U M = U T . Also we have A0 M = P M and C 0 T = P T

⇒ P V = A0 V − A0 P = (P M + P T ) − 2P M = P T − P M

⇒ TV = PT − PV = PM ⇒ UP = UV
Similarly, we can show that RP = RS, so point K lies on the perpendicular
bisector of P V and P S. It means that K is the citcumcenter of triangle P SV .
Therefore the points P , K and O00 are collinear.
28

5. Do there exist six points X1 , X2 , Y1 , Y2 , Z1 , Z2 in the plane such that all of


the triangles Xi Yj Zk are similar for 1 ≤ i, j, k ≤ 2.

P roposed by M orteza Saghaf ian

−−−−−−−−−−−−−−−−−−−−−−−−−−−−−−−−−−−−−−−−−
Solution. (by Ilya Bogdanov f rom Russia)
Suppose a triangle XY Z, in such a way that XY = 1, Y Z = t2 , ZX = t3 and
∠Z = ∠X + 2∠Y .

Such a triangle exists, because for the minimum possible value of t, we have
∠Z > ∠X + 2∠Y and for t = 1 we have ∠Z < ∠X + 2∠Y . So there exists a
triangle with the above properties. Now consider the following 6 points, these
points have the properties of the problem.
29

So there exist the points X1 , X2 , Y1 , Y2 , Z1 , Z2 in the plane such that Xi Yj Zk


be the similar triangles for all of 1 ≤ i, j, k ≤ 2
4th Iranian Geometry Olympiad

Problems and Solutions - 2017


This booklet is prepared by Hirad Aalipanah, Iman Maghsoudi.
With special thanks to Morteza Saghafian, Mahdi Etesami Fard,
Davood Vakili, Hooman Fattahimoghaddam.
Copyright c Young Scholars Club 2017. All rights reserved.
Ministry of education, Islamic Republic of Iran.
www.ysc.ac.ir - www.igo-official.ir

1
Problems of 4th Iranian Geometry Olympiad 2017 (Elementary)

1. Each side of square ABCD with side length of 4 is divided into equal parts
by three points. Choose one of the three points from each side, and connect the
points consecutively to obtain a quadrilateral. Which numbers can be the area of
this quadrilateral? Just write the numbers without proof.

P roposed by Hirad Aalipanah

2. Find the angles of triangle ABC.

P roposed by M orteza Saghaf ian

3. In the regular pentagon ABCDE, the perpendicular at C to CD meets AB


at F . Prove that AE + AF = BE.

P roposed by Alireza Cheraghi

2
4. P1 , P2 ,..., P100 are 100 points on the plane, no three of them are collinear. For
each three points, call their triangle clockwise if the increasing order of them is in
clockwise order. Can the number of clockwise triangles be exactly 2017?

P roposed by M orteza Saghaf ian

5. In the isosceles triangle ABC (AB = AC), let l be a line parallel to BC through
A. Let D be an arbitrary point on l. Let E, F be the feet of perpendiculars through
A to BD, CD respectively. Suppose that P , Q are the images of E, F on l. Prove
that AP + AQ ≤ AB.

P roposed by M orteza Saghaf ian

3
Problems of 4th Iranian Geometry Olympiad 2017 (Intermediate)

1. Let ABC be an acute-angled triangle with A = 60◦ . Let E, F be the feet of


altitudes through B, C respectively. Prove that CE − BF = 23 (AC − AB).

P roposed by F atemeh Sajadi

2. Two circles ω1 , ω2 intersect at A, B. An arbitrary line through B meets ω1 ,


ω2 at C, D respectively. The points E, F are chosen on ω1 , ω2 respectively so that
CE = CB, BD = DF . Suppose that BF meets ω1 at P , and BE meets ω2 at Q.
Prove that A, P , Q are collinear.

P roposed by Iman M aghsoudi

3. On the plane, n points are given (n > 2). No three of them are collinear. Through
each two of them the line is drawn, and among the other given points, the one nearest
to this line is marked (in each case this point occurred to be unique). What is the
maximal possible number of marked points for each given n?

P roposed by Boris F renkin (Russia)

4. In the isosceles triangle ABC (AB = AC), let l be a line parallel to BC through
A. Let D be an arbitrary point on l. Let E, F be the feet of perpendiculars through
A to BD, CD respectively. Suppose that P , Q are the images of E, F on l. Prove
that AP + AQ ≤ AB.

P roposed by M orteza Saghaf ian

5. Let X, Y be two points on the side BC of triangle ABC such that 2XY = BC.
(X is between B, Y ) Let AA0 be the diameter of the circumcircle of triangle AXY .
Let P be the point where AX meets the perpendicular from B to BC, and Q be
the point where AY meets the perpendicular from C to BC. Prove that the tangent
line from A0 to the circumcircle of AXY passes through the circumcenter of triangle
AP Q.

P roposed by Iman M aghsoudi

4
Problems of 4th Iranian Geometry Olympiad 2017 (Advanced)

1. In triangle ABC, the incircle, with center I, touches the side BC at point D.
Line DI meets AC at X. The tangent line from X to the incircle (different from AC)
intersects AB at Y . If Y I and BC intersect at point Z, prove that AB = BZ.

P roposed by Hooman F attahimoghaddam

2. We have six pairwise non-intersecting circles that the radius of each is at least
one. Prove that the radius of any circle intersecting all the six circles, is at least one.

P roposed by M ohammad Ali Abam− M orteza Saghaf ian

3. Let O be the circumcenter of triangle ABC. Line CO intersects the altitude


through A at point K. Let P , M be the midpoints of AK, AC respectively. If P O
intersects BC at Y , and the circumcircle of triangle BCM meets AB at X, prove
that BXOY is cyclic.

P roposed by Ali Daeinabi − Hamid P ardazi

4. Three circles ω1 , ω2 , ω3 are tangent to line l at points A, B, C (B lies between


A, C) and ω2 is externally tangent to the other two. Let X, Y be the intersection
points of ω2 with the other common external tangent of ω1 , ω3 . The perpendicular
line through B to l meets ω2 again at Z. Prove that the circle with diameter AC
touches ZX, ZY .

P roposed by Iman M aghsoudi − Siamak Ahmadpour

5. Sphere S touches a plane. Let A, B, C, D be four points on this plane such


that no three of them are collinear. Consider the point A0 such that S is tangent to
the faces of tetrahedron A0 BCD. Points B 0 , C 0 , D0 are defined similarly. Prove that
A0 , B 0 , C 0 , D0 are coplanar and the plane A0 B 0 C 0 D0 touches S.

P roposed by Alexey Zaslavsky (Russia)

5
Solutions of 4th Iranian Geometry Olympiad 2017 (Elementary)

1. Each side of square ABCD with side length of 4 is divided into equal parts
by three points. Choose one of the three points from each side, and connect the
points consecutively to obtain a quadrilateral. Which numbers can be the area of
this quadrilateral? Just write the numbers without proof.

P roposed by Hirad Aalipanah


−−−−−−−−−−−−−−−−−−−−−−−−−−−−−−−−−−−−−−−−−−−−−
Solution.

To find the area of the quadrilaterals, it’s enough to find sum of the areas of four
right triangles and subtract it from area of the square. Finally, these numberes will
be found:
6, 7, 7.5, 8, 8.5, 9, 10

6
2. Find the angles of triangle ABC.

P roposed by M orteza Saghaf ian


−−−−−−−−−−−−−−−−−−−−−−−−−−−−−−−−−−−−−−−−−−−−−
Solution.

Let ∠ACB = x. The quadrilateral with equal sides in triangle ABC is a rhom-
bus so it has parallel sides.

According to the angles shown in the figure, we can say that sum of the angles of
triangle ABC is equal to 180◦ .
x
(90◦ − ) + 2x + x = 180◦ ⇒ x = 36◦
2
⇒ ∠A = 72◦ , ∠B = 72◦ , ∠C = 36◦

7
3. In the regular pentagon ABCDE, the perpendicular at C to CD meets AB at F .
Prove that AE + AF = BE.

P roposed by Alireza Cheraghi


−−−−−−−−−−−−−−−−−−−−−−−−−−−−−−−−−−−−−−−−−−−−−
Solution.

Suppose that P is the intersection of AE and F C. We know that:

∠ECD = 36◦ ⇒ ∠ECP = 54◦ , ∠AEC = 72◦ ⇒ ∠EP C = 54◦

Therefore, CE = P E. On the other hand, we know that ∠ECB = ∠EBC = 72◦ .


Therefore, BE = CE = P E. Also we have ∠EAB = 108◦ so ∠AF P = ∠AP F = 54◦ .
It means that AF = AP .

⇒ AE + AF = AE + AP = P E = CE = BE 

8
4. P1 , P2 ,..., P100 are 100 points on the plane, no three of them are collinear. For
each three points, call their triangle clockwise if the increasing order of them is in
clockwise order. Can the number of clockwise triangles be exactly 2017?

P roposed by M orteza Saghaf ian


−−−−−−−−−−−−−−−−−−−−−−−−−−−−−−−−−−−−−−−−−−−−−
Solution.
First, suppose that P1 , P2 ,..., P100 are located on a circle in a counter-clockwise order.
In this case, the number of clockwise triangles is zero. Now, start to move the points,
when a point Pi passes through a line Pj Pk , the state of triangle Pi Pj Pk (clockwise
or counter-clockwise) changes and the state of the other triangles does not change
(assume that the points move in such a way that no point passes through two lines
at the same time). So the number of clockwise triangles changes by one. Suppose
that the points P1 , P2 ,..., P100 move to obtain their
 clockwise order on a circle. In this
100
case, the number of clockwise triangles is 3 which is greater than 2017. In this
process, the number of clockwise triangles changes one by one. So there is a moment
in which the number of clockwise triangles is exactly 2017. 

9
5. In the isosceles triangle ABC (AB = AC), let l be a line parallel to BC through
A. Let D be an arbitrary point on l. Let E, F be the feet of perpendiculars through
A to BD, CD respectively. Suppose that P , Q are the images of E, F on l. Prove
that AP + AQ ≤ AB.

P roposed by M orteza Saghaf ian


−−−−−−−−−−−−−−−−−−−−−−−−−−−−−−−−−−−−−−−−−−−−−
Solution1.

Suppose that M , N are the midpoints of AB , AC respectively. We have:


AB AC
∠AEB = ∠AF C = 90◦ ⇒ ME = NF = =
2 2
Let X, Y be the feet of perpendiculars through M , N to line l respectively. We know
that two triangles AM A and AN Y are equal so AX = AY .
AB AC
XP = AX + AP ≤ M E = , AQ − AY = Y Q ≤ N F =
2 2
⇒ AP + AQ ≤ AB 

10
Solution2.

Let P 0 , E 0 , be the reflection of P , E with respect to the perpendicular bisector


of BC. Therefore, we have ∠AE 0 C = ∠AP 0 E = 90◦ and AP 0 = AP . We have:

AP + AQ = AP 0 + AQ = QP 0 ≤ F E 0 ≤ AC = AB 

11
Solutions of 4th Iranian Geometry Olympiad 2017 (Intermediate)

1. Let ABC be an acute-angled triangle with A = 60◦ . Let E, F be the feet of


altitudes through B, C respectively. Prove that CE − BF = 23 (AC − AB).

P roposed by F atemeh Sajadi


−−−−−−−−−−−−−−−−−−−−−−−−−−−−−−−−−−−−−−−−−−−−−
Solution.

AB AC
∠A = 60◦ ⇒ ∠ABF = ACF = 30◦ ⇒ AE = , AF =
2 2
⇒ CE − BF = (AC − AE) − (AB − AF )
3
= (AC − AB) + (AF − AE) = (AC − AB) 
2

12
2. Two circles ω1 , ω2 intersect at A, B. An arbitrary line through B meets ω1 , ω2
at C, D respectively. The points E, F are chosen on ω1 , ω2 respectively so that
CE = CB, BD = DF . Suppose that BF meets ω1 at P , and BE meets ω2 at Q.
Prove that A, P , Q are collinear.

P roposed by Iman M aghsoudi


−−−−−−−−−−−−−−−−−−−−−−−−−−−−−−−−−−−−−−−−−−−−−
Solution.

We know that:
∠BF D = ∠DBF = 180◦ − ∠CBP = ∠CEP
⇒ ∠CEB + ∠BEP = ∠BF Q + ∠QF D
∠CEB = ∠CBE = ∠QBD = ∠QF D
⇒ ∠BEP = ∠BF Q ⇒ ∠BAP = ∠BEP = ∠BF Q = ∠BAQ
So A, P , Q are collinear. 

13
3. On the plane, n points are given (n > 2). No three of them are collinear. Through
each two of them the line is drawn, and among the other given points, the one nearest
to this line is marked (in each case this point occurred to be unique). What is the
maximal possible number of marked points for each given n?

P roposed by Boris F renkin (Russia)

−−−−−−−−−−−−−−−−−−−−−−−−−−−−−−−−−−−−−−−−−−−−−
Solution.

The case n = 3 is obvious. For n > 4, take a regular n-gon and slightly deform
it to make the assumptions of the problem valid. Each vertex is the nearest to the
line connecting two adjacent vertices (this is directly proved by computing angles).
If n = 4 then two cases are possible.

1) The given points form a convex quadrilateral (say ABCD). Since for each side
the nearest vertex is unique, no two sides are parallel. Let lines AD and BC meet
beyond A and B, and lines AB and CD meet beyond B and C respectively. Then
points A, B and C are marked. Suppose D is marked as well. Then it is the nearest
vertex to AC, and the area of 4 ACD is less than that of 4 ACB. On the other
hand, draw the line through C parallel to AD. Let it meet AB at point E. Then
CE < AD, so S4 ACD > S4 ACE > S4 ACB , a contradiction.

2) One of the given points (say O) is inside the triangle ABC formed by the re-
maining points. Obviously O is nearest of the given points to lines AB, BC and AC.
Without loss of generality, the nearest point to line AO is B. If the nearest point
to line BO is A then C cannot be marked. Suppose the nearest point to line BO is
C. If the remaining point A is also marked then it is nearest to line CO. Let line
AO intersect BC at point A1 , and let points B1 , C1 be defined similarly. We have
S4 AA1 B < S4 AA1 C , hence A1 B < A1 C. Similarly B1 C < B1 A, C1 A < C1 B. Then
A1 B · B1 C · C1 A < A1 C · B1 A · C1 B, a contradiction with Ceva theorem. 

−−−−−−−−−−−−−−−−−−−−−−−−−−−−−−−−−−−−−−−−−−−−−
Comment.

In fact, it is not necessary to apply Ceva theorem. Let MA , MB , MC be the bases


of the corresponding medians, and M be the centroid. Then O must belong to the
interior of each of triangles M MA B, M MB C, M MC A which is impossible.

14
4. In the isosceles triangle ABC (AB = AC), let l be a line parallel to BC through
A. Let D be an arbitrary point on l. Let E, F be the feet of perpendiculars through
A to BD, CD respectively. Suppose that P , Q are the images of E, F on l. Prove
that AP + AQ ≤ AB.

P roposed by M orteza Saghaf ian


−−−−−−−−−−−−−−−−−−−−−−−−−−−−−−−−−−−−−−−−−−−−−
Solution1.

Suppose that M , N are the midpoints of AB , AC respectively. We have:


AB AC
∠AEB = ∠AF C = 90◦ ⇒ ME = NF = =
2 2
Let X, Y be the feet of perpendiculars through M , N to line l respectively. We know
that two triangles AM A and AN Y are equal so AX = AY .
AB AC
XP = AX + AP ≤ M E = , AQ − AY = Y Q ≤ N F =
2 2
⇒ AP + AQ ≤ AB 

15
Solution2.

Let P 0 , E 0 , be the reflection of P , E respect to the perpendicular bisector of BC.


Therefore, we have ∠AE 0 C = ∠AP 0 E = 90◦ and AP 0 = AP . We have:

AP + AQ = AP 0 + AQ = QP 0 ≤ F E 0 ≤ AC = AB 

16
5. Let X, Y be two points on the side BC of triangle ABC such that 2XY = BC.
(X is between B, Y ) Let AA0 be the diameter of the circumcircle of triangle AXY .
Let P be the point where AX meets the perpendicular from B to BC, and Q be
the point where AY meets the perpendicular from C to BC. Prove that the tangent
line from A0 to the circumcircle of AXY passes through the circumcenter of triangle
AP Q.

P roposed by Iman M aghsoudi


−−−−−−−−−−−−−−−−−−−−−−−−−−−−−−−−−−−−−−−−−−−−−
Solution.

Let O, M , N be the circumcenter of triangle AP Q, midpoint of AP , midpoint of AQ


respectively. We know that ∠OM A = ∠ON A = 90◦ so the quadrilateral AM ON is
cyclic. We have to prove ∠OA0 A = 90◦ . Therefore, we have to show that AM OA0 N
is cyclic. It’s enought to show that the quadrilateral AM A0 N is cyclic. We should
prove ∠A0 XM = ∠A0 N Y . We will show that 4A0 N Y ∼ 4A0 XM .

We know that the quadrilateral AXA0 Y is cyclic so ∠A0 XM = ∠A0 Y N . Therefore,


it’s enough to show that:
A0 X A0 Y
= (1)
MX NY

17
Let H, K, M 0 , N 0 be the feet of perpendiculars through A, A0 , M , N to BC respec-
tively. We know that M , N are the midpoints of AP , AQ so we have:
BH
BM 0 = HM 0 ⇒ XM 0 = HM 0 − XH = − XH (2)
2
CH BC − BH
CN 0 = HN 0 ⇒ Y N 0 = HY − HN 0 = HY − = (XY − XH) −
2 2
BC BH BH
⇒ Y N 0 = (XY − )+( − XH) = − XH (3)
2 2 2

According to the equations (2), (3) we can say that XM 0 = Y N 0 . On the other hand,
we have AA0 is the diameter of the circumcircle of triangle AXY . Therefore:

∠A0 Y A = ∠A0 XA = 90◦ ⇒ ∠KA0 Y = ∠N Y N 0 , ∠KA0 X = ∠M XM 0

⇒ 4A0 KX ∼ 4XM 0 M , 4A0 KY ∼ 4Y N 0 N


A0 X A0 K A0 K A0 Y A0 X A0 Y
⇒ = , = ⇒ =
MX XM 0 NY 0 NY MX NY
So equation (1) is proved. 

18
Solutions of 4th Iranian Geometry Olympiad 2017 (Advanced)

1. In triangle ABC, the incircle, with center I, touches the side BC at point D.
Line DI meets AC at X. The tangent line from X to the incircle (different from AC)
intersects AB at Y . If Y I and BC intersect at point Z, prove that AB = BZ.

P roposed by Hooman F attahimoghaddam


−−−−−−−−−−−−−−−−−−−−−−−−−−−−−−−−−−−−−−−−−−−−−
Solution.

In triangle AXY , the point I is excenter so ∠XIY = 90◦ − A2 .

A A
⇒ ∠DIZ = 90◦ − ⇒ ∠IZB = = ∠BAI
2 2
Also we know that ∠IBZ = ∠IBA and BI = BI so two triangles ABI and ZBI are
equal. Thus, AB = BZ. 

19
2. We have six pairwise non-intersecting circles that the radius of each is at least one.
Prove that the radius of any circle intersecting all the six circles, is at least one.

P roposed by M ohammad Ali Abam− M orteza Saghaf ian

−−−−−−−−−−−−−−−−−−−−−−−−−−−−−−−−−−−−−−−−−−−−−
Solution.

Call the centers of these six circles O1 , O2 ,..., O6 , and their radii R1 , R2 ,..., R6 . Sup-
pose that a circle with center O and radius R intersects these six circles. Obviously
there exist i, j such that Oi OOj < 60◦ . The length of Oi Oj is at least Ri + Rj and the
lengths of OOi , OOj are less than or equal to Ri + R, Rj + R, respectively. If R < 1
then considering Ri , Rj > 1 we conclude that in triangle Oi OOj the longest side is
Oi Oj , so Oi OOj ≥ 60◦ which yields a contradiction. So we must have R ≥ 1. 

20
3. Let O be the circumcenter of triangle ABC. Line CO intersects the altitude
through A at point K. Let P , M be the midpoints of AK, AC respectively. If P O
intersects BC at Y , and the circumcircle of triangle BCM meets AB at X, prove
that BXOY is cyclic.

P roposed by Ali Daeinabi − Hamid P ardazi


−−−−−−−−−−−−−−−−−−−−−−−−−−−−−−−−−−−−−−−−−−−−−
Solution.

We want to show that ∠XOP = ∠B. We know that:

∠B = ∠XM A ⇒ ∠XM O = 90◦ − ∠B = ∠XAK

We will prove that two triangles XOP and XM A are similar. It’s enough to show
that two triangles XP A and XOM are similar. We have ∠XM O = ∠XAK so it’s
enough to prove:
AX AP
=
XM OM

21
The quadrilateral BXM C is cyclic. Therefore 4AXM ∼ 4ACB.
AX AC
⇒ = (1)
XM BC

The point O is the circumcenter of triangle ABC. Thus, ∠OCA = 90◦ − ∠B and
∠AKC = 180◦ − ∠A. According to law of sines in triangles AKC, OM C and ABC
we can say that:
AC AK OM
= ◦
, OC =
sin∠A sin(90 − ∠B) sin(90◦ − ∠B)

AK AC AP AC AC
⇒ = ⇒ = = (2)
OM OC.sin∠A OM 2OC.sin∠A BC

According to equations (1) and (2), we can conclude that two triangles XP A and
XOM are similar. Therefore two triangles XOP and XM A are similar so:

∠XBP = ∠XM A = ∠B 

22
4. Three circles ω1 , ω2 , ω3 are tangent to line l at points A, B, C (B lies between
A, C) and ω2 is externally tangent to the other two. Let X, Y be the intersection
points of ω2 with the other common external tangent of ω1 , ω3 . The perpendicular
line through B to l meets ω2 again at Z. Prove that the circle with diameter AC
touches ZX, ZY .

P roposed by Iman M aghsoudi − Siamak Ahmadpour


−−−−−−−−−−−−−−−−−−−−−−−−−−−−−−−−−−−−−−−−−−−−−
Solution.

Let S be the intersection of XY and l. Suppose that ω1 , ω2 are tangent to each


other at point E and ω2 , ω3 are tangent to each other at point F . We know that S is
the external homothetic center for ω1 , ω3 and E, F are antihomologous points so E,
F , S are collinear. Suppose that XY meets ω1 , ω2 at P , Q respectively so we have:

SE · SF = SP · SQ = SA · SC

Therefore, the quadrilaterals P EF Q and AEF C are cyclic. Let T be the midpoint
of arc XY in circle ω2 (the arc which has not Z).We know that the line tangent to
ω2 at T is parallel to XY so T , E, P are collinear. Similarly T , Q, F are collinear.

TE · TP = TF · TQ ⇒ Pω1 (T ) = Pω3 (T ) (1)


On the other hand, the line which is tangent to ω2 at Z is parallel to l so Z, E, A
are collinear. Similarly Z, F , C are collinear.

ZE · ZA = ZF · ZC ⇒ Pω1 (Z) = Pω3 (Z) (2)

23
According to the equations (1), (2) we can say that ZT is the radical axis of ω1 , ω3 .
Therefore, if M is the midpoint of AC then Z, T , M are collinear because M lies on
radical axis of ω1 , ω3 too. Let D be the intersection of ZM , P Q and H be the feet
of perpendiculars through Z to P Q. We know that:

|∠ZXY − ∠ZY X|
∠HZD = ∠M ZB = =α
2

Let x = ZY , y = ZX, t = XY . We can say that:

ZB ZB ZB ZB 2xy (∠XZY )
AM = = ZH = · ZD = · · cos
cos α ZD
ZH ZH x + y 2
To prove the problem, it’s enough to show that the distances of M from two lines
ZX, ZY are equal to AC2
. Thus, we have to show that:
∠XZY AC
AM · sin =
2 2
(∠XZY ) ZB 2xy
⇐⇒ 2AM · sin = · · sin (∠XZY ) = AC
2 ZH x + y
On the other hand, we have:

1 1 xy sin (∠XZY )
S4XY Z = t · ZH = xy sin (∠XZY ) ⇒ t=
2 2 ZH
So it’s enough to show that:
2t · ZB
AC =
x+y

24
We know that:

∠BF Z = ∠BEZ = 90◦ ⇒ ZB 2 = ZE · ZA = ZF · ZC

Therefore, lenghts of tangents from Z to ω1 , ω3 are equal to ZB. Now, According


to Casey’s theorem (generalization of Ptolemy’s theorem) in two cyclic quadrilaterals
ZY ω1 X and ZXω3 Y we can say that:

ZY ω1 X : x · P X + y · (t + P X) = t · ZB ⇒ (x + y)P X + yt = t · ZB

ZXω3 Y : y · Y Q + x · (t + Y Q) = t · ZB ⇒ (x + y)Y Q + xt = t · ZB

2t · ZB 2t · ZB
⇒ PX + Y Q + t = ⇒ AC = 
x+y x+y

25
5. Sphere S touches a plane. Let A, B, C, D be four points on this plane such that
no three of them are collinear. Consider the point A0 such that S is tangent to the
faces of tetrahedron A0 BCD. Points B 0 , C 0 , D0 are defined similarly. Prove that A0 ,
B 0 , C 0 , D0 are coplanar and the plane A0 B 0 C 0 D0 touches S.

P roposed by Alexey Zaslavsky (Russia)


−−−−−−−−−−−−−−−−−−−−−−−−−−−−−−−−−−−−−−−−−−−−−
Solution.

Let S touch the plane at point P . D is the point of concurrency of the planes
passing through AB, BC, CA and touching S (similarly for A0 , B 0 , C 0 ). Suppose
that the plane passing through X, Y and touching S, touches it at Pxy . The points
P , Pab , Pac , Pad are on a circle Wa , because when we connect them to P , the resulted
line touches S. Similarly we can find other triples which are concyclic with P , name
the circles with Wb , Wc , Wd . Now use inversion with center P and arbitrary radius,
we get 4 lines with three points on each line. Considering the concurrency of four
circles at Michels Point we conclude that the circles Pab Pbc Pca , Pab Pbd Pda , Pac Pcd Pda ,
Pbc Pcd Pdb (call them Wd0 , Wc0 , Wb0 , Wa0 ) are concurrent on S at a point P 0 . Wd0 is
the locus of feet of tangencies from D0 to S, so D0 P 0 is tangent to S and D0 is on
the tangent plane at P 0 to S. Similarly we conclude that A0 , B 0 , C 0 are also on the
tangent plane at P 0 to S. So A0 , B 0 , C 0 , D0 are coplanar and this plane is tangent to
S. 

26
The Fifth Iranian Geometry
Olympiad

The problems along with their solutions


Contents

Elementary Level 2
Problems 2

Solutions 4

Intermediate Level 11
Problems 11

Solutions 13

Advanced Level 20
Problems 20

Solutions 21
Elementary Level

1
Problems

1. As shown below, there is a 40 × 30 paper with a filled 10 × 5 rectangle inside of it. We want
to cut out the filled rectangle from the paper using four straight cuts. Each straight cut is a
straight line that divides the paper into two pieces, and we keep the piece containing the filled
rectangle. The goal is to minimize the total length of the straight cuts. How to achieve this
goal, and what is that minimized length? Show the correct cuts and write the final answer.
There is no need to prove the answer.

2. Convex hexagon A1 A2 A3 A4 A5 A6 lies in the interior of convex hexagon B1 B2 B3 B4 B5 B6 such


that A1 A2 ∥ B1 B2 , A2 A3 ∥ B2 B3 ,..., A6 A1 ∥ B6 B1 . Prove that the areas of simple hexagons
A1 B2 A3 B4 A5 B6 and B1 A2 B3 A4 B5 A6 are equal. (A simple hexagon is a hexagon which does
not intersect itself.)

3. In the given figure, ABCD is a parallelogram.


√ We

know that ∠D = 60 , AD = 2 and AB = 3 + 1.
Point M is the midpoint of AD. Segment CK is
the angle bisector of C. Find the angle CKB.

2
4. There are two circles with centers O1 , O2 lie inside of circle ω and are tangent to it. Chord AB
of ω is tangent to these two circles such that they lie on opposite sides of this chord. Prove
that ∠O1 AO2 + ∠O1 BO2 > 90◦ .

5. There are some segments on the plane such that no two of them intersect each other (even at
the ending points). We say segment AB breaks segment CD if the extension of AB cuts CD
at some point between C and D.

(a) Is it possible that each segment when extended from


both ends, breaks exactly one other segment from each
way?

(b) A segment is called surrounded if from both sides of


it, there is exactly one segment that breaks it.
(e.g. segment AB in the figure.) Is it possible to have
all segments to be surrounded?

3
Solutions

1. As shown below, there is a 40 × 30 paper with a filled 10 × 5 rectangle inside of it. We want
to cut out the filled rectangle from the paper using four straight cuts. Each straight cut is a
straight line that divides the paper into two pieces, and we keep the piece containing the filled
rectangle. The goal is to minimize the total length of the straight cuts. How to achieve this
goal, and what is that minimized length? Show the correct cuts and write the final answer.
There is no need to prove the answer.

Proposed by Morteza Saghafian


------------------------------------------------------------------
Solution. The answer is 65. Here is an example of the solution:

4
2. Convex hexagon A1 A2 A3 A4 A5 A6 lies in the interior of convex hexagon B1 B2 B3 B4 B5 B6 such
that A1 A2 ∥ B1 B2 , A2 A3 ∥ B2 B3 ,..., A6 A1 ∥ B6 B1 . Prove that the areas of simple hexagons
A1 B2 A3 B4 A5 B6 and B1 A2 B3 A4 B5 A6 are equal. (A simple hexagon is a hexagon which does
not intersect itself.)

Proposed by Mahdi Etesamifard - Hirad Aalipanah


------------------------------------------------------------------
Solution. As you can see, we have divided the area between two polygons into 6 trapezoids.
In each trapezoid is it easy to see that the triangles which have the same area (like B1 A1 A2 and
B2 A1 A2 ) each belongs to one of the simple hexagons. Therefore, if we add up their areas and
add the common area (the area of A1 A2 A3 A4 A5 A6 ) to them, we can conclude that the areas of
the two simple hexagons are equal.

5
3. In the given figure, ABCD is a parallelogram.
√ We
know that ∠D = 60◦ , AD = 2 and AB = 3 + 1.
Point M is the midpoint of AD. Segment CK is
the angle bisector of C. Find the angle CKB.

Proposed by Mahdi Etesamifard


- - - - - - - - - - - - - - - - - - - - - - - - - - - - - - - - - - - - - - - - - - - - - - - - - - - -√
--------------
Solution 1. Let X be a point on AB such that AX = 1 and XB = 3.√ We know that
∠M AX = 120◦ . Therefore by Pythagoras theorem we know that M X = 3. So we have
∠M BX = 15◦ and ∠CBK = 45◦ . Hence, ∠CKB = 180◦ − 60◦ − 45◦ = 75◦ .


Solution 2. Let N be the midpoint of side BC. M N intersects CK at L. It’s clear that
the triangle CN L is equilateral. Therefore, we have LN = CN =√N B. So, BCL is a right-
√ Because of Pythagoras’s theorem we have BL = 3. On the other hand, we
angled triangle.
have M L = 3 and ∠BLN = 30◦ . Because of that, we have ∠LBM = 15◦ and so we have
∠CBK = 30◦ + 15◦ = 45◦ . Hence, ∠CKB = 180◦ − 60◦ − 45◦ = 75◦ .

6
4. There are two circles with centers O1 , O2 lie inside of circle ω and are tangent to it. Chord AB
of ω is tangent to these two circles such that they lie on opposite sides of this chord. Prove
that ∠O1 AO2 + ∠O1 BO2 > 90◦ .
Proposed by Iman Maghsoudi
------------------------------------------------------------------
Solution. Let AC, BC be tangents from A, B to the circle with center O1 and AD, BD be
tangents from A, B to the circle with center O2 . It’s enough to show that ∠CAD + ∠CBD >
180◦ . Or to show that ∠ACB + ∠ADB < 180◦ .
We know that C, D lie on the outside of circle ω. Therefore, we can always say that ∠ACB <
∠AXB and ∠ADB < ∠AY B because of the exterior angles. But we know that ∠AXB +
∠AY B = 180◦ . Hense, we can conclude that ∠ACB + ∠ADB < 180◦ and the statement is
proven.

7
5. There are some segments on the plane such that no two of them intersect each other (even at
the ending points). We say segment AB breaks segment CD if the extension of AB cuts CD
at some point between C and D.

(a) Is it possible that each segment when extended from


both ends, breaks exactly one other segment from each
way?

(b) A segment is called surrounded if from both sides of


it, there is exactly one segment that breaks it.
(e.g. segment AB in the figure.) Is it possible to have
all segments to be surrounded?

Proposed by Morteza Saghafian


------------------------------------------------------------------

8
Solution.

(a) No. Consider the convex hull of the endpoints of these segments. Let A be a vertex of the
convex hull, where AB is one of the segments.

We know that there exist segments CD, EF as in the figure. So A lies inside the convex hull
of C, D, E, F and therefore it cannot be a vertex of the main convex hull. Contradiction!

(b) Yes. The figure below shows that it is possible for all segments to be surrounded.

9
Intermediate
Level

10
Problems

1. There are three rectangles in the following figure. The lengths of some segments are shown.
Find the length of the segment XY .

2. In convex quadrilateral ABCD, the diagonals AC and BD meet at the point P . We know
that ∠DAC = 90◦ and � 2∠ADB = ∠ACB. If we have ∠DBC + 2∠ADC = 180◦ prove that
2AP = BP .

3. Let ω1 , ω2 be two circles with centers O1 and O2 , respectively. These two circles intersect each
other at points A and B. Line O1 B intersects ω2 for the second time at point C, and line O2 A
intersects ω1 for the second time at point D . Let X be the second intersection of AC and ω1 .
Also Y is the second intersection point of BD and ω2 . Prove that CX = DY .

4. We have a polyhedron all faces of which are triangle. Let P be an arbitrary point on one of
the edges of this polyhedron such that P is not the midpoint or endpoint of this edge. Assume
that P0 = P . In each step, connect Pi to the centroid of one of the faces containing it. This
line meets the perimeter of this face again at point Pi+1 . Continue this process with Pi+1 and
the other face containing Pi+1 . Prove that by continuing this process, we cannot pass through
all the faces. (The centroid of a triangle is the point of intersection of its medians.)

11
5. Suppose that ABCD is a parallelogram such that ∠DAC = 90◦ . Let H be the foot of perpen-
dicular from A to DC, also let P be a point along the line AC such that the line P D is tangent
to the circumcircle of the triangle ABD. Prove that ∠P BA = ∠DBH.

12
Solutions

1. There are three rectangles in the following figure. The lengths of some segments are shown.
Find the length of the segment XY .

Proposed by Hirad Aalipanah


------------------------------------------------------------------
Let us continue the rectangular sides to get the ABC triangle. Because AB = BC we can say
that ∠BCA = ∠BAC = 45◦ . Therefore, √ we can√determine some
√ of the segments

using the
2
Pythagoras’s theorem such as AD = 2 2, DX = 2, CE = 2 2 and EY = 2 . So, we have
√ √
√ √ √ √ 2 9 2
XY = AC − AD − DX − CE − EY = 10 2 − 2 2 − 2 − 2 2 − =
2 2

13
14
2. In convex quadrilateral ABCD, the diagonals AC and BD meet at the point P . We know
that ∠DAC = 90◦ and � 2∠ADB = ∠ACB. If we have ∠DBC + 2∠ADC = 180◦ prove that
2AP = BP .
Proposed by Iman Maghsoudi
------------------------------------------------------------------
Solution.

Let M be the intersection point of the angle bisector of ∠P CB with segment P B. Since
∠P CM = ∠P DA = θ and ∠AP D = ∠M P C, we get that △P M C ∼ △P AD, which means
∠P M C = 90◦ .
Now in triangle CP B, the angle bisector of vertex C is the same as the altitude from C, this
means CP B is an isosceles triangle and so P M = M B, P C = CB.
In triangle DBC, we have

\ + 2θ + P
DBC \ \
CD + P DC = 180◦ .

This along with the assumption that ∠DBC + 2∠ADC = 180◦ , implies ∠P CD = ∠P DC.
Therefore P C = P D and so △P M C ∼= △P AD, hence AP = P M = P2B . ■

15
3. Let ω1 , ω2 be two circles with centers O1 and O2 , respectively. These two circles intersect each
other at points A and B. Line O1 B intersects ω2 for the second time at point C, and line O2 A
intersects ω1 for the second time at point D . Let X be the second intersection of AC and ω1 .
Also Y is the second intersection point of BD and ω2 . Prove that CX = DY .
Proposed by Alireza Dadgarnia
------------------------------------------------------------------
Solution. First, we use a well-known lemma.
Lemma. Let P QRS be a convex quadrilateral with RQ = RS, ∠RP Q = ∠RP S and P Q ̸= P S.
Then P QRS is cyclic.
Proof. Assume the contrary, and let P ′ ̸= P be the intersection point of the circle passing
through R, S, Q with line P R.
Since P ′ QRS is cyclic and RQ = RS, we get ∠SP ′ R = ∠QP ′ R. Now let’s considerate
on triangles SP ′ P and QP ′ P . In these two triangles we have ∠SP ′ P = ∠QP ′ P and also
∠P ′ P Q = ∠P ′ P S. This means these two triangles are congruent, hence P Q = P S, which is a
contradiction. So the lemma is proved.
Back to the problem.

Triangles ADY and BXC are similar, because


\ = BXC
ADY \ = 180◦ − BXA,
\

and
\
DY \ = 180◦ − AY
A = BCX [ B.
Note that O2 lies on the angle bisector of ∠AO1 B, O2 A = O2 C and also O1 A ̸= O1 C. So we
can use the lemma and conclude that O1 AO2 C is cyclic. Similarly, we get that O2 BO1 D is
cyclic.
\
AY D = 180◦ − AY[ B=O \ \ \
1 CA = O1 O2 A = O1 BD.

Which means AC ∥ BD and so AY = BC. But since △ADY ∼ △BXC, we get that these
two triangles are congruent and so CX = DY . ■

16
4. We have a polyhedron all faces of which are triangle. Let P be an arbitrary point on one of
the edges of this polyhedron such that P is not the midpoint or endpoint of this edge. Assume
that P0 = P . In each step, connect Pi to the centroid of one of the faces containing it. This
line meets the perimeter of this face again at point Pi+1 . Continue this process with Pi+1 and
the other face containing Pi+1 . Prove that by continuing this process, we cannot pass through
all the faces. (The centroid of a triangle is the point of intersection of its medians.)
Proposed by Mahdi Etesamifard - Morteza Saghafian
------------------------------------------------------------------
Solution. Suppose that AB is the edge that P lies on. Let M be the midpoint of AB
and without loss of generality, assume that P lies between B and M . We will prove that it is
impossible to pass through a face which doesn’t contain A. (Such face exists in any polyhedron)
Let B = B0 , B1 , B2 , . . . be the vertices adjacent to A in this order. Let Mi be the midpoint of
ABi . By using induction, we prove that for each i, Pi lies on edge ABi , between Bi and Mi .
For i = 0 the claim is true. Now assume the claim for i and consider the triangle ABi Bi+1 with
centroid Gi .

Since Pi lies between Mi and Bi , we get that Pi Gi lies between Mi Gi and Bi Gi , which are the
medians of this triangle. So Pi+1 lies on ABi+1 , between Mi+1 and Bi+1 . So the claim is proved.
We proved that Pi ’s lie on ABi ’s, so the sequence of points Pi goes around A and therefore does
not pass through a face which doesn’t contain A. ■

17
5. Suppose that ABCD is a parallelogram such that ∠DAC = 90◦ . Let H be the foot of perpen-
dicular from A to DC, also let P be a point along the line AC such that the line P D is tangent
to the circumcircle of the triangle ABD. Prove that ∠P BA = ∠DBH.
Proposed by Iman Maghsoudi
------------------------------------------------------------------

Suppose that AB, AD meet the circumcircle of triangle P DB for the second time at points
X, Y respectively. Let ∠CDB = α and ∠ADB = θ. Therefore, we have ∠ABD = α, and so
∠ADP = α.
Also ∠P DB = ∠P XB = α + θ, and ∠P AX = ∠ACD = ∠DAH. Which implies
△ △ AP AX
AP X ∼ ADH =⇒ = ,
AH AD
△ △ AY AX
XAD ∼ Y AB =⇒ = ,
AB AD
AP AY
=⇒ = .
AH AB
△ △
Now since ∠HAB = ∠P AY = 90◦ , It can be written that AP Y ∼ AHB.
\ = P[
=⇒ HBA \
YA=P BD =⇒ P[ \
BA = DBH.

18
Advanced Level

19
Problems

1. Two circles ω1 , ω2 intersect each other at points A, B. Let P Q be a common tangent line of
these two circles with P ∈ ω1 and Q ∈ ω2 . An arbitrary point X lies on ω1 . Line AX intersects
ω2 for the second time at Y . Point Y ′ ̸= Y lies on ω2 such that QY = QY ′ . Line Y ′ B intersects
ω1 for the second time at X ′ . Prove that P X = P X ′

2. In acute triangle ABC, ∠A = 45◦ . Points O, H are the circumcenter and the orthocenter of
ABC, respectively. D is the foot of altitude from B. Point X is the midpoint of arc AH of the
circumcircle of triangle ADH that contains D. Prove that DX = DO.

3. Find all possible values of integer n > 3 such that there is a convex n-gon in which, each
diagonal is the perpendicular bisector of at least one other diagonal.

4. Quadrilateral ABCD is circumscribed around a circle. Diagonals AC, BD are not perpendicular
to each other. The angle bisectors of angles between these diagonals, intersect the segments
AB, BC, CD and DA at points K, L, M and N . Given that KLM N is cyclic, prove that so is
ABCD.

5. ABCD is a cyclic quadrilateral. A circle passing through A, B is tangent to segment CD at


point E. Another circle passing through C, D is tangent to AB at point F . Point G is the
intersection point of AE, DF , and point H is the intersection point of BE, CF . Prove that the
incenters of triangles AGF, BHF, CHE, DGE lie on a circle.

20
Solutions

1. Two circles ω1 , ω2 intersect each other at points A, B. Let P Q be a common tangent line of
these two circles with P ∈ ω1 and Q ∈ ω2 . An arbitrary point X lies on ω1 . Line AX intersects
ω2 for the second time at Y . Point Y ′ ̸= Y lies on ω2 such that QY = QY ′ . Line Y ′ B intersects
ω1 for the second time at X ′ . Prove that P X = P X ′
Proposed by Morteza Saghafian
------------------------------------------------------------------
Solution.

QY = QY ′ implies ∠QY Y ′ = ∠QY ′ Y . Considering circle ω2 , we have ∠QY Y ′ = ∠Y ′ QP . This


means Y Y ′ ∥ P Q.
We also have ∠Y ′ Y A = ∠Y ′ BA and ∠ABX ′ = ∠AXX ′ . This means XX ′ ∥ Y Y ′ ∥ P Q.
Therefore ∠P XX ′ = ∠X ′ P Q = ∠P X ′ X, so P X = P X ′ ■

21
2. In acute triangle ABC, ∠A = 45◦ . Points O, H are the circumcenter and the orthocenter of
ABC, respectively. D is the foot of altitude from B. Point X is the midpoint of arc AH of the
circumcircle of triangle ADH that contains D. Prove that DX = DO.
Proposed by Fatemeh Sajadi
------------------------------------------------------------------
Solution.

Since ∠AXH = 90◦ and XA = XH, we conclude that ∠AHX = 45◦ = ∠ADX.
Also ∠BOC = 2∠A = 90◦ , therefore points O, D lie on a circle with diameter BC. This implies

\ = OBC
ODA \ = 45◦ =⇒ ODX
\ = 90◦ .

But note that


\ = 90◦ − A
ACH b = 45◦ = 1 AXH.
\
2
This alongside with XA = XH means X is the circumcenter of triangle ACH and so XA = XC.
Thus OX is the perpendicular bisector of AC and so OX ⊥ AC. Now in triangle ODX, the
angle bisector of vertex D is the same as the altitude from D, hence it is an isosceles triangle
with DX = DO. ■

22
3. Find all possible values of integer n > 3 such that there is a convex n-gon in which, each
diagonal is the perpendicular bisector of at least one other diagonal.
Proposed by Mahdi Etesamifard
------------------------------------------------------------------
Solution. Let m be the total number of the perpendicular bisectors of all diagonals in the given
n-gon. The statement of the problem implies that m is not less than the number of diagonals.
But it is clear that the total number of perpendicular bisectors of the diagonals does not exceed
the number of diagonals! Hence, we conclude that each diagonal is the perpendicular bisector
of exactly one other diagonal. Since the perpendicular bisector of a diagonal is a unique line, we
get that for each diagonal d, there is exactly one diagonal d′ such that d′ is the perpendicular
bisector of d.
Consider three adjacent vertices B, A, C of the n-gon, where A lies between B and C. BC is a
diagonal of the n-gon, and only diagonals that contain A have an intersection point with BC.
Specially, the diagonal which is the perpendicular bisector of BC passes through A. Hence
AB = AC. Using this similar idea, it is deduced that all sides of this n-gon have the same
length.

Similar to the previous part, consider four adjacent points of the n-gon, A, B, C, D with the
given order. If n > 4, then AD is a diagonal of the n-gon, and the only diagonals that contain
B or C, have an intersection point with AD. Therefore either B or C lie on the perpendicular
bisector of AD. Without loss of generality, assume that BA = BD. According to the previous
argument, BA = BC = CD. Thus triangle BCD is an equilateral and so ∠BCD = 60◦ . (In
the other case we would have ∠ABC = 60◦ .)

23
This implies that between any two adjacent vertices, there is one that has a 60 degree angle.
Hence there is at least n2 angles of 60◦ in this n-gon.
It is known that the total number of 60 degree angles in an n-gon with n > 3 is at most 2. So
we must have n2 ≤ 2 which means n ≤ 4, a contradiction.
Clearly, any rhombus satisfies the desired property. So the answer is n = 4 . ■

24
4. Quadrilateral ABCD is circumscribed around a circle. Diagonals AC, BD are not perpendicular
to each other. The angle bisectors of angles between these diagonals, intersect the segments
AB, BC, CD and DA at points K, L, M and N . Given that KLM N is cyclic, prove that so is
ABCD.
Proposed by Nikolai Beluhov (Bulgaria)
------------------------------------------------------------------
Solution. Let P be the intersection point of AC, BD. First we claim that KL and M N are not
parallel. Assume the contrary, that KL ∥ M N . Since KLM N is cyclic, we have KN = M L,
and P K = P L, P M = P N . We also have
KP PL
= .
PM PN
Let AP = x, BP = y, CP = z and DP = t. Also let ∠AP B = 2α and ∠BP C = 2θ. We have
1
xy zt KP z
+ 1t
KP = cos α, P M = cosα =⇒ = 1 .
x+y z+t PM x
+ y1

Similarly,
1
PL x
+ 1t
= 1 .
PN y
+ z1
KP PL
Since PM
= PN
,with a little of calculation, we shall have
( )( )
1 1 1 1 1 1 1 1 1 1 1 1
+ + = + + =⇒ − + + + = 0.
yz z 2 zt tx x2 xy x y x y z t

Which means x = z. But note that P K = P L implies


xy yz
cos α = cos θ.
x+y y+z
But θ = 90◦ − α, so we must have α = θ = 45◦ , and so AC ⊥ BD, which is a contradiction.
Therefore the claim is proved. With the similar idea, we can show that KN and LM are not
parallel.

25
AQ AP
By Menelaus’ theorem, KL and M N meet at a point Q on AC such that QC
= PC
and LM
BR
and N K meet at a point R on BD such that RD = PBPD .
Let the incircle ω of ABCD touch its sides at K ′ , L′ , M ′ , and N ′ . By Brianchon’s theorem,
AL′ , CK ′ , and BD are concurrent. By Ceva’s and Menelaus’ theorems, K ′ , L′ , and Q are
collinear. Analogously, M ′ , N ′ , and Q are collinear and L′ M ′ and N ′ K ′ meet at R.
By Brianchon’s theorem, K ′ M ′ and L′ N ′ meet at P . It follows the diagonals and opposite
sides of both KLM N and K ′ L′ M ′ N ′ intersect at the vertices of △P QR. Therefore, both the
circumcircle of KLM N and ω coincide with the polar circle of △P QR.
Since K is a common point of AB and ω, K ≡ K ′ . Analogously, L ≡ L′ , M ≡ M ′ , and N ≡ N ′ .
Hence the angle bisector KM of AC and BD makes equal angles with AB and CD and ABCD
is cyclic, as needed. ■

26
5. ABCD is a cyclic quadrilateral. A circle passing through A, B is tangent to segment CD at
point E. Another circle passing through C, D is tangent to AB at point F . Point G is the
intersection point of AE, DF , and point H is the intersection point of BE, CF . Prove that the
incenters of triangles AGF, BHF, CHE, DGE lie on a circle.
Proposed by Le Viet An (Vietnam)
------------------------------------------------------------------
Solution.

Let I, J, K, L be the incenters of the triangles AGF, BHF, CHE, DGE respectively. Let ω be
the circumcircle of ABCD. In case of AB ∥ CD, we would conclude that ABCD is an isosceles
trapezoid and it is easy to see that IJKL is also an isosceles trapezoid.
So assume that AB ̸∥ CD and let M be the intersection point of rays BA and CD. Since
ABCD is cyclic, it is obtained that

M A · M B = M D · M C = PM (ω)

Since M E is tangent to ⊙ABE, we get


\
M \
EA = M BE.

We also have M E 2 = M A · M B = PM (⊙ABE) and M F 2 = M D · M C = PM (⊙CDF ), which


\
implies M E = M F , and so M \
EF = M F E. Therefore,
[ =M
AEF \ \
EF − M \
EA = M \
FE − M \.
BE = BEF

The latest equation means that EF is the interior angle bisector of ∠AEB. Similarly, F E is
the interior angle bisector of ∠CF D.

27
∠F BH
Note that H, J, K are collinear and ∠F JH = 90◦ + 2
. Thus

\
M BE \
M EA
F[
JK = 90◦ + = 90◦ +
2 2
◦ [
180 − AEC [
AEC
= 90◦ + = 180◦ −
2 2
[ + BEC
AEB \ ( )

= 180 − ◦ \ \
= 180 − F EB + BEK
2
\
= 180◦ − F EK

This results in that EF JK is cyclic. With similar arguments, EF IL is also cyclic.


Since EF is the interior angle bisector of ∠GEH and ∠GF H, it is easy to see that triangles
GEF and HEF are congruent. Therefore EG = EH and F G = F H, and so GE GF
= HE
HF
= k.
Consider three lines, the exterior angle bisector of vertex G in △GEF , the exterior angle bisec-
tor of vertex H in △HEF and the line EF . According to the latest equation, there is two cases:

• These three lines are pairwise parallel. This means EF JK and EF IL are isosceles trape-
zoids. Hence the segments EF , JK and IL have the same perpendicular bisector and so
IJKL is an isosceles trapezoid. □
PE
• These three points are concurrent at a point P where PF
= k. Now we simply have

P J · P K = PP (⊙EF JK) = P E · P F = PP (⊙EF IL) = P I · P L.

Which means IJKL is cyclic. □

28
6th Iranian Geometry
Olympiad

Contest problems with solutions


6th Iranian Geometry Olympiad
Contest problems with solutions.

This booklet is prepared by Alireza Dadgarnia and Benyamin Ghaseminia.


Copyright c Iranian Geometry Olympiad Secretariat 2018-2019. All rights reserved.
Contents

Elementary Level 3
Problems . . . . . . . . . . . . . . . . . . . . . . . . . . . . . . . . . . . . . . . . . . . . . . . . . . . . . 3

Solutions . . . . . . . . . . . . . . . . . . . . . . . . . . . . . . . . . . . . . . . . . . . . . . . . . . . . . 5

Intermediate Level 15
Problems . . . . . . . . . . . . . . . . . . . . . . . . . . . . . . . . . . . . . . . . . . . . . . . . . . . . . 15

Solutions . . . . . . . . . . . . . . . . . . . . . . . . . . . . . . . . . . . . . . . . . . . . . . . . . . . . . 17

Advanced Level 29
Problems . . . . . . . . . . . . . . . . . . . . . . . . . . . . . . . . . . . . . . . . . . . . . . . . . . . . . 29

Solutions . . . . . . . . . . . . . . . . . . . . . . . . . . . . . . . . . . . . . . . . . . . . . . . . . . . . . 31
Elementary Level

1
Problems

1) There is a table in the shape of a 8 × 5 rectangle with four holes on its


corners. After shooting a ball from points A, B and C on the shown paths,
will the ball fall into any of the holes after 6 reflections? (The ball reflects
with the same angle after contacting the table edges.)

4 B 4
◦ ◦
45 45

2
45◦
A 4

45◦

3 45◦
C
45◦ 1

(→ p.5)

2) As shown in the figure, there are two rectangles ABCD and P QRD with
the same area, and with parallel corresponding edges. Let points N, M
and T be the midpoints of segments QR, P C and AB, respectively. Prove
that points N, M and T lie on the same line.

3
4 Elementary Level

P Q

N
A D
M R

B C

(→ p.8)
3) There are n > 2 lines on the plane in general position; Meaning any two
of them meet, but no three are concurrent. All their intersection points
are marked, and then all the lines are removed, but the marked points
are remained. It is not known which marked point belongs to which two
lines. Is it possible to know which line belongs where, and restore them
all? (→ p.9)
4) Quadrilateral ABCD is given such that

∠DAC = ∠CAB = 60◦ ,

and
AB = BD − AC.
Lines AB and CD intersect each other at point E. Prove that

∠ADB = 2∠BEC.

(→ p.10)
5) For a convex polygon (i.e. all angles less than 180◦ ) call a diagonal bi-
sector if its bisects both area and perimeter of the polygon. What is the
maximum number of bisector diagonals for a convex pentagon?
(→ p.11)
Solutions

1) There is a table in the shape of a 8 × 5 rectangle with four holes on its


corners. After shooting a ball from points A, B and C on the shown paths,
will the ball fall into any of the holes after 6 reflections? (The ball reflects
with the same angle after contacting the table edges.)

4 B 4

45 45◦

2
45◦
A 4

45◦

3 45◦
C
45◦ 1

Proposed by Hirad Alipanah


-------------------------------------------------
Solution. It’s easy to track the trajectory of the ball.
Point A:

5
6 Elementary Level

2 2 2
1 4
45◦
A A

45
3
5 3
1 1 6 2 1
3 1 1
(a) The ball goes through a hole with one (b) The ball doesn’t go through a hole af-
reflection. ter six reflections.

Point B:
2 B 4 4 B 2
3 45◦ 45◦ 3
2 2
4 4

1 1
5 2 1 1 2 5

4 1 1 4
(a) The ball goes through a hole with five (b) The ball goes through a hole with five
reflections. reflections.

4 2 2
1 4 2
2 2
5 3

45◦
2 C C
3 6 1 4 1
1 45 1◦

1 4 3 1
(a) The ball goes through a hole with six (b) The ball goes through a hole with four
refelctions. reflections.

There are two point of views to the problem:


Solutions 7

(a) Looking for the trajectories where the ball goes through a hole with
at most 6 reflections:
In this case, all cases except A(b) are desired.
(b) Looking for the trajectories where the ball goes through a hole with
exactly 6 reflections:
In this case, C(a) is the only answer to the problem.

8 Elementary Level

2) As shown in the figure, there are two rectangles ABCD and P QRD with
the same area, and with parallel corresponding edges. Let points N, M
and T be the midpoints of segments QR, P C and AB, respectively. Prove
that points N, M and T lie on the same line.

P Q

N
A D
M R

B C

Proposed by Morteza Saghafian


-------------------------------------------------
Solution. Let L be the intersection point of P Q and CR, and let K be
the intersection point of BC and AP .

P Q L
A N
D
M R
T

K B C

Since QR k P C, it is deduced that L, N and M are collinear; Similarly,


K, T and M are collinear. Therefore it suffices to prove that P LCK is
a parallelogram to deduce that K, M and L are collinear, and get the
desired result of the problem. Since P L k CK, it suffices to show that
P K k CL, or P A k CR. Since the areas of the two rectangles are equal,
it is implied that
PD AD
P D · DR = AD · CD =⇒ = .
CD DR
Which implies AP k CR, and the proof is complete. 
Solutions 9

3) There are n > 2 lines on the plane in general position; Meaning any two
of them meet, but no three are concurrent. All their intersection points
are marked, and then all the lines are removed, but the marked points are
remained. It is not known which marked point belongs to which two lines.
Is it possible to know which line belongs where, and restore them all?
Proposed by Boris Frenkin - Russia
-------------------------------------------------
Answer. Yes, it is.

Solution. Draw the lines which each of them contains n−1 marked points,
at least. All the original lines are among these lines. Conversely, let some
line ` contains some n − 1 marked points. They are points of meet of some
pairs of the original lines (`1 , `2 ) , (`3 , `4 ) , . . . , (`2n−3 , `2n−2 ). Since n > 2,
we have 2n − 2 > n, so `i coincides with `j for some 1 ≤ i < j ≤ 2n − 2.
Then these lines belong to distinct pairs in the above list, and the two
corresponding marked points belong to `i = `j . But then also ` = `i , and
we are done. 
10 Elementary Level

4) Quadrilateral ABCD is given such that

∠DAC = ∠CAB = 60◦ ,

and
AB = BD − AC.
Lines AB and CD intersect each other at point E. Prove that

∠ADB = 2∠BEC.

Proposed by Iman Maghsoudi


-------------------------------------------------
Solution. Consider point F on ray BA such that AF = AC.

F
A

D E
C

Knowing that AB = BD − AC, it is implied that BF = BD. Therefore



AF = AC 
AD = AD =⇒ 4F AD ∼ = 4CAD. (1)
∠F AD = ∠CAD = 60◦

Note that
(1)
∠BEC = ∠F AD − ∠ADC = 60◦ − ∠ADF. (2)
On the other hand

∠ADB = ∠F DB − ∠ADF = ∠AF D − ∠ADF


= (120◦ − ∠ADF ) − ∠ADF
= 120◦ − 2∠ADF
(2)
= 2∠BEC.

So the claim of the problem is proved. 


Solutions 11

5) For a convex polygon (i.e. all angles less than 180◦ ) call a diagonal bi-
sector if its bisects both area and perimeter of the polygon. What is the
maximum number of bisector diagonals for a convex pentagon?
Proposed by Morteza Saghafian
-------------------------------------------------
Answer. The maximum number of bisector diagonals is 2.

Solution. Note that for each vertex, there is at most one bisector diagonal
that passes through it; Therefore there are at most 2 bisector diagonals in
the pentagon. The following figure shows an example where the pentagon
has two bisector diagonals.

B D

A E


12 Elementary Level
Intermediate Level

13
Problems

1) Two circles ω1 and ω2 with centers O1 and O2 respectively intersect each


other at points A and B, and point O1 lies on ω2 . Let P be an arbitrary
point lying on ω1 . Lines BP, AP and O1 O2 cut ω2 for the second time
at points X, Y and C, respectively. Prove that quadrilateral XP Y C is a
parallelogram.
(→ p.17)
2) Find all quadrilaterals ABCD such that all four triangles DAB, CDA,
BCD and ABC are similar to one-another.
(→ p.19)
3) Three circles ω1 , ω2 and ω3 pass through one common point, say P . The
tangent line to ω1 at P intersects ω2 and ω3 for the second time at points
P1,2 and P1,3 , respectively. Points P2,1 , P2,3 , P3,1 and P3,2 are similarly de-
fined. Prove that the perpendicular bisector of segments P1,2 P1,3 , P2,1 P2,3
and P3,1 P3,2 are concurrent.
(→ p.20)
4) Let ABCD be a parallelogram and let K be a point on line AD such
that BK = AB. Suppose that P is an arbitrary point on AB, and the
perpendicular bisector of P C intersects the circumcircle of triangle AP D
at points X, Y . Prove that the circumcircle of triangle ABK passes
through the orthocenter of triangle AXY .
(→ p.23)

5) Let ABC be a triangle with ∠A = 60 . Points E and F are the foot of
angle bisectors of vertices B and C respectively. Points P and Q are con-
sidered such that quadrilaterals BF P E and CEQF are parallelograms.
Prove that ∠P AQ > 150◦ . (Consider the angle P AQ that does not con-
tain side AB of the triangle.)
(→ p.25)

15
16 Intermediate Level
Solutions

1) Two circles ω1 and ω2 with centers O1 and O2 respectively intersect each


other at points A and B, and point O1 lies on ω2 . Let P be an arbitrary
point lying on ω1 . Lines BP, AP and O1 O2 cut ω2 for the second time
at points X, Y and C, respectively. Prove that quadrilateral XP Y C is a
parallelogram.
Proposed by Iman Maghsoudi
-------------------------------------------------
Solution.

ω2

A
ω1
X

O1 O2 C

One can obtain


> >
AO1 B XCY
∠AP B = + . (1)
2 2
Note that ∠AO2 O1 = ∠BO2 O1 , therefore ∠AO1 O2 = ∠BO1 O2 . Now

17
18 Intermediate Level

since O1 is the circumcenter of triangle AP B, it is deduced that


> >
∠AO1 B AXC ABC
∠AP B = 180◦ − = 180◦ − = . (2)
2 2 2
Hence, according to equations 1 and 2 it is obtained that
> > >
AO1 B XCY ABC > >
+ = =⇒ XCY = BY C,
2 2 2
thus BX k CY , similarly AY k XC. This imlplies that XP Y C is a
parallelogram and the claim of the problem is proved. 
Solutions 19

2) Find all quadrilaterals ABCD such that all four triangles DAB, CDA,
BCD and ABC are similar to one-another.
Proposed by Morteza Saghafian
-------------------------------------------------
Answer. All rectangles.

Solution. First assume that ABCD is a concave quadrilateral. Without


loss of generality one can assume ∠D > 180◦ , in other words D lies inside
of triangle ABC. Again without loss of generality one can assume that
∠ABC is the maximum angle in triangle ABC. Therefore

∠ADC = ∠ABC + ∠BAD + ∠BCD > ∠ABC.

Thus ∠ADC is greater than all the angles of triangle ABC, so triangles
ABC and ADC cannot be similar. So it is concluded that ABCD must
be convex.
Now let ABCD be a convex quadrilateral. Without loss of generality one
can assume that the ∠B is the maximum angle in the quadrilateral. It
can be written that

∠ABC > ∠DBC, ∠ABC ≥ ∠ADC ≥ ∠BCD.

Since triangles ABC and BCD are similar, it is implied that ∠ABC =
∠BCD and similarly, all the angles of ABCD are equal; Meaning ABCD
must be a rectangle. It is easy to see that indeed, all rectangles satisfy
the conditions of the problem. 
20 Intermediate Level

3) Three circles ω1 , ω2 and ω3 pass through one common point, say P . The
tangent line to ω1 at P intersects ω2 and ω3 for the second time at points
P1,2 and P1,3 , respectively. Points P2,1 , P2,3 , P3,1 and P3,2 are similarly de-
fined. Prove that the perpendicular bisector of segments P1,2 P1,3 , P2,1 P2,3
and P3,1 P3,2 are concurrent.
Proposed by Mahdi Etesamifard
-------------------------------------------------
Solution.
P1,2

P2,1

P3,1 P
P3,2

P1,3

P2,3

First assume that no two of the lines `1 ≡ P2,1 P3,1 , `2 ≡ P1,2 P3,2 and
`3 ≡ P1,3 P2,3 are parallel; Consider triangle XY Z made by intersecting
these lines, where

X ≡ `2 ∩ `3 ,
Y ≡ `1 ∩ `3 ,
Z ≡ `2 ∩ `1 .

Note that
∠P3,2 P1,2 P = ∠P3,2 P P2,3 = ∠P P1,3 P2,3 ,
meaning XP1,2 = XP1,3 . Similarly, it is implied that Y P2,1 = Y P2,3 and
ZP3,1 = ZP3,2 . Therefore, the angle bisectors of angles Y XZ, XY Z and
Solutions 21

Y ZX are the same as the perpendicular bisectors of segments P1,2 P1,3 ,


P2,1 P2,3 and P3,1 P3,2 ; Thus, these three perpendicular bisectors are con-
current at the incenter of triangle XY Z, resulting in the claim of the
problem.
Now assume that at least two of the lines `1 = P2,1 P3,1 , `2 = P1,2 P3,2 and
`3 = P1,3 P2,3 are parallel; Without loss of generality assume that `1 and
`2 are parallel. Similar to the previous case,
∠P1,2 P3,2 P = ∠P1,2 P P2,1 = ∠P2,1 P3,1 P.
But since `1 k `2 , it is also true that
∠P1,2 P3,2 P + ∠P2,1 P3,1 P = 180◦ ,
Hence ∠P1,2 P3,2 P = ∠P2,1 P3,1 P = 90◦ . This equation immediately im-
plies `3 ∦ `2 , because otherwise it would be deduced that `3 ⊥ P1,3 P1,2
and `2 ⊥ P1,3 P1,2 , resulting in P1,3 P1,2 k P3,1 P3,2 ; Which is clearly not
possible. Now consider trapezoid XY P2,1 P1,2 . The problem is now equiv-
alent to show that the angle bisector of ∠X, angle bisector of ∠Y and the
perpendicular bisector of P3,1 P3,2 concur. Note that `1 and `2 are parallel
to the perpendicular bisector of P3,1 P3,2 , and in fact, the perpendicular
bisector of P3,1 P3,2 connects the midpoints of XY and P3,1 P3,2 . Now the
claim of the problem is as simple as follows.

P1,2
P2,1

0 K 0
P3,1 P3,2
P3,1 P3,2

K0

Claim. In trapezoid XY P2,1 P1,2 , the angle bisector of ∠X, the angle
bisector of ∠Y , and the mid-line of the trapezoid are concurrent.

Proof. Let K be the intersection of the angle bisector of ∠X and the angle
0 0
bisector of ∠Y . Let P3,2 , P3,1 and K 0 be the foot of perpendicular lines
22 Intermediate Level

from K to lines P1,2 X, P2,1 Y and XY , respectively. Since K lies on the


0
angle bisector of ∠X, it is deduced that KP3,2 = KK 0 , and similarly since
K lies on the angle bisector of ∠Y , KP3,1 = KK 0 ; Thus KP3,1
0 0
= KP3,20
,
meaning K lies on the mid-line of trapezoid XY P2,1 P1,2 .

This result leads to the conclusion of the problem. 


Solutions 23

4) Let ABCD be a parallelogram and let K be a point on line AD such


that BK = AB. Suppose that P is an arbitrary point on AB, and the
perpendicular bisector of P C intersects the circumcircle of triangle AP D
at points X, Y . Prove that the circumcircle of triangle ABK passes
through the orthocenter of triangle AXY .
Proposed by Iman Maghsoudi
-------------------------------------------------
Solution. Let AN be the altitude of triangle AXY . Suppose that the
circumcircle of triangle ABK intersects AN at the point R. It is enough
to show that R is the orthocenter of triangle AXY .

A P B

R X

N T

D C
Q
Y
S

Suppose that P C and AN intersects the circumcircle of triangle AP D for


the second time at point T and S, respectively and AN intersects CD at
Q. We know that

∠BRS = ∠AKB = ∠KAB = ∠P T D =⇒ ∠BRS = ∠P T D (1)

Notice that XY is perpendicular to AN and P C, so AN k P C. Also


AP k CQ so AP CQ is parallelogram. Now we have

∠BAR = ∠T CD 
(1) =⇒ ∠ARB = ∠CT D =⇒ 4ARB ∼ = 4CT D =⇒ CT = AR.
AB = CD

24 Intermediate Level

So P T QR is also a parallelogram. AP T S is an isosceles trapezoid so

CQ = AP = T S.

Therefore T QSC is also an isosceles trapezoid. Finally

CS = T Q = P R =⇒ CSRP is an isosceles trapezoid.

Therefore, S and R are symmetric with respect to XY . So R should be


the orthocenter of triangle AXY . 
Solutions 25

5) Let ABC be a triangle with ∠A = 60◦ . Points E and F are the foot of
angle bisectors of vertices B and C respectively. Points P and Q are con-
sidered such that quadrilaterals BF P E and CEQF are parallelograms.
Prove that ∠P AQ > 150◦ . (Consider the angle P AQ that does not con-
tain side AB of the triangle.)
Proposed by Alireza Dadgarnia
-------------------------------------------------
Solution. Let I and be the intersection point of lines BE and CF , and
let R be the intersection point of lines QE and P F . It is easy to see that
∠BIC = 120◦ . Thus AEIF is a cyclic quadrilateral and so

CE · CA = CI · CF (1)

Also ∠P RQ = ∠BIC = 120◦ , therefore it suffices to show that at least


on of the angles ∠AP R or ∠AQR is greater than or equal to 30◦ .

Q K
A P

R
E
F

B
C

Assume the contrary, meaning both of these angles are less than 30◦ .
Hence there exists a point K on the extension of ray CA such that
∠KQE = 30◦ . Since ∠IAC = 30◦ ∠ACI = ∠KEQ, it is deduced that
4AIC ∼ 4QKE. This implies

CI KE AE CF · CI (1)
= > =⇒ AE < = CE.
CA QE CF CA
26 Intermediate Level

Similarly, it is obtained that AF < BF . On the other hand at least


on of the angles ∠ABC or ∠ACB are not less than 60◦ . Without loss
of generality one can assume that ∠ABC ≥ 60◦ thus AC ≥ BC and
according to angle bisector theorem it is obtained that AF ≥ BF , which
is a contradiction. Hence the claim of the problem. 
Advanced Level

27
Problems

1) Circles ω1 and ω2 intersect each other at points A and B. Point C lies on


the tangent line from A to ω1 such that ∠ABC = 90◦ . Arbitrary line `
passes through C and cuts ω2 at points P and Q. Lines AP and AQ cut
ω1 for the second time at points X and Z respectively. Let Y be the foot
of altitude from A to `. Prove that points X, Y and Z are collinear.
(→ p.31)

2) Is it true that in any convex n-gon with n > 3, there exists a vertex and a
diagonal passing through this vertex such that the angles of this diagonal
with both sides adjacent to this vertex are acute?
(→ p.33)

3) Circles ω1 and ω2 have centres O1 and O2 , respectively. These two circles


intersect at points X and Y . AB is common tangent line of these two
circles such that A lies on ω1 and B lies on ω2 . Let tangents to ω1 and ω2
at X intersect O1 O2 at points K and L, respectively. Suppose that line
BL intersects ω2 for the second time at M and line AK intersects ω1 for
the second time at N . Prove that lines AM, BN and O1 O2 concur.
(→ p.34)

4) Given an acute non-isosceles triangle ABC with circumcircle Γ. M is the


>
midpoint of segment BC and N is the midpoint of BC of Γ (the one that
doesn’t contain A). X and Y are points on Γ such that BX k CY k AM .
Assume there exists point Z on segment BC such that circumcircle of
triangle XY Z is tangent to BC. Let ω be the circumcircle of triangle
ZM N . Line AM meets ω for the second time at P . Let K be a point
on ω such that KN k AM , ωb be a circle that passes through B, X and
tangents to BC and ωc be a circle that passes through C, Y and tangents
to BC. Prove that circle with center K and radius KP is tangent to 3
circles ωb , ωc and Γ.

29
30 Advanced Level

(→ p.35)
5) Let points A, B and C lie on the parabola ∆ such that the point H,
orthocenter of triangle ABC, coincides with the focus of parabola ∆.
Prove that by changing the position of points A, B and C on ∆ so that the
orthocenter remain at H, inradius of triangle ABC remains unchanged.
(→ p.38)
Solutions

1) Circles ω1 and ω2 intersect each other at points A and B. Point C lies on


the tangent line from A to ω1 such that ∠ABC = 90◦ . Arbitrary line `
passes through C and cuts ω2 at points P and Q. Lines AP and AQ cut
ω1 for the second time at points X and Z respectively. Let Y be the foot
of altitude from A to `. Prove that points X, Y and Z are collinear.
Proposed by Iman Maghsoudi
-------------------------------------------------
Solution.

ω1
A
ω2
Z

P
Y

Q C
B

Since ∠AY C = ∠ABC = 90◦ , it is concluded that AY BC is a cyclic


quadrilateral. Hence

∠BY C = ∠BAC = ∠BXA = ∠BXP.

31
32 Advanced Level

So P Y BX, and similarly QBY Z are cyclic quadrilaterals. Therefore it is


implied that

∠BY X = ∠BP X = ∠AQB = ∠ZQB = 180◦ − ∠ZY B.

Meaning points X, Y and Z are collinear. 


Solutions 33

2) Is it true that in any convex n-gon with n > 3, there exists a vertex and a
diagonal passing through this vertex such that the angles of this diagonal
with both sides adjacent to this vertex are acute?
Proposed by Boris Frenkin - Russia
-------------------------------------------------
Answer. Yes, it is true.

Solution. Suppose the answer is no. Given a convex n-gon (n > 3), con-
sider its longest diagonal AD (if the longest diagonal is not unique, choose
an arbitrary on among them). Let B and C be the vertices neighboring
to A. Without loss of generality assume that ∠BAD ≥ 90◦ . This means
BD > AD, so BD is not a diagonal and hence is a side of the n-gon.
Furthermore, ∠ADB < 90◦ . Let C 0 be the vertex neighboring to D and
distinct from B. Then ∠ADC 0 ≥ 90◦ . Similarly, AC 0 > AD, so AC 0 is
a side, C 0 ≡ C and n = 4. Angles BAC and BDC are obtuse, so BC
is longer than AC and BD, hence BC > AD and AD is not the longest
diagonal, a contradiction. Hence the claim. 
34 Advanced Level

3) Circles ω1 and ω2 have centres O1 and O2 , respectively. These two circles


intersect at points X and Y . AB is common tangent line of these two
circles such that A lies on ω1 and B lies on ω2 . Let tangents to ω1 and ω2
at X intersect O1 O2 at points K and L, respectively. Suppose that line
BL intersects ω2 for the second time at M and line AK intersects ω1 for
the second time at N . Prove that lines AM, BN and O1 O2 concur.
Proposed by Dominik Burek - Poland
-------------------------------------------------
Solution.
Let P be the midpoint of AB; Since P has the same power with respect
to both circles, it lies on the radical axis of them, which is line XY .

A0

X B0

L
O1
O2 K

M N

A P B

According to the symmetry, KY is tangent to ω1 , therefore XY is the


polar of K with respect to ω1 . Since P lies on XY , the polar of P passes
through K, and similarly, it also passes through A; Meaning AK is the
polar of P with respect to ω1 and P N is tangent to ω1 . Similarly, P M is
tangent to ω2 ; Thus points A, B, M and N lie on a circle with center P
and ∠AM B = ∠AN P = 90◦ . Let A0 be the antipode of A in circle ω1 ,
and let B 0 be the antipode of B. Line BN passes through A0 and line
AM passes through B 0 . Note that AA0 B 0 B is a trapezoid and O1 and O2
are the midpoints of its bases; Hence A0 B, B 0 A and O1 O2 are concurrent,
resulting in the claim of the problem. 
Solutions 35

4) Given an acute non-isosceles triangle ABC with circumcircle Γ. M is the


>
midpoint of segment BC and N is the midpoint of BC of Γ (the one that
doesn’t contain A). X and Y are points on Γ such that BX k CY k AM .
Assume there exists point Z on segment BC such that circumcircle of
triangle XY Z is tangent to BC. Let ω be the circumcircle of triangle
ZM N . Line AM meets ω for the second time at P . Let K be a point
on ω such that KN k AM , ωb be a circle that passes through B, X and
tangents to BC and ωc be a circle that passes through C, Y and tangents
to BC. Prove that circle with center K and radius KP is tangent to 3
circles ωb , ωc and Γ.

Proposed by Tran Quan - Vietnam


-------------------------------------------------
Solution. Let I, J and O be the centers of circles ωb , ωc and Γ, re-
spectively. It’s easy to see that I, J and O are collinear. Let S be the
intersection of IJ and BC. Since X and Y are symmetrical to B and C
with respect to IJ, it is implied that S, X and Y are collinear. Let T
>
be a point on BC of Γ (the one that doesn’t contain A) such that ST is
tangent to Γ. T Z meets Γ again at Q.

Q
A Y

R
O
X
I

S K
D Z M
B C

P
T
ω N L
36 Advanced Level

Since
ST 2 = SX · SY = SZ 2 ,
ZT is the interior angle bisector of ∠BT C, it is concluded that Q is the
>
midpoint of BAC of Γ. This leads to ZT ⊥ N T , resulted in T lies on ω.
Let R be the intersection of AM and T Q, and Ω be the circumcircle of
triangle RT P . Since

∠RP T = ∠M P T = ∠SZT = ∠ST R,

ST is tangent to Ω. Therefore Ω is tangent to Γ at T . We will prove that


K is the center of Ω. Now

SOM T is cyclic =⇒ ∠OSM = ∠OT M

OS ⊥ AM and M S ⊥ OM =⇒ ∠OM A = ∠OSM = ∠OT M

∠OT Q = ∠OQT

Which implies

∠M T R = ∠OT Q + ∠OT M = ∠OQT + ∠OM A = ∠M RT,

and thus M R = M T . Let L be the intersection of AM and T N . Since


4RT L is a right triangle at T , it is concluded that M is the midpoint of
RL.

KN k M L =⇒ ∠M T N = ∠M LT = ∠KN T
=⇒ KM k T N
=⇒ KM ⊥ RT

Thus, M K is the perpendicular bisector of RT . Note that

∠ZP R = ∠ZT M = ∠ZRP =⇒ ZR = ZP.

Since ZN is the diameter of ω it is implied that

ZK ⊥ KN =⇒ ZK ⊥ RP.

Therefore ZK is the perpendicular bisector of RP . Hence K is the center


of Ω.
We will prove Ω is tangent to ωb and ωc . Let D be the intersection of
T N and BC, and denote (S, SZ) to be circle with center S and radius
SZ. Since T Z is the interior angle bisector of ∠BT C, we have T D is the
Solutions 37

exterior angle bisector of the same angle. This leads to (DZ, BC) = −1.
Since M is the midpoint of BC, we have

M B 2 = M Z · M D,

which implies M lies on the radical axis of ωb and (S, SZ). Combining
with M A ⊥ SI, we have M A is the radical axis of ωb and (S, SZ). Thus,
the powers of point R with respect to ωb and (S, SZ) are equal. Invert
about the circle centered at R with radius

r = RZ · RT ,

which inverts Ω 7→ ZM ≡ BC and ωb 7→ ωb . Since circle ωb is tangent to


BC, we have Ω is tangent to ωb .
Analogously, we have Ω is tangent to ωc . This completes the proof. 
38 Advanced Level

5) Let points A, B and C lie on the parabola ∆ such that the point H,
orthocenter of triangle ABC, coincides with the focus of parabola ∆.
Prove that by changing the position of points A, B and C on ∆ so that the
orthocenter remain at H, inradius of triangle ABC remains unchanged.
Proposed by Mahdi Etesamifard
-------------------------------------------------
Solution. Since H coincides with the focus of parabola ∆, the circles
wA = (A, AH), wB = (B, BH) and wC = (C, CH) are tangent to line `,
the directrix of ∆.


ωC
Directrix
A Axis of Symmetry
ωA H
`

ωB

Now consider triangle ABC.


A
B0
C0
H

B A0 C
Solutions 39

It is well-known that

HA · HA0 = HB · HB 0 = HC · HC 0 = t.

Also

HA = 2R cos A
=⇒ t = 4R2 cos A cos B cos C. (1)
HA0 = 2R cos B cos C

Inversion with center H and inversion radius −2t, inverts the three circles
wA , wB and wC to lines BC, AC and AB respectively. In this inversion,
line ` inverts to incircle of triangle ABC. Therefore IH ⊥ `, thus point I
lies on axis of symmetry of ∆. Also point H lies on the incircle of triangle
ABC. Hence HI = r.

Directrix
A H I F Axis of Symmetry
K

As a result, if orthocenter of ABC lies on its incircle; Also

HI 2 = 2r2 − 4R2 cos A cos B cos C


=⇒ r2 = 2r2 − 4R2 cos A cos B cos C
=⇒ r2 = 4R2 cos A cos B cos C
40 Advanced Level

According to (1), it is implied that r2 = t = HA · HA0 . In inversion,


points K and F are invert points, thus
−−→ −−→
HK · HF = −2t = −2r2 =⇒ HK = r.

Which gives the result that inradius of triangle ABC is constant. 


7th Iranian Geometry
Olympiad

Contest problems with solutions


7th Iranian Geometry Olympiad
Contest problems with solutions.

This booklet is prepared by Sina Qasemi Nezhad, Alireza Dadgarnia, Hesam


Rajabzadeh, Siavash Rahimi, Mahdi Etesamifard and Morteza Saghafian.
With special thanks to Matin Yousefi and Alireza Danaei.

Copyright ©Iranian Geometry Olympiad Secretariat 2019-2020. All rights


reserved.
Contents

Elementary Level 3
Problems . . . . . . . . . . . . . . . . . . . . . . . . . . . . . . . . . . . . . . . . . . . . . . . . . . . . . 3

Solutions . . . . . . . . . . . . . . . . . . . . . . . . . . . . . . . . . . . . . . . . . . . . . . . . . . . . . 5

Intermediate Level 15
Problems . . . . . . . . . . . . . . . . . . . . . . . . . . . . . . . . . . . . . . . . . . . . . . . . . . . . . 15

Solutions . . . . . . . . . . . . . . . . . . . . . . . . . . . . . . . . . . . . . . . . . . . . . . . . . . . . . 17

Advanced Level 29
Problems . . . . . . . . . . . . . . . . . . . . . . . . . . . . . . . . . . . . . . . . . . . . . . . . . . . . . 29

Solutions . . . . . . . . . . . . . . . . . . . . . . . . . . . . . . . . . . . . . . . . . . . . . . . . . . . . . 31
Elementary Level

1
Problems

Problem 1. By a fold of a polygon-shaped paper, we mean drawing a


segment on the paper and folding the paper along that. Suppose that a paper
with the following figure is given. We cut the paper along the boundary of
the shaded region to get a polygon-shaped paper.
Start with this shaded polygon and make a rectangle-shaped paper from it
with at most 5 number of folds. Describe your solution by introducing the
folding lines and drawing the shape after each fold on your solution sheet.
(Note that the folding lines do not have to coincide with the grid lines of the
shape.)

(→ p.5)
Problem 2. A parallelogram ABCD is given (AB 6= BC). Points E and
G are chosen on the line CD such that AC is the angle bisector of both
angles ∠EAD and ∠BAG. The line BC intersects AE and AG at F and H,
respectively. Prove that the line F G passes through the midpoint of HE.
(→ p.8)
Problem 3. According to the figure, three equilateral triangles with side

3
4 Elementary Level

lengths a, b, c have one common vertex and do not have any other common
point. The lengths x, y and z are defined as in the figure. Prove that
3(x + y + z) > 2(a + b + c).

c b

y z

(→ p.9)
Problem 4. Let P be an arbitrary point in the interior of triangle ABC.
Lines BP and CP intersect AC and AB at E and F , respectively. Let K
and L be the midpoints of the segments BF and CE, respectively. Let the
lines through L and K parallel to CF and BE intersect BC at S and T ,
respectively; moreover, denote by M and N the reflection of S and T over
the points L and K, respectively. Prove that as P moves in the interior of
triangle ABC, line M N passes through a fixed point.
(→ p.10)
Problem 5. We say two vertices of a simple polygon are visible from each
other if either they are adjacent, or the segment joining them is completely
inside the polygon (except two endpoints that lie on the boundary). Find all
positive integers n such that there exists a simple polygon with n vertices in
which every vertex is visible from exactly 4 other vertices.
(A simple polygon is a polygon without hole that does not intersect itself.)
(→ p.11)
Solutions

Problem 1. By a fold of a polygon-shaped paper, we mean drawing a


segment on the paper and folding the paper along that. Suppose that a paper
with the following figure is given. We cut the paper along the boundary of
the shaded region to get a polygon-shaped paper.
Start with this shaded polygon and make a rectangle-shaped paper from it
with at most 5 number of folds. Describe your solution by introducing the
folding lines and drawing the shape after each fold on your solution sheet.
(Note that the folding lines do not have to coincide with the grid lines of the
shape.)

Proposed by Mahdi Etesamifard


---------------------------------------------------
Solution. There are different ways of folding to get a rectangle. For instance,
a solution can be given with only 4 number of folds, as following

First fold:

5
6 Elementary Level

Second fold:

Third fold:

Fourth fold:
Solutions 7

Comment. that by moving the folding lines slightly in 3rd and 4th folds (to
down and up respectively), all the folding segments will be internal.

8 Elementary Level

Problem 2. A parallelogram ABCD is given (AB 6= BC). Points E and


G are chosen on the line CD such that AC is the angle bisector of both
angles ∠EAD and ∠BAG. The line BC intersects AE and AG at F and H,
respectively. Prove that the line F G passes through the midpoint of HE.
Proposed by Mahdi Etesamifard
---------------------------------------------------
Solution. Since AD and BC are parallel, we deduce that ∠F CA = ∠DAC =
∠F AC. So F A = F C. Similarly, GA = GC. So triangles 4GAF and
4GCF have a common side and two equal sides and are congruent. Result-
ing ∠GAF = ∠GCF which leads to ∠HAF = ∠ECF and ∠AF H = ∠CF E.
Therefore triangles 4AF H and 4CF E are congruent as well and we get
F E = F H. Similarly, GE = GH. So both points F and G lie on perpendicu-
lar bisector of segment HE. Resulting that F G is the perpendicular bisector
of segment HE.


Solutions 9

Problem 3. According to the figure, three equilateral triangles with side


lengths a, b, c have one common vertex and do not have any other common
point. The lengths x, y and z are defined as in the figure. Prove that
3(x + y + z) > 2(a + b + c).

c b

y z

Proposed by Mahdi Etesamifard


---------------------------------------------------
Solution. Consider the three white triangles in the picture, rotating each
triangle 60◦ degrees, clock-wise, will make a side of it coincide with another
side of another triangle. So we can rotate one of them and glue it to the next,
then rotating the glued figure a broken path will be formed between two
points with distances like 2a which has length x + y + z. Thus x + y + z > 2a
summing up all three possible inequalities proves the desired. 
10 Elementary Level

Problem 4. Let P be an arbitrary point in the interior of triangle ABC.


Lines BP and CP intersect AC and AB at E and F , respectively. Let K
and L be the midpoints of the segments BF and CE, respectively. Let the
lines through L and K parallel to CF and BE intersect BC at S and T ,
respectively; moreover, denote by M and N the reflection of S and T over
the points L and K, respectively. Prove that as P moves in the interior of
triangle ABC, line M N passes through a fixed point.
Proposed by Ali Zamani
---------------------------------------------------
Solution. Since in quadrilateral EM CS, diagonals bisect each other, this
quadrilateral is a parallelogram. So, EM k BC. Let X be the intersection of
EM and CF . Note that M L k CX and L is the midpoint of CE, resulting
that M is the midpoint of EX as well. Since EX k BC, using parallel lines,
one can find that M P passes through the midpoint of BC. Similarly, N P
passes through the midpoint of BC. Hence proved.


Solutions 11

Problem 5. We say two vertices of a simple polygon are visible from each
other if either they are adjacent, or the segment joining them is completely
inside the polygon (except two endpoints that lie on the boundary). Find all
positive integers n such that there exists a simple polygon with n vertices in
which every vertex is visible from exactly 4 other vertices.
(A simple polygon is a polygon without hole that does not intersect itself.)
Proposed by Morteza Saghafian
---------------------------------------------------
Solution.
First we prove there is no such polygon for n > 6. Let A1 , A2 , . . . , An be the
vertices.
Lemma 1. Let Ai be visible from Ai−1 , Aj , Ak , Ai+1 in clockwise order (note
that the first and the last one are the edge-neighbors). Then Ai−1 , Aj can see
each other, Aj , Ak can see each other and Ak , Ai+1 can see each other.
Proof. One can consider the triangulation of the three parts of polygon sep-
arated by Ai Aj and Ai Ak .

Lemma 2. Using the same naming as Lemma 1, Aj Ak is a side.

Proof. Assume that Aj Ak is an internal diagonal. By Lemma 1, Aj can see


Aj−1 . But Aj Ai and Aj Ak are internal diagonals. So Aj Ai−1 is a side. So
there is only one vertex between Ai , Aj on the perimeter of polygon. Similarly,
there is only one vertex between Aj , Ak and only one vertex between Ak , Ai
on the perimeter of polygon. This contradicts n > 6. So Aj Ak is a side and
k = j − 1.
Now let i be such that Ai−1 , Ai+1 are visible from each other. We know that
such i exists, for instance you cam take an ear triangle in the triangulation of
the polygon. By Lemma 2, Ai−1 can see Ai+2 , Ai+1 can see Ai−2 and Ai−2
can see Ai+2 . So we found the four vertices visible from Ai−1 , Ai+1 . If Ai
can see a vertex, then it is visible by either Ai−1 or Ai+1 (by Lemma 1). So
Ai should see Ai−2 , Ai+2 and this means Ai−2 Ai+2 is a side (by Lemma 2).
Any convex pentagon is an example.
The only remaining case is n = 6 which means in Lemma 2 there are vertices
Ai , Aj , Ak such that Ai Aj , Aj Ak , Ak Ai are internal diagonals. Let them be
A2 , A4 , A6 in the hexagon. So A3 is not visible from A6 , meaning that one
of the angles A2 , A4 is larger that 180◦ . But then A3 cannot see either A1 or
A5 . Which contradicts the fact that A3 is visible from 4 other vertices. So
n = 6 is also not possible and the only possible n is 5.

12 Elementary Level
Intermediate Level

13
Problems

Problem 1. A trapezoid ABCD is given where AB and CD are parallel.


Let M be the midpoint of the segment AB. Point N is located on the segment
1 1
CD such that ∠ADN = ∠M N C and ∠BCN = ∠M N D. Prove that N
2 2
is the midpoint of the segment CD.
(→ p.17)

Problem 2. Let ABC be an isosceles triangle (AB = AC) with its circum-
center O. Point N is the midpoint of the segment BC and point M is the
reflection of the point N with respect to the side AC. Suppose that T is a
1
point so that AN BT is a rectangle. Prove that ∠OM T = ∠BAC.
2
(→ p.18)

Problem 3. In acute-angled triangle ABC (AC > AB), point H is the


orthocenter and point M is the midpoint of the segment BC . The median
AM intersects the circumcircle of triangle ABC at X. The line CH intersects
the perpendicular bisector of BC at E and the circumcircle of the triangle
ABC again at F . Point J lies on circle ω, passing through X, E, and F , such
that BCHJ is a trapezoid (CB k HJ). Prove that JB and EM meet on ω.
(→ p.19)

Problem 4. Triangle ABC is given. An arbitrary circle with center J,


passing through B and C, intersects the sides AC and AB at E and F ,
respectively. Let X be a point such that triangle F XB is similar to triangle
EJC (with the same order) and the points X and C lie on the same side of
the line AB. Similarly, let Y be a point such that triangle EY C is similar to
triangle F JB (with the same order) and the points Y and B lie on the same
side of the line AC. Prove that the line XY passes through the orthocenter
of the triangle ABC.

15
16 Intermediate Level

(→ p.21)
Problem 5. Find all numbers n ≥ 4 such that there exists a convex polyhe-
dron with exactly n faces, whose all faces are right-angled triangles.
(Note that the angle between any pair of adjacent faces in a convex polyhe-
dron is less than 180◦ .)
(→ p.23)
Solutions

Problem 1. A trapezoid ABCD is given where AB and CD are parallel.


Let M be the midpoint of the segment AB. Point N is located on the segment
1 1
CD such that ∠ADN = ∠M N C and ∠BCN = ∠M N D. Prove that N
2 2
is the midpoint of the segment CD.
Proposed by Alireza Dadgarnia
---------------------------------------------------
Solution. We have
1
∠BCN + ∠ADN = (∠M N D + ∠BDN ) = 90◦ .
2
Hence, AD and BC intersect in point P such that ∠DP C = 90◦ . Since M is
the midpoint of AB,
∠P M A = 2∠P BA = 2∠P CD = ∠M N D.
Note that AB and CD are parallel, therefore, P M and M N are parallel to
and M, N and P lie on a straight line, hence N is the midpoint of segment
CD.
P

A M B

D N C

17
18 Intermediate Level

Problem 2. Let ABC be an isosceles triangle (AB = AC) with its circum-
center O. Point N is the midpoint of the segment BC and point M is the
reflection of the point N with respect to the side AC. Suppose that T is a
1
point so that AN BT is a rectangle. Prove that ∠OM T = ∠BAC.
2
Proposed by Ali Zamani
---------------------------------------------------
Since 4ABC is an isosceles triangle, we have ∠AN C = 90◦ . Therefore,

∠OCM = ∠OCA + ∠M CA = ∠OAC + ∠N CA = 90◦ = ∠T AO.

Also we have CM = CN = BN = AT and OC = OA; So triangles 4OCM


and 4OAT are congruent. Which leads to OT = OM and

∠AOT = ∠M OC =⇒ ∠T OM = ∠AOC.

Thus, 4AOC ∼ 4M OT and ∠OM T = ∠OAC = 12 ∠A.

T A

B N C


Solutions 19

Problem 3. In acute-angled triangle ABC (AC > AB), point H is the


orthocenter and point M is the midpoint of the segment BC . The median
AM intersects the circumcircle of triangle ABC at X. The line CH intersects
the perpendicular bisector of BC at E and the circumcircle of the triangle
ABC again at F . Point J lies on circle ω, passing through X, E, and F , such
that BCHJ is a trapezoid (CB k HJ). Prove that JB and EM meet on ω.
Proposed by Alireza Dadgarnia
---------------------------------------------------
Solution. Let D be the foot of altitude passing through A and P, K be the
intersection of lines EM, AC and JH, AM , respectively.
A

F
J H K
E

B D M C

From parallel lines, we have


ME DH MK
= = =⇒ EK k AC. (1)
EP HA KA
Note that ∠XKE = ∠XAC = ∠XF E. So K lies on ω. Let Q be the second
intersection point of line EM and circle ω. We have
(1)
∠KJQ = ∠KEP = ∠EP C = ∠QP C.
Note that it suffices to prove that ∠KJQ = ∠CBQ or prove that CP BQ is
a cyclic quadrilateral. Which is equivalent to M P · M Q = M B · M C. Also,
20 Intermediate Level

noting the parallel lines we can write M A = M K·M


ME
P
. Using this equation
and power of the point M with respect to the circumcircle of triangle 4ABC,
we have
MK · MX
MB · MC = MA · MX = · M P = M Q · M P.
ME
Where the last equation comes from power of the point M with respect to
circle ω. Hence proved. 

Comment. The same proof can be used to obtain the following generalised
result:
In triangle ABC point P is an arbitrary point and point D lies on the line
BC. The line AD intersects the circumcircle of triangle ABC at X. The line
CP intersects the line parallel to AP through D at E and the circumcircle
of triangle ABC again at F . Suppose that P lies inside of circle ω, passing
through X, E, and F . Point J lies on ω such that BCP J is a trapezoid
(CB k P J). Then JB and ED meet on ω.
Solutions 21

Problem 4. Triangle ABC is given. An arbitrary circle with center J,


passing through B and C, intersects the sides AC and AB at E and F ,
respectively. Let X be a point such that triangle F XB is similar to triangle
EJC (with the same order) and the points X and C lie on the same side of
the line AB. Similarly, let Y be a point such that triangle EY C is similar to
triangle F JB (with the same order) and the points Y and B lie on the same
side of the line AC. Prove that the line XY passes through the orthocenter
of the triangle ABC.
Proposed by Nguyen Van Linh - Vietnam
---------------------------------------------------
Solution. Let H be the orthocenter of triangle 4ABC, P be the intersection
of BE and CF . P H cuts the perpendicular bisector of BC at Z.
A
Z

E
B0
F

C0 P
H
X J
B C
Y

We have

∠HBP = ∠ABH −∠ABP = 90◦ −∠BAC−∠ABP = 90◦ −∠BEC = ∠JBC.

Then BH and BJ are isogonal lines with respect to angle ∠P BC. Similarly,
CH and CJ are isogonal lines with respect to angle ∠P CB. From this, we
deduce that H and J are isogonal conjugate with respect to triangle 4BP C.
Then ∠HP B = ∠JP C. But ZB = ZC, JF = JE and 4P F E ∼ 4P BC.
Therefore 4P F E ∪ {J} ∼ 4P BC ∪ {Z}. Which follows that 4JEF ∼
22 Intermediate Level

4ZCB.
Let B 0 , C 0 be the intersections of BH and AC, CH and AB, repectively. We
have
H
P(BE) = HB · HB 0 = HC · HC 0 = P(CF
H
),
P P
P(BE) = P B · P E = P C · P F = P(CF ).

We get Z lies on HP , which is the radical axis of circles with diameters BE


and CF . Analogously, X, Y also lie on HP . Therefore XY passes through
the orthocenter of triangle 4ABC. 
Solutions 23

Problem 5. Find all numbers n ≥ 4 such that there exists a convex polyhe-
dron with exactly n faces, whose all faces are right-angled triangles.
(Note that the angle between any pair of adjacent faces in a convex polyhe-
dron is less than 180◦ .)
Proposed by Hesam Rajabzadeh
---------------------------------------------------
Solution. If such a polyhedron exists for some n, the total number of sides
of faces is from one hand equal to 3n , and on the other is twice the number
of edges. So 3n is divisible by 2 and n must be even. We will give an example
of such a polyhedron for any even number n ≥ 4.

To this purpose, we need the following lemma.


Lemma 1. Let O be the origin in the 3-dimensional space and suppose X, Y
are two distinct points (different from O) in the xy-plane so that ∠OXY =
90◦ . Then for any point O0 on the z-axis, the triangle O0 XY is right-angled
(with ∠O0 XY = 90◦ ).

Proof. The proof is based on the Pythagorean Theorem. If O0 = O, there


is nothing to prove. If O0 6= O, the line OO0 (the z-axis) is perpendicular
to the xy-plane and so is perpendicular to every line in this plane passing
through O. In particular two triangles, O0 OX and O0 OY are right-angled.
According to the Pythagorean Theorem in these two triangles together with
triangle OXY , we have

O0 Y 2 = O0 O2 + OY 2 = O0 O2 + OX 2 + XY 2 = O0 X 2 + XY 2 .

which implies ∠O0 XY = 90◦ .


24 Intermediate Level

Now we return to the main problem. If n = 4, the tetrahedron with vertices


O0 , O, X, Y as in the lemma works (above figure). So we can assume n ≥ 6.
Take m = n−2 2 ≥ 2. First, we construct a convex (m + 2)-gon OA0 A1 · · · Am
in the xy-plane (take O to be the origin) so that
• OA0 = OAm .
• All the triangles of the form OAi Ai+1 (for 0 ≤ i ≤ m − 1) are right-
angled.

Consider m different rays with initial point O (denote them by l1 , . . . , lm


respectively in clockwise order) so that for a sufficiently small value of α,

∠l1 Ol2 = ∠l2 Ol3 = · · · = ∠lm−1 Olm = α. (1)

Take an arbitrary point on the ray l1 and call it A1 . Start from A1 and induc-
tively by drawing perpendiculars from Ai to li+1 define the points A2 , A3 , . . . , Am
so that
∠OA2 A1 = ∠OA3 A2 = · · · = OAm Am−1 = 90◦ . (2)
By (1) and (2) all the triangles OA1 A2 , OA2 A3 , . . . , OAm−1 OAm are similar.
OAm OA3
Therefore OA m−1
= · · · = OA 2
= OA OA1 . We denote this common value by
2

r < 1. Note that r can be arbitrarily close to 1 by taking α small. Now we


have
OAm OA3 OA2
OAm = .··· . . .OA1 = rm OA1 .
OAm−1 OA2 OA1
Note that since α is small all the points A2 , A3 , . . . , Am are on the same side
of the line OA1 . Take the point A0 on the other side of this line so that
∠OA0 A1 = 90◦ and OA0 = rm .OA1 (A0 is one of the intersection points
Solutions 25

of the circle with diameter OA1 and the circle with center O and radius
rm .OA1 ). If r is sufficiently close to 1 (equivalently α sufficiently close to
zero), rm will be close to one and we can ensure that ∠A0 OA1 is small and
so the polygon satisfies all desired properties.
After construction of the polygon, consider two points O0 , O00 on the z-axis (on
different sides of the xy-plane) with OO0 = OO00 = OA0 = OAm . Then the
polyhedron with vertices O0 , O00 , A0 , A1 , . . . , Am (convex hull of these points)
have exactly n = 2m + 2 faces, and all are right-angled triangles. Indeed, it
has 2m faces of the form O0 Ai Ai+1 and O00 Ai Ai+1 which are all right-angled
according to the lemma and two faces O0 A0 O00 and O0 Am O00 that are isosceles
right triangles.


26 Intermediate Level
Advanced Level

27
Problems

Problem 1. Let M , N , and P be the midpoints of sides BC, AC, and AB


of triangle ABC, respectively. E and F are two points on the segment BC
1 1
so that ∠N EC = ∠AM B and ∠P F B = ∠AM C. Prove that AE = AF .
2 2
(→ p.31)

Problem 2. Let ABC be an acute-angled triangle with its incenter I. Sup-


pose that N is the midpoint of the arc BAC of the circumcircle of triangle
ABC, and P is a point such that ABP C is a parallelogram. Let Q be the
reflection of A over N , and R the projection of A on QI. Show that the line
AI is tangent to the circumcircle of triangle P QR.
(→ p.33)

Problem 3. Assume three circles mutually outside each other with the
property that every line separating two of them have intersection with the
interior of the third one. √Prove that the sum of pairwise distances between
their centers is at most 2 2 times the sum of their radii.
(A line separates two circles, whenever the circles do not have intersection
with the line and are on different sides of it.)

Note. Weaker results with 2 2 replaced √ by some other c may be awarded
points depending on the value of c > 2 2.
(→ p.35)

Problem 4. Convex circumscribed quadrilateral ABCD with incenter I


is given such that its incircle is tangent to AD, DC, CB, and BA at K,
L, M , and N . Lines AD and BC meet at E and lines AB and CD meet
at F . Let KM intersects AB and CD at X and Y , respectively. Let LN
intersects AD and BC at Z and T , respectively. Prove that the circumcircle
of triangle XF Y and the circle with diameter EI are tangent if and only if

29
30 Advanced Level

the circumcircle of triangle T EZ and the circle with diameter F I are tangent.
(→ p.37)
Problem 5. Consider an acute-angled triangle ABC (AC > AB) with its
orthocenter H and circumcircle Γ. Points M and P are the midpoints of the
segments BC and AH, respectively. The line AM meets Γ again at X and
point N lies on the line BC so that N X is tangent to Γ. Points J and K
lie on the circle with diameter M P such that ∠AJP = ∠HN M (B and J
lie on the same side of AH) and circle ω1 , passing through K, H, and J,
and circle ω2 , passing through K, M , and N , are externally tangent to each
other. Prove that the common external tangents of ω1 and ω2 meet on the
line N H.
(→ p.43)
Solutions

Problem 1. Let M , N , and P be the midpoints of sides BC, AC, and AB


of triangle ABC, respectively. E and F are two points on the segment BC
1 1
so that ∠N EC = ∠AM B and ∠P F B = ∠AM C. Prove that AE = AF .
2 2
Proposed by Alireza Dadgarnia
---------------------------------------------------
Solution. Let H be the foot of the altitude passing through Q, A be the
midpoint of N P and K be the intersection point of N E and P F .

P Q N

B E H F M C

If we prove that points K, H and Q are collinear, using parallel lines ,we get
that H is the midpoint of EF which is equivalent to the problem. Clearly, AM
passes through Q and H is the reflection of A with respect to N P . Therefore,
∠P QH = ∠AQP = ∠AM B. So it suffices to show that ∠P QK = ∠AM B.
Note that
1
∠N EC + ∠P F B = (∠AM B + ∠AM C) = 90◦ =⇒ ∠EKF = 90◦ .
2

31
32 Advanced Level

So KQ is a median on the hypotenuse in triangle 4P KN and we’ll get

∠P QK = 2∠P N K = 2∠N EC = ∠AM B

which completes the proof. 


Solutions 33

Problem 2. Let ABC be an acute-angled triangle with its incenter I. Sup-


pose that N is the midpoint of the arc BAC of the circumcircle of triangle
ABC, and P is a point such that ABP C is a parallelogram. Let Q be the
reflection of A over N , and R the projection of A on QI. Show that the line
AI is tangent to the circumcircle of triangle P QR.
Proposed by Patrik Bak - Slovakia
---------------------------------------------------
Solution. Let M, S be the midpoint of segments BC, AI, respectively. By
a homothety with center A and ratio 21 , P goes to M , Q to N and R to
T ; Where T is the projection of A on SN . So it suffices to show that the
circumcircle of triangle 4M N T is tangent to AI.

N
A

T
S

B M C

We claim that this circle is tangent to AI at point I. We know that ∠N AS =


90◦ , So by the similarity of two triangles 4ASN , 4T SA, we’ll get

ST · SN = SA2 = SI 2 .

Therefore, SI is tangent to the circumcircle of triangle 4IT N . Now if we


show that SI is tangent to the circumcircle of triangle 4N IM as well, our
proof is completed; Because the circle passing through I and N and tangent
to SI is unique. Let D be the second intersection point of AI and circumcircle
of triangle 4ABC. Note that ∠DBM = ∠DCB = ∠DN B. Therefore,

DM · DN = DB 2 = DI 2 .
34 Advanced Level

Thus, DI is tangent to the circumcircle of triangle 4N IM and we’re done.



Solutions 35

Problem 3. Assume three circles mutually outside each other with the
property that every line separating two of them have intersection with the
interior of the third one. √Prove that the sum of pairwise distances between
their centers is at most 2 2 times the sum of their radii.
(A line separates two circles, whenever the circles do not have intersection
with the line and are on different sides of it.)

Note. Weaker results with 2 2 replaced √ by some other c may be awarded
points depending on the value of c > 2 2.
Proposed by Morteza Saghafian
---------------------------------------------------
Solution. According to the figure, we denote the radii of the circles by
r1 , r2 , r3 and the distance Oi Oj by dij . Moreover, let l, l0 be two interior
common tangents of circles ω1 and ω2 . We denote the tangency points of l
and l0 as in the figure. Obviously d12 = rsin1 +r2
α (α is defined in the figure).
Withour loss of generality we assume that r1 ≤ r2 .

l0

l
B0
A r2
r1
α
A0

By assumption we can deduce that both lines l and l0 must intersect the third
circle (ω3 ). If the intersection point of l and ω3 lies outside between A and
B, we can find a line separating ω1 and ω2 so which does not intersect ω3 and
this is a contradiction with the assumptions. We have similar arguments for
l0 . So we can assume that the intersection of ω3 with l and l0 is below B and
A0 respectively. Therefore, r3 is at least the radius of the circle tangent to l
at B and also is tangent to l0 (why?). The radius of this circle is r2 cot2 α.
Hence
1 − sin2 α d212
   
2 r1 + r2
r3 ≥ r2 cot α = r2 ≥ −1 .
sin2 α 2 (r1 + r2 )2
36 Advanced Level

Consequently,
d212 ≤ (r1 + r2 )2 + 2r3 (r1 + r2 ), (∗)
We have similar equations for d13 and d23 . Summing these three together
with Cauchy-Shwarz Inequality gives the assertion. Indeed,
X 2 X X X  X 2
dij ≤3 d2ij ≤ 6 ri2 + 18 ri rj ≤ 8 ri

Here the first and third inequality are coming from Cauchy-Shwarz Inequality
and the second inequality is the consequence of summing (∗) and two other
similar inequlities.

Remark. Using upper bound (r1 + r2 + r3 )2 for the right-hand side of (∗)
gives√d12 ≤ r1 + r2 + r3 . Summing these, gives a weaker result with 3 replaced
by 2 2. 
Solutions 37

Problem 4. Convex circumscribed quadrilateral ABCD with incenter I


is given such that its incircle is tangent to AD, DC, CB, and BA at K,
L, M , and N . Lines AD and BC meet at E and lines AB and CD meet
at F . Let KM intersects AB and CD at X and Y , respectively. Let LN
intersects AD and BC at Z and T , respectively. Prove that the circumcircle
of triangle XF Y and the circle with diameter EI are tangent if and only if
the circumcircle of triangle T EZ and the circle with diameter F I are tangent.
Proposed by Mahdi Etesamifard
---------------------------------------------------
Solution. First, let us prove these lemmas:
Lemma 1. Lines AC, BD, KM and LN are concurrent.
Proof. Using Brianchon’s Theorem in quadrilateral ABCD, one can simply
conclude the fact that AC, BD, KM and LN are concurrent.

Lemma 2. Let P be the point of concurrency of lines in Lemma 1. Therefore,


P is also the intersection point of quadrilateral ABCD’s diagonals and we
have IP ⊥EF .
Proof. We know that polar of point P is in fact line EF . Therefore, we’ll get
IP ⊥EF .

Lemma 3. A circle with diameter EI and the circumcircle of triangle 4XY J


are tangent.

C
J

D M

K I
Y

X F A B
38 Advanced Level

Proof. For the proof of tangency of circumcircle of triangle 4XY J to the


circle with diameter EI (circle ω2 ), it suffices that the equation of Casey’s
Theorem hold for points X, Y, J and circle ω2 .

±XY · PωJ2 ± XJ · PωY2 ± Y J · PωX2 = 0.

Since PωJ2 = 0, Therefore,


√ √
XJ Y K · Y M = Y J XK · XM (1)
Since X, Y lie on the radical axis of two circles ω and ω2 , We have:
(1)
Y K · Y M = Y L2 , XK · XM = XN 2 =⇒ XJ · Y L = Y J · XN (2)

So, we have to prove equation (2). Using Menelaus’s Theorem for triangle
4XF Y and line LN P , We have:
XN F L Y P F N =F L XN XP
· · =⇒ = .
F N Y L XP YL YP
From equation (2), we get:
XJ XN XP
= = .
YJ YL YP
Therefore we need to prove that JP is the exterior angle bisector of angle
∠XJY . Since JQ⊥JP , we need to prove that (XY, QP ) = −1.
NL
(XY, P Q) = F (XY, P Q) = (N L, P U ) = −1.

And since point U lies on EF (polar of P ), the last equation holds and we’re
done.

Lemma 4. AK is tangent to the circumcircle of triangle 4ABC if and only


if
 2
BK AB
= .
KC AC

Proof. Using The Law of Sines and Ratio Lemma, one can simply get the
desired results.

Lemma 5. If angle bisectors of angles ∠E and ∠F are perpendicular, then


ABCD is a cyclic quadrilateral.
Solutions 39

Proof. It’s trivial.

J
C
L
D M
K
I
S
Y

X F A N B

Now, Let’s get back to the problem. First, we assume that two circles ω1 and
ω2 are tangent to each other. Let S be the foot of the perpendicular line to
F I passing through E. Using Casey’s Theorem for points X, F, Y and circle
ω2 , we have:

q q q
± XF PωY2 ± Y F
PωX2 ± XY PωF2 = 0
√ √ √
=⇒ ± XF Y K · Y M ± Y F XK · XM ± XY F S · F I = 0. (3)

Points X and Y lie on the radical axis of circles ω and ω2 . Therefore we have:

Y K · Y M = Y L2 , XK · XM = XN 2 .

So equation (1) can be written as:



± XF · Y L ± Y F.XN ± XY F S · F I = 0. (4)
α+β
According to the figure, We have: ∠F1 = ∠F2 = 2 .

 
α+β
Y L = F L ± F Y = F I · cos (F1 ) ± F Y = F I · cos ± F Y,
2
 
α+β
XN = F N ∓ XF = F I · cos (F2 ) ∓ XF = F I · cos ∓ XF.
2
40 Advanced Level

Now, by putting them in equation (4), We’ll get:

   
α+β
± XF · F I · cos ± FY
2

   
α+β
± Y F · F I · cos ∓ XF ± XY F S · F I = 0
2

   
α+β
=⇒ ± F I XF + Y F cos = ±XY F S · F I
2

   
XF + Y F α+β
=⇒F I cos = FS · FI
XY 2
    r
α+β sin α + sin β FS
=⇒ cos · =
2 sin α + β FI
 
α−β FS
=⇒ cos2 = . (5)
2 FI

C
R
M
D L
V Q
K
I
Y β S

α
X F A N B

Also, we have:

   
α+β α−β
∠F RS = 90◦ − ⇒ ∠QV R = 90◦ −
2 2
 
α − β
⇒ ∠EIF = 90◦ − .
2

So, by equation (5), we have:


FS
sin2 (EIF ) = . (6)
FI
Solutions 41

We consider three cases for point S on line F I:


Case 1) ∠EIF = 90◦ . Which gives us that S and I coincide.
FS
sin2 (EIF ) = = 1.
FI
Now, by Lemma 5, ABCD is a cyclic quadrilateral. On the other
hand, ABCD is circumscribed and every equation resulted from Casey’s
Theorem for the circumcircle of triangle 4XF Y and the circle with
diameter EI, can be written for the circumcircle of triangle 4T EZ and
the circle with diameter F I as well. So by Casey’s Theorem, these two
circles are tangent to each other.
Case 2) ∠EIF < 90◦ . 
2 ES FS
sin (EIF ) = = .
EI FI
Now by Lemma 4, we get that EF is tangent to the circumcircle of
triangle 4ESI and
∠F ES = ∠F IF =⇒ ∠IEF = 90◦ .

J ≡E Y
T
C
D

F X A B

Now since ∠IEF = 90◦ , the foot of perpendicular line to EF passing


through I, (Point J) coincides with point E. By Lemma 3, the cir-
cumcircle of triangle 4T JZ (which is also the circumcircle of triangle
42 Advanced Level

4T EZ), will be tangent to the circle with diameter F I. In this case,


tangency point of the circumcircle of triangle 4T EZ and the circle with
diameter EI, will be point I and tangency point of the circumcircle of
triangle 4T EZ and the circle with diameter F I, will be point E.

Case 3) ∠EIF > 90◦ . Since

FS
sin2 (EIF ) = > 1,
FI
this case will never happen.


Solutions 43

Problem 5. Consider an acute-angled triangle ABC (AC > AB) with its
orthocenter H and circumcircle Γ. Points M and P are the midpoints of the
segments BC and AH, respectively. The line AM meets Γ again at X and
point N lies on the line BC so that N X is tangent to Γ. Points J and K
lie on the circle with diameter M P such that ∠AJP = ∠HN M (B and J
lie on the same side of AH) and circle ω1 , passing through K, H, and J,
and circle ω2 , passing through K, M , and N , are externally tangent to each
other. Prove that the common external tangents of ω1 and ω2 meet on the
line N H.
Proposed by Alireza Dadgarnia
---------------------------------------------------
Solution 1. Let D be the intersection of AH and BC. Denote Ω by the
circle with diameter P M . It’s obvious that D lies on Ω. Also since ABC is
acute, H lies on the segment P D and so inside of Ω. N lies on the extension
of DM and so outside of Ω. We claim that there are at most two possible
cases for K. The following lemma proves our claim.

Lemma. Given a circle ω and four points A, B, C, and D, such that A and
B lie on the circle, C inside and D outside of the circle. There are exactly
two points like K on ω so that the circumcircles of triangles ACK and BDK
are tangent to each other.

Proof. Invert the whole diagram at center A with arbitrary radius, the images
of points and circle are denoted by primes. Since A lies on ω, ω 0 is a line,
passes through B 0 and K 0 . Notice that C 0 and D0 lie on the different sides of
ω 0 . Since the circumcircles of triangles ACK and BDK are tangent to each
other, we have C 0 K 0 is tangent to the circumcircle of triangle B 0 D0 K 0 . It
means ∠C 0 K 0 B 0 = ∠B 0 D0 K 0 . Let X and Y be two arbitrary points, lie on
ω 0 and the different sides of B 0 .

C0

B0 K0 ω0
Y −→ X

D0
44 Advanced Level

First assume that K 0 ≡ B 0


so ∠C 0 B 0 Y = ∠C 0 K 0 B 0 > 0 = ∠K 0 D0 B 0 and when
−−0→
K moves along the ray B X, ∠C 0 K 0 B 0 decreases and ∠K 0 D0 B 0 increases. It
0
−−→
yields there is exactly one point K 0 on the ray B 0 X so that ∠C 0 K 0 B 0 =
∠B 0 D0 K 0 . In the same way we get there is only one possible case for K 0 on
−−→
the ray B 0 Y and the result follows.

Γ
A A0

L Ω
P

H
J

B D M C N

Denote ω1 and ω2 by the circumcircles of triangles AJP and HN D. Let H


be the indirect homothety that sends ω1 to ω2 . Notice that J and N lie on
the different sides of AH. Now since the arc AP of ω1 is equal to the arc
HD of ω2 and AP k HD, H sends A to D and P to H therefore (A, H) and
(P, D) are anti-homologous pairs. Let L be the anti-homologous point of J
under H. It’s well-known that the pairs of anti-homologous points lie on a
circle so ALHJ and LP JD are cyclic quadrilaterals.
Let E be the reflection of A over the point M . We claim that HDEN is
cyclic. A0 lies on Γ so that AA0 k BC. We know that (A0 X, BC) = −1 hence
N A0 is tangent to Γ. Also by symmetry N E is tangent to the circumcircle
of triangle CEB. Now since HE is the diameter of this circle, we have
∠N EH = 90◦ = ∠N DH and our claim is proved. The line AM meets the
Solutions 45

circumcircle of triangle P DM again at L0 . We have

AL0 · AM = AP · AD =⇒ AL0 · AE = AH · AD

it follows that L0 HDEN is cyclic so L0 ≡ L. We have

∠P JH = ∠AJH − ∠AJP = ∠HLM − ∠HN D


= ∠HLM − ∠HLD = ∠DLM = ∠DJM

therefore ∠HJD = 90◦ . From this we can conclude that the cirucmcircles
of triangles DHJ and DM N are tangent to each other and the common
external tangents of them are concurrent at H since the tangent line to the
circumcircle of triangle DHJ through H is parallel to DM N . So the problem
is proved for K ≡ D, now suppose that K 6= D. Since ∠AHL = ∠LN M
the circumcircles of triangles LHJ and LM N are tangent to each other. So
L ≡ K. Denote O1 and O2 by the circumcenters of triangles LHJ and LM N .
It’s obvious that O1 , L, and O2 are collinear so ∠O1 LH + ∠O2 LN = 90◦ . It
yields

∠HO1 L = 180◦ − 2∠O1 LH = 2∠O2 LN = 180◦ − ∠LO2 N =⇒ O1 H k O2 N

therefore the direct homothety that sends (O1 ) to (O2 ), sends H to N and
the conclusion follows. 

Solution 2. Let D be the intersection of AH and BC. Denote Ω by the


circle with diameter P M . It’s obvious that D lies on Ω.

A J0 Γ

L Ω
P

F
H O
J

N0 B D M C N

X0 X
46 Advanced Level

Let F be the intersection of N H and M J. Since J and B lie on the same


side of P D, J lies on the arc P D (the one that does not contain M ) so J and
H lie on the same side of BC. Also

∠HN M = ∠AJP < ∠JP D = ∠JM D

therefore F and J lie on the same side of N M and we have 4F M N ∼


4AP J since ∠JP D = ∠JM D. It follows that A, F , H, and J are concylic.
Let J 0 and N 0 be the reflections of J and N over the points P and M ,
respectively. Since P is the midpoint of AH, AJ 0 HJ is a parallelogram. The
A−symmedian meets Γ again at X 0 . Since XX 0 k BC, by symmetry N 0 X 0
is tangent to Γ, too. Also we know that (AX 0 , BC) = −1 so N 0 A is tangent
to Γ. Now 4F M N ∼ 4AP J yields 4F M N 0 ∼ 4AP J 0 . It follows that

∠N 0 F M = ∠J 0 AP = ∠AHJ = 180◦ − ∠AF J

hence A, F , and N 0 are collinear. Again from 4F M N 0 ∼ 4AP J 0 we get

∠P JH = ∠AJ 0 P = ∠F N 0 M = 90◦ − ∠P M N 0 = ∠DP M = ∠DJM

In the third equality we used that M P ⊥ AN 0 (It’s a well-known property,


If we let O be the center of Γ then AP M O is a parallelogram). It yields
∠HJD = ∠P JM = 90◦ . Like the first solution we know that there are at
most two possible cases for K and we can conclude that D is one of them.
Now we suppose that K 6= D. Let AM meets Ω again at L. We have

∠LAH = 90◦ − ∠LM D = ∠LJD − 90◦ = ∠LJH

therefore ALHJ is cyclic. Since M P ⊥ AN 0 and AP ⊥ M N 0 , P is the


orthocenter of triangle AN 0 M and N 0 P ⊥ AM . It follows that N 0 , P and L
lie on a same line. Now since ∠ALP = ∠N 0 LM = 90◦ and ∠AP L = ∠N 0 M L,
we have 4AP L ∼ 4N 0 M L. It yields 4LM N ∼ 4LP H. Hence

∠M LN = ∠P LH =⇒ ∠HLN = ∠P LM = 90◦

so LN DH is cyclic and ∠AHL = ∠LN M . It follows that the circumcircles


of triangles LHJ and LM N are tangent to each other. So L ≡ K. Denote
O1 and O2 by the circumcenters of triangles LHJ and LM N . It’s obvious
that O1 , L, and O2 are collinear so ∠O1 LH + ∠O2 LN = 90◦ . It yields

∠HO1 L = 180◦ − 2∠O1 LH = 2∠O2 LN = 180◦ − ∠LO2 N =⇒ O1 H k O2 N

therefore the direct homothety that sends (O1 ) to (O2 ), sends H to N and
the conclusion follows. 
Solutions 47

Comment. We can also prove LHDN is cyclic by angle-chasing. We have

∠DLM = ∠DP M = 90◦ − ∠P M D = ∠P JD − 90◦ = ∠P JH

also ∠HLM = ∠AJH so ∠HLD = ∠AJP = ∠HN D and it follows that


LHDN is cyclic.
8th Iranian Geometry Olympiad
November 5, 2021

Contest problems with solutions


8th Iranian Geometry Olympiad
Contest problems with solutions.

This booklet is prepared by Elahe Zahiri, Mahdi Shavali, Amirmohammad Derakhshandeh and
Alireza Dadgarnia.
With special thanks to Hesam Rajabzade, Mahdi Etesamifard and Morteza Saghafian.

Copyright ©Iranian Geometry Olympiad Secretariat 2020-2021. All rights reserved.


3

Participating Countries
List of participated nations at the 8th Iranian Geometry Olympiad:

Afghanistan Albania Argentina


Armenia Austria Bangladesh
Belarus Bolivia Bosnia and Herzegovina
Brazil Bulgaria China
Colombia Costa Rica Croatia
Cuba Czech Repbulic Dominican Republic
El Salvador Estonia Finland
Hong Kong India Iran
Ireland Italy Kazakhstan
Kosovo Kyrgyzstan Macedonia
Malaysia Mexico Mongolia
Nepal Netherlands Nicaragua
Nigeria Pakistan Panama
Paraguay Philippines Poland
Republic of Moldova Romania Russia
Slovakia Slovenia South Africa
Sweden Syria Tajikistan
Turkey Turkmenistan Ukraine
Uzbekistan Venezuela Vietnam
4

Contributing Countries
The Organizing Committee and the Scientific Committee of the IGO 2021 thank the following
countries for contributing 100 problem proposals:
Czech Republic, Hong Kong, India, Iran, Mexico,
Poland, Russia, Slovakia, United Kingdom, Vietnam.

Scientific Committee

Patrik Bak Alireza Mahdi Boris Frenkin Radek Olšák Parsa Hosseini
(Slovakia) Dadgarnia Etesamifard (Russia) (Czech Nayyeri
(Iran) (chair - Iran) Republic) (Iran)

Morteza Josef Tkadlec Davood Vakili Ali Zamani Alexey


Saghafian (Czech (Iran) (Iran) Zaslavsky
(Iran) Republic) (Russia)

With special thanks to:


Alireza Shavali, Elahe Zahiri, Seyyed Mohammadreza Khosravian, Mahdi Shavali, Arefeh Booshehrian,
Alireza Danaei, Mohamad Moshtaghifar, Amirmohammad Derakhshandeh, Seyyed Yasin Kho-
rashadizadeh, Seyyed Yasin Moosavi, Niki Hasani, Morteza Gholizadeh, Seyyed Hassan Ghafoory.
Contents
Elementary Level 3
Problems . . . . . . . . . . . . . . . . . . . . . . . . . . . . . . . . . . . . . . . . . . . . . . . . . . . . . . . . . . . . . . . . . . . . . . 3

Solutions . . . . . . . . . . . . . . . . . . . . . . . . . . . . . . . . . . . . . . . . . . . . . . . . . . . . . . . . . . . . . . . . . . . . . . 5

Intermediate Level 15
Problems . . . . . . . . . . . . . . . . . . . . . . . . . . . . . . . . . . . . . . . . . . . . . . . . . . . . . . . . . . . . . . . . . . . . . . 15

Solutions . . . . . . . . . . . . . . . . . . . . . . . . . . . . . . . . . . . . . . . . . . . . . . . . . . . . . . . . . . . . . . . . . . . . . . 17

Advanced Level 27
Problems . . . . . . . . . . . . . . . . . . . . . . . . . . . . . . . . . . . . . . . . . . . . . . . . . . . . . . . . . . . . . . . . . . . . . . 27

Solutions . . . . . . . . . . . . . . . . . . . . . . . . . . . . . . . . . . . . . . . . . . . . . . . . . . . . . . . . . . . . . . . . . . . . . . 29
Elementary Level

1
Problems

Problem 1. With putting the four shapes drawn in the following figure together make a shape
with at least two reflection symmetries.

(→ p.5)
Problem 2. Points K, L, M , N lie on the sides AB, BC, CD, DA of a square ABCD,
respectively, such that the area of KLM N is equal to one half of the area of ABCD. Prove that
some diagonal of KLM N is parallel to some side of ABCD.
(→ p.6)
Problem 3. As shown in the following figure, a heart is a shape consist of three semicircles with
diameters AB, BC and AC such that B is midpoint of the segment AC.
A heart ω is given. Call a pair (P, P ′ ) bisector if P and P ′ lie on ω and bisect its perimeter.
Let (P, P ′ ) and (Q, Q′ ) be bisector pairs. Tangents at points P , P ′ , Q, and Q′ to ω construct a
convex quadrilateral XY ZT . If the quadrilateral XY ZT is inscribed in a circle, find the angle
between lines P P ′ and QQ′ .

A C
B

(→ p.7)

3
4 Elementary Level

Problem 4. In isosceles trapezoid ABCD (AB ∥ CD) points E and F lie on the segment CD in
such a way that D, E, F and C are in that order and DE = CF . Let X and Y be the reflection
of E and C with respect to AD and AF . Prove that circumcircles of triangles ADF and BXY
are concentric.
(→ p.9)
Problem 5. Let A1 , A2 , . . . , A2021 be 2021 points on the plane, no three collinear and

∠A1 A2 A3 + ∠A2 A3 A4 + · · · + ∠A2021 A1 A2 = 360◦ ,

in which by the angle ∠Ai−1 Ai Ai+1 we mean the one which is less than 180◦ (assume that A2022 =
A1 and A0 = A2021 ). Prove that some of these angles will add up to 90◦ .
(→ p.12)
Solutions

Problem 1. With putting the four shapes drawn in the following figure together make a shape
with at least two reflection symmetries.

Proposed by Mahdi Etesamifard - Iran


------------------------------------------------------------------
Solution. One can put the figures together like the below figure.

5
6 Elementary Level

Problem 2. Points K, L, M , N lie on the sides AB, BC, CD, DA of a square ABCD,
respectively, such that the area of KLM N is equal to one half of the area of ABCD. Prove that
some diagonal of KLM N is parallel to some side of ABCD.
Proposed by Josef Tkadlec - Czech Republic
------------------------------------------------------------------
Solution 1. Let [P ] denote the area of a polygon P . Suppose that LN ∦ AB and let N ′ ̸= N
be the point on AD such that LN ′ ∥ AB. Note that [KLM N ′ ] = 21 [ABCD]. Thus [KLM N ] =
[KLM N ′ ], hence [KM N ] = [KM N ′ ] implying that KM ∥ N N ′ = AD.

A K B

N′ L

D M C

Solution 2 (Proposed Solution from Slovakia).


Denote the length as on the figure and let the side of ABCD be a. The sum of the areas of the
right triangles in the corners is a2 /2 and so

1 1 1 1 1
wx + (1 − x)y + (1 − y)(1 − z) + z(1 − w) = a2
2 2 2 2 2
which can be modifier into (x − z)(w − y), and so x = z or y = w, which gives the desired
conclusion.

A x K B

y
w
L

D z C
M
Solutions 7

Problem 3. As shown in the following figure, a heart is a shape consist of three semicircles with
diameters AB, BC and AC such that B is midpoint of the segment AC.
A heart ω is given. Call a pair (P, P ′ ) bisector if P and P ′ lie on ω and bisect its perimeter.
Let (P, P ′ ) and (Q, Q′ ) be bisector pairs. Tangents at points P , P ′ , Q, and Q′ to ω construct a
convex quadrilateral XY ZT . If the quadrilateral XY ZT is inscribed in a circle, find the angle
between lines P P ′ and QQ′ .

A C
B

Proposed by Mahdi Etesamifard - Iran


------------------------------------------------------------------
Solution. We prove the statement of the problem for both convex and non-convex quadrilaterals.
Lemma 1. If a pair (P, P ′ ) is bisector, then the points P , P ′ , and B are collinear.

Proof. Without loss of generality suppose that P lies on the arc AC in such a way that the
intersection of P B with the arc AB is P ′′ . It’s clear that the length of the arc P ′′ A is equal to

2∠P ′′ BA
× (the perimeter of the semicircle with diameter AB)
180◦
and length of the arc P C is equal to

∠P BC
× (the perimeter of the semicircle with diameter AC)
180◦
We know that the perimeter of semicircle with diameter AB is π·AB
2
and the perimeter of semicircle
′′ BA
′′
with diameter AC is 2 thus the length of the arc P A is equal to 2∠P
π·AC
180◦
× π·AB
2
and the length
∠P BC π·AC ′′
of the arc P C is equal to 180◦ × 2 which are equal with each other. So (P, P ) is a bisector
pair. But there is exactly one point for each P like P ′ such that the pair (P, P ′ ) is bisector, so
P ′ ≡ P ′′ . Hence P P ′ passes through B.

P ′′

A C
B

P
8 Elementary Level

Without loss of generality suppose P and Q are on the arc AC. Now we consider these 2 cases:
Case 1. P and Q lie on the arc AC in such a way that P ′ and Q′ both lie on the arc AB.
Notice that ∠P BQ = 2∠XP Q = 2∠XQP and ∠P ′ BQ′ = ∠ZP ′ Q′ = ∠ZQ′ P ′ . Therefore

180◦ = ∠P XQ + ∠P ′ Q′ X = (180◦ − ∠P BQ) + (180◦ − 2∠P ′ BQ′ = 360◦ − 3∠P BQ,

hence ∠P BQ = 60◦ .

Z Z ℓ

P′ Q′
T
P′ Q′ Y
A C A B
B C
P P
Q
X
Q Y

X
Case 1 Case 2

Case 2. P and Q lie on the arc AC in such a way that P ′ lies on the arc AB and Q′ lies on the
arc BC.
Let ℓ be the tangent line from B to the heart. Thus ∠P ′ Bℓ = ∠BP ′ Z and ∠Q′ Bℓ = ∠BQ′ Z. So
∠P ′ ZQ′ = 360◦ − 2∠P ′ BQ′ . Now it’s not hard to see that ∠P BQ = 2∠XP Q = 2∠XQP , hence
∠P XQ = 180◦ − ∠P BQ. Thus we must have

180◦ = ∠P XQ + ∠P ′ ZQ′ = (180◦ − ∠P BQ) + (360◦ − 2∠P ′ BQ′ ) = 540◦ − 3∠P BQ,

so ∠P BQ = 120◦ .
Hence in both cases the angle between the lines P P ′ and QQ′ is 60◦ .
Solutions 9

Problem 4. In isosceles trapezoid ABCD (AB ∥ CD) points E and F lie on the segment CD in
such a way that D, E, F and C are in that order and DE = CF . Let X and Y be the reflection
of E and C with respect to AD and AF . Prove that circumcircles of triangles ADF and BXY
are concentric.
Proposed by Iman Maghsoudi - Iran
------------------------------------------------------------------
Solution 1. Consider point Z on AB in such a way that AZF D is an isosceles trapezoid hence
it is cyclic. Let O be the circumcenter of △AF D. Since X and Y are the reflections of E and C
with respect to AD and AF , and ZBCF is a parallelogram, then

ED = XD = CF = F Y = ZB

Suppose that AF meets CY at H. Now notice that

∠OZB = ∠OZF + ∠F ZB = 90◦ − ∠ZDF + ∠BCD = 90◦ − ∠AF D + ∠ADC


∠ODX = ∠ODA + ∠ADX = 90◦ − ∠AF D + ∠ADC
∠OF Y = 180◦ − ∠OF A − ∠Y F H = 180◦ − (90◦ − ∠ADC) − ∠HF C = 90◦ − ∠AF D + ∠ADC

These three equality with ZB = XD = F Y and OZ = OD = OF gives us that

△OZB ∼
= △ODX ∼
= △OF Y =⇒ OB = OX = OY

Which concludes the proof.

A Z B

O
F
D C
E
H

Solution 2. Let ω be the circumcircle of AF D with center O and let this circle meets AB, XD,
and Y F at T , K, and L respectively. Clearly AX = AE, AC = AY and ABF E is an isosceles
trapezoid, since X and Y are reflections of E and C with respect to AD and AF , and DE = CF .
Then

∠BAE = ∠AED = ∠AXD


∠ABE = ∠AF D = ∠AKX
10 Elementary Level

These two equality with AX = AE show that △ABE ∼ = △AXK and it yields that AB = XK.
Similarly one can show that AB = Y L. Now notice that

PωB = BT · AB = OB 2 − OD2
PωX = XD · XK = OX 2 − OD2
PωY = Y F · Y L = OY 2 − OD2

We have already shown that AB = XK = Y L and BT = XD = Y F . So the power of the points


B, X, and Y with respect to ω are all equal, hence OB = OX = OY and the conclusion follows.

A T B

X L
O

F
D C
E

K
Y

Solution 3 (Proposed Solution from Slovakia).


Lemma 1. Let ABCD be a parallelogram and D′ be the reflection of D in AC. Then the
perpendicular bisector of AC coincidences with the perpendicular bisector of BD′ (which in case
B = D′ degenerated to the perpendicular line to AC through B).

Proof. Obvious when we place AC horizontally:


D′

A
B

D C
Solutions 11

Let G be such a point on opposite ray AB such that AG = CF (which equals do ED). Clearly,
DEAG and F CAG are parallelograms. If we apply our lemma to them, we can conclude that
the perpendicular bisectors of line pairs (GX, AD) and (GY, AF ) coincidence, which gives that
triangles GXY and ADF have a common circumcenter O.
To finish the proof, we need to show that B lies on the circle GXY . Let H be the second
intersection point of circumcircle of △ADF and AB. Then AHF D is an isosceles trapezoid and
clearly HBCF is a parallelogram, and so AG = CF = BH. Then △OAG ∼ = △OHB, and so
OG = OB, therefore points X, Y, B, G are concyclic and we’re done.

G A H B

F
D C
E O

Y
12 Elementary Level

Problem 5. Let A1 , A2 , . . . , A2021 be 2021 points on the plane, no three collinear and

∠A1 A2 A3 + ∠A2 A3 A4 + · · · + ∠A2021 A1 A2 = 360◦ ,

in which by the angle ∠Ai−1 Ai Ai+1 we mean the one which is less than 180◦ (assume that A2022 =
A1 and A0 = A2021 ). Prove that some of these angles will add up to 90◦ .
Proposed by Morteza Saghafian - Iran
------------------------------------------------------------------
Solution. Let αi be the angle ∠Ai−1 Ai Ai+1 which is less than 180◦ .

Ai+1 Ai
αi

Ai−1

Starting from A1 , we walk on the perimeter of the (not necessarily simple) polygon A1 A2 . . . A2021 .
As we reach a vertex Ai , we turn by angle 180◦ − αi in clockwise or counterclockwise direction.
After walking one round and returning back to the edge A1 A2 , we have turned by a multiple of
360◦ in total. Therefore, the signed sum of turning angles is a multiple of 360◦ . More formally, if
we define C1 and C2 for the set of clockwise and counterclockwise angles, then for some integer
number k we have X X
360◦ k = (180◦ − αi ) − (180◦ − αj )
αi ∈C1 αj ∈C2

But the total number of angles we have is 2021 which is an odd number, so if we cancel numbers
180 as much as possible from the above expression, we can conclude that
X X
360◦ t + 180◦ = αi − αj
αi ∈C1 αj ∈C2

for
P some integer P number t. On the other hand, by the assumption of the
P problem, the sum
◦ ◦
P
αi ∈C1 αi + αj ∈C2 αj is equal to 360 . This implies that αi ∈C1 αi − αj ∈C2 αj = ±180 .
◦ ◦
Therefore, the two sums should be 90 and 270 .
Intermediate Level

13
Problems

Problem 1. Let ABC be a triangle with AB = AC. Let H be the orthocenter of ABC. Point
E is the midpoint of AC and point D lies on the side BC such that 3CD = BC. Prove that
BE ⊥ HD.
(→ p.17)
Problem 2. Let ABCD be a parallelogram. Points E, F lie on the sides AB, CD respectively,
such that ∠EDC = ∠F BC and ∠ECD = ∠F AD. Prove that AB ≥ 2BC.
(→ p.18)
Problem 3. Given a convex quadrilateral ABCD with AB = BC and ∠ABD = ∠BCD = 90◦ .
Let point E be the intersection of diagonals AC and BD. Point F lies on the side AD such that
AF CE
= . Circle ω with diameter DF and the circumcircle of triangle ABF intersect for the
FD EA
second time at point K. Point L is the second intersection of EF and ω. Prove that the line KL
passes through the midpoint of CE.
(→ p.21)
Problem 4. Let ABC be a scalene acute-angled triangle with its incenter I and circumcircle Γ.
Line AI intersects Γ for the second time at M . Let N be the midpoint of BC and T be the point
on Γ such that IN ⊥ M T . Finally, let P and Q be the intersection points of T B and T C,
respectively, with the line perpendicular to AI at I. Show that P B = CQ.
(→ p.22)
Problem 5. Consider a convex pentagon ABCDE and a variable point X on its side CD.
Suppose that points K, L lie on the segment AX such that AB = BK and AE = EL and that
the circumcircles of triangles CXK and DXL intersect for the second time at Y . As X varies,
prove that all such lines XY pass through a fixed point, or they are all parallel.
(→ p.24)

15
16 Intermediate Level
Solutions

Problem 1. Let ABC be a triangle with AB = AC. Let H be the orthocenter of ABC. Point
E is the midpoint of AC and point D lies on the side BC such that 3CD = BC. Prove that
BE ⊥ HD.
Proposed by Tran Quang Hung - Vietnam
------------------------------------------------------------------
Solution. Let G be the centroid of triangle ABC. Notice that
CD 1 EG
= = ,
BC 3 EB
hence GD ∥ EC. Since H is orthocenter of ABC, BH ⊥ AC, we deduce that BH ⊥ GD.
Combining with GH ⊥ BC, we get that H is the orthocenter of triangle BGD, therefore DH ⊥
BE. This completes the proof.

B C
D

17
18 Intermediate Level

Problem 2. Let ABCD be a parallelogram. Points E, F lie on the sides AB, CD respectively,
such that ∠EDC = ∠F BC and ∠ECD = ∠F AD. Prove that AB ≥ 2BC.
Proposed by Pouria Mahmoudkhan Shirazi - Iran
------------------------------------------------------------------
Solution 1. First we do some angle-chasing. It’s clear that ∠DAF = ∠ECB = ∠CEB and
∠EBC = ∠ADF . Hence △DAF ∼ △BEC and it yields that AD BE
= DF
BC
. Similarly we can show
CF BC
that AD = AE . Multiplying these two equality gives that
CF DF
= =⇒ EF ∥ AD ∥ BC.
BE AE
Let F ′ be the reflection of F in E. So BCEF ′ and ADEF ′ are parallelograms. Thus ∠BF ′ E =
∠BCE = ∠BAF and ∠AF ′ E = ∠ADE = ∠ABF . Summing up these two equality implies that
∠AF ′ B = 180◦ − ∠AF B. Therefore the quadrilateral AF ′ BF is cyclic. It means that
AE · EB = EF · EF ′ = EF 2 = BC 2 .
Finally, by AM-GM inequality we have
 2  2
2 AE + EB AB
BC = AE · EB ≤ = =⇒ 2BC ≤ AB.
2 2

F′

A E
B

D C
F

Solution 2. Let CE and DE meet AD and BC at X and Y , respectively. It’s clear that the
quadrilaterals DY BF and CXAF are cyclic. The power of the points C and D with respect to
the circumcircles of these quadrilaterals gives that CF · CD = CB · CY and DF · CD = DA · DX.
Summing up these two equations implies that
(CF + DF ) · CD = CB · CY + DA · DX
=⇒ CD2 = CB · CY + CB · DX
=⇒ CD2 = (CY + DX) · CB (1)
Now let the line parallel to BC from E intersects CD at G. Then
DX CD CD · EG CD · BC
= =⇒ DX = =
EG CG CG CG
CY CD CD · EG CD · BC
= =⇒ CY = = .
EG DG DG DG
Solutions 19

E
A B

D C
F ≡G

Thus
 
1 1
DX + CY = CD · BC + .
CG DG

By applying AM-HM inequality on the last equation we get

!
1 1 4
CD · BC · + ≥ CD · BC ·
CG DG CG + DG
4
= CD · BC ·
CD
= 4BC.

Now from (1) we have

CD2
= DX + CY ≥ 4BC =⇒ CD2 ≥ 4BC 2
BC
=⇒ AB ≥ 2BC,

as required.
Remark. From the first solution we know that the points F and G are the same but in the
second solution we do not need to prove that.
Solution 3 (Proposed Solution from Slovakia).
As in the official solution, we prove that AD ∥ EF ∥ BC. Along the way, we have △EAD ∼
△BCF . Therefore:

AB AE + BE AE BE BC CF
= = + = + ≥2
AD AD AD AD CF BC
20 Intermediate Level

A E B

D C
F

Solution 4 (Proposed Solution from Slovakia).


As in every solution, we notice △ADF ∼ △EBC and △ADE ∼ △CF B. Now, denote lengths
AD EB AD CF
as on the picture. The similarities give us = , i.e. b2 = (a − x)y, and = , i.e.
DF CB AE CB
b2 = (a − y)x. By comparing these two, we have (a − x)y = (a − y)x, i.e. ay = ax, and so x = y.
Therefore b2 = (a − x)x, which modifies as x2 − ax + b2 = 0. This quadratic equation has a
discriminant of a2 − 4b2 , which must be non-negative, which already gives a ≥ 2b.

A x E a−x B

b
b

D y a−y C
F
Solutions 21

Problem 3. Given a convex quadrilateral ABCD with AB = BC and ∠ABD = ∠BCD = 90◦ .
Let point E be the intersection of diagonals AC and BD. Point F lies on the side AD such that
AF CE
= . Circle ω with diameter DF and the circumcircle of triangle ABF intersect for the
FD EA
second time at point K. Point L is the second intersection of EF and ω. Prove that the line KL
passes through the midpoint of CE.
Proposed by Mahdi Etesamifard and Amir Parsa Hosseini - Iran
------------------------------------------------------------------
Solution. Let K ′ be a point such that the triangles ABC and AK ′ D are spirally similar. We
claim that K and K ′ are the same point. Since EC
AE
= DF
FA
, the triangles F K ′ D and EBA are also
′ ◦
similar. Then ∠F K D = ∠ABE = 90 , so to prove the claim it suffices to show that A, B, F ,
and K ′ lie on a circle. From the other hand

∠DEC = 90◦ − ∠BAC = 90◦ − ∠BCA = ∠DCE, (1)

hence DC = DE. Also since ABC and AK ′ D are spirally similar, we have △ABK ′ ∼ △ACD.
Therefore
(1)
∠ABK ′ = ∠ACD = ∠DEC = ∠AEB = ∠K ′ F D,
so ABK ′ F is cyclic and the claim is proved.
Now we are ready to prove the statement of problem. Let M be the midpoint of CE. Notice that
∠DM E = 90◦ , since DC = DE. It yields that the points E, L, D, and M lie on a circle. Then

∠M LD = ∠M ED = ∠AEB = ∠KF D = ∠KLD,

which implies the desired collinearity.

A
E
M
C
L
K ≡ K′
F

D
22 Intermediate Level

Problem 4. Let ABC be a scalene acute-angled triangle with its incenter I and circumcircle Γ.
Line AI intersects Γ for the second time at M . Let N be the midpoint of BC and T be the point
on Γ such that IN ⊥ M T . Finally, let P and Q be the intersection points of T B and T C,
respectively, with the line perpendicular to AI at I. Show that P B = CQ.
Proposed by Patrik Bak - Slovakia
------------------------------------------------------------------
Solution 1. Let S be the midpoint of arc BAC of Γ and let Q′ be the reflection of S in the per-
pendicular bisector of IC. We will prove that Q = Q′ , which would mean that SI = CQ.
Analogously, we would have SI = P B, and so the proof would be finished.

S
A

Q′
Γ

I
T

B N C

It is well-known that M is the circumcenter of BIC, so the perpendicular bisector of CI passes


through M . Since M S is a diameter of Γ, we have SC ⊥ M C, and by the symmetry subsequently
Q′ I ⊥ M I. Now, it is enough to show that Q′ , C, T are collinear.

A
Q′
Γ

I T

N
B C

M
Solutions 23

We have ∠IQ′ M = ∠M SC = ∠N CM . Notice also M N ⊥ N C. Together, triangles M IQ′ ,


M N C are directly similar, which gives that triangles M IN , M Q′ C are also directly similar.
Since IN ⊥ M T and ST ⊥ M T , we have ST ∥ IN , so ∠M CT = ∠M ST = ∠SN I. This
together with ∠IN M = ∠Q′ CM finally gives us that points T , C, and Q′ are collinear, which
concludes the proof.
Solution 2 (Proposed Solution from Slovakia). We define P and Q as the points on the line
perpendicular to AI at I such that △M BC and △M P Q are spirally similar. The equality M B =
M C then implies M P = M Q. Since N and I are the midpoints of BC and P Q, respectively,
spiral similarity gives that all triangles △M P B, △M IN and △M QC are similar. The similarity
gives ∠M P B = ∠M QC, which means that if we define T as the intersection point of lines CQ
and P B, then P, T, Q, M are concyclic, and so 180◦ − ∠CT B = ∠QT P = ∠QM P = ∠CM B,
therefore T lies on Γ. To finish our proof, we need to show that IN ⊥ M T :

∠M IN + ∠T M I = ∠M IN + ∠T M Q + ∠QM I
= ∠M P B + ∠T P Q + ∠IM P
= ∠M P I + ∠IM P = 90◦

A Q

I T

N
C
B
P

M
24 Intermediate Level

Problem 5. Consider a convex pentagon ABCDE and a variable point X on its side CD.
Suppose that points K, L lie on the segment AX such that AB = BK and AE = EL and that
the circumcircles of triangles CXK and DXL intersect for the second time at Y . As X varies,
prove that all such lines XY pass through a fixed point, or they are all parallel.
Proposed by Josef Tkadlec - Czech Republic
------------------------------------------------------------------
Solution. Let ωb , ωe be the circles centered at B, E and passing through A (note that K ∈ ωb
and L ∈ ωe ). By (XY Z) we denote the circumcircle of triangle XY Z. Let ∠(p, q) denote the
directed angle between lines p, q.
First we show that all the circles (DXL) pass through a fixed point D′ on ωe . Motivated by the
limiting case X = D, let L′ = AD ∩ ωe and let D′ be the intersection of ωe and the circle through
L′ and D tangent to CD. Then D′ is the fixed point: Indeed, for any X and the corresponding
L we have

∠(D′ L, D′ D) = ∠(D′ L, D′ L′ ) + ∠(D′ L′ , D′ D) = ∠(AL, AL′ ) + ∠(DL′ , DX)


= ∠(AX, AD) + ∠(DA, DX) = ∠(XA, XD) = ∠(XL, XD),

hence (XLD) passes through D′ . Similarly, all circles (CXK) pass through a fixed point C ′ on
ωb .

A ωe A ωe
ωb

B E C0 Y

K C
C0 Y D0
L0
L L X
C D
D0 D0

X X
D D

Now the problem can be conveniently rephrased with respect to a (fixed) quadrilateral C ′ CDD′ .
Note that the angle ∠(Y C ′ , Y D′ ) is fixed: Indeed,

∠(Y C ′ , Y D′ ) = ∠(Y C ′ , Y X) + ∠(Y X, Y D′ ) = ∠(CC ′ , CX) + ∠(DX, DD′ ) = ∠(CC ′ , DD′ ).

We conclude by distinguishing two cases.

1. Y ∈ C ′ D′ :
Since ∠(Y C ′ , Y X) = ∠(CC ′ , CX) is fixed, all lines XY form the same angle with line C ′ D′
and are thus parallel.

2. Y ̸∈ C ′ D′ :
Since ∠(Y C ′ , Y X) = ∠(CC ′ , CX) is fixed, lines XY all intersects (Y C ′ D′ ) at a fixed point.
Advanced Level

25
Problems

Problem 1. Acute-angled triangle ABC with circumcircle ω is given. Let D be the midpoint
of AC, E be the foot of altitude from A to BC, and F be the intersection point of AB and DE.
Point H lies on the arc BC of ω (the one that does not contain A) such that ∠BHE = ∠ABC.
Prove that ∠BHF = 90◦ .
(→ p.29)
Problem 2. Two circles Γ1 and Γ2 meet at two distinct points A and B. A line passing through
A meets Γ1 and Γ2 again at C and D respectively, such that A lies between C and D. The tangent
at A to Γ2 meets Γ1 again at E. Let F be a point on Γ2 such that F and A lie on different sides
of BD, and 2∠AF C = ∠ABC. Prove that the tangent at F to Γ2 , and lines BD and CE are
concurrent.
(→ p.30)
Problem 3. Consider a triangle ABC with altitudes AD, BE, and CF , and orthocenter H. Let
the perpendicular line from H to EF intersects EF , AB and AC at P , T and L, respectively.
Point K lies on the side BC such that BD = KC. Let ω be a circle that passes through H and
P , that is tangent to AH. Prove that circumcircle of triangle AT L and ω are tangent, and KH
passes through the tangency point.
(→ p.32)
Problem 4. 2021 points on the plane in the convex position, no three collinear and no four
concyclic, are given. Prove that there exist two of them such that every circle passing through
these two points contains at least 673 of the other points in its interior.
(A finite set of points on the plane are in convex position if the points are the vertices of a convex
polygon.)
(→ p.36)
Problem 5. Given a triangle ABC with incenter I. The incircle of triangle ABC is tangent to BC
at D. Let P and Q be points on the side BC such that ∠P AB = ∠BCA and ∠QAC = ∠ABC,
respectively. Let K and L be the incenter of triangles ABP and ACQ, respectively. Prove that
AD is the Euler line of triangle IKL.
(The Euler line of a triangle is the line going through the circumcenter and orthocenter of that
triangle.)
(→ p.37)

27
28 Advanced Level
Solutions

Problem 1. Acute-angled triangle ABC with circumcircle ω is given. Let D be the midpoint
of AC, E be the foot of altitude from A to BC, and F be the intersection point of AB and DE.
Point H lies on the arc BC of ω (the one that does not contain A) such that ∠BHE = ∠ABC.
Prove that ∠BHF = 90◦ .
Proposed by Harris Leung - Hong Kong
------------------------------------------------------------------
Solution. Note that DC = DE = DA since ∠CEA = 90◦ . Thus

∠EF A = ∠DEA − ∠F AE = (90◦ − ∠C) − (90◦ − ∠B) = ∠B − ∠C,

and
∠EHA = ∠BHE − ∠BHA = ∠B − ∠C = ∠EF A.
This implies A, E, H, F are concyclic. Therefore

∠BHF = ∠AHF − ∠AHB = ∠AEF − ∠C = (90◦ + ∠C) − ∠C = 90◦ .

A ω

B C
E

Remark.When AB > AC, F lies on the other side, so some of the arguments should be modified.
Though the result still holds.

29
30 Advanced Level

Problem 2. Two circles Γ1 and Γ2 meet at two distinct points A and B. A line passing through
A meets Γ1 and Γ2 again at C and D respectively, such that A lies between C and D. The tangent
at A to Γ2 meets Γ1 again at E. Let F be a point on Γ2 such that F and A lie on different sides
of BD, and 2∠AF C = ∠ABC. Prove that the tangent at F to Γ2 , and lines BD and CE are
concurrent.
Proposed by Tak Wing Ching - Hong Kong
------------------------------------------------------------------
Solution 1.

D
Γ1
A

Γ2
C

E
B
F

Clearly, the point F is uniquely determined. We redefine the point F as follows. Let BD meet
CE at P , and let F be the contact point of the tangent from P to Γ2 that lies on different side
of BD as A. It suffices to prove that ∠AF C = 21 ∠ABC. By (XY Z) we denote the circumcircle
of triangle XY Z.
First, since ∠ECB = ∠EAB = ∠ADB, the line CP is tangent to (CBD). Therefore

P C = P B · P D = P F.

Then

∠F BC = ∠BF P + ∠F P C + ∠P CB
= ∠BDF + (180◦ − 2∠CF P ) + ∠CDB
= ∠CDF + 180◦ − 2∠CF P.

(Note that ∠F BC may refer to a reflex angle in some configurations.) It follows that

∠ABC = ∠F BC − ∠F BA
= (∠CDF + 180◦ − 2∠CF P ) − (180◦ − ∠ADF )
= 2∠CDF − 2∠CF P
= 2∠AF P − 2∠CF P
= 2∠AF C,
Solutions 31

and this completes the proof.


Solution 2 (Proposed Solution from Czech republic).
We apply inversion centered at A.
inverted problem: Let C ′ B ′ D′ be a triangle. On sides C ′ B ′ and C ′ D′ are points E ′ and A
respectively, such that AE ′ ∥ D′ B ′ . Point F ′ lies on D′ B ′ such that 2∠F ′ C ′ A = ∠B ′ C ′ A. Denote
ω1 circle passing through A and F ′ that is touching D′ B ′ at F ′ . Denote ω2 circumcircle AB ′ D′
and ω3 circumcircle C ′ AE ′ . Prove that these three circles pass through fixed point different from
A.
From statement we get that C ′ F ′ is angle bisector of ∠B ′ C ′ D′ . Denote G′ intersection of external
bisector of ∠B ′ C ′ D′ with C ′ D′ . Denote Ω circle with diameter G′ F ′ . Denote ω4 circumcircle of
(C ′ , B ′ , D′ ). We will prove that Ω is perpendicular to all three circles ω1 , ω2 and ω3 . For ω1 it’s
immediate as they have perpendicular diameters. Then because (G, B ′ , F ′ , D′ ) is harmonic, we
have that ω2 ⊥ Ω and ω4 ⊥ Ω. And from homothethy we have that ω3 and ω4 are parallel at A.
Hence ω3 ⊥ Ω. Denote S center of Ω. From perpendicular circles we have that S has the same
power to all three circles ω1 , ω2 and ω3 . But so does point A. Hence these circles form a pencil,
hence they pass through another fixed point.

Remark. the point P cannot be defined if Γ1 passes through the centre of Γ2 , since in that
case CE and BD are parallel. Indeed, in that case the three lines are parallel to each other, so
the assertion of this question should be changed to ’concurrent or parallel’.
32 Advanced Level

Problem 3. Consider a triangle ABC with altitudes AD, BE, and CF , and orthocenter H. Let
the perpendicular line from H to EF intersects EF , AB and AC at P , T and L, respectively.
Point K lies on the side BC such that BD = KC. Let ω be a circle that passes through H and
P , that is tangent to AH. Prove that circumcircle of triangle AT L and ω are tangent, and KH
passes through the tangency point.
Proposed by Mahdi Etesamifard - Iran
------------------------------------------------------------------
Solution 1. Perform an inversion centered at H with radius −AH ·HD. The images of the points
are denoted by primes. It’s clear that F ′ ≡ C and E ′ ≡ B so P ′ lies on the circumcircle of triangle
HBC and ∠HCP ′ = 90◦ . Also T ′ and L′ lie on the line HP ′ such that ∠CT ′ H = ∠BL′ H = 90◦ ,
since circumcircle of triangles EHD and F HD passes through T ′ and L′ , respectively. Note that
ACP ′ B is a parallelogram, so by symmetry one can show that P ′ K ⊥ BC. Therefore P ′ K is the
image of ω under the inversion and it suffices to show that P ′ K touch the circumcircle of triangle
DT ′ L′ .
L

T E
P
ω F
H

N
L′

B K C
D M

T′

P′

Let M and N be the midpoints of BC and BH. Then

∠L′ DH = ∠L′ BH = ∠F EH = ∠F AH.

It implies that DL′ ∥ AB. Also M N ⊥ AB so M N must be the perpendicular bisector of DL′ ,
since N L′ = N D. This means M L′ = M D. Similarly, one can show that M T ′ = M D. Also it
is clear that M D = M K, therefore the quadrilateral DL′ KT ′ is inscribed in a circle centered at
M . Now since ∠M KP ′ = 90◦ the result follows.
Solutions 33

Solution 2 (Proposed Solution from Slovakia).


Tangency part. Consider the spiral similarity which maps LE to T F . Since A = LE ∩ T F , its
center S lies on the circumcircles of △AT L and △AEF . Moreever, S is also the center of the
spiral similarity mapping T L to EF . Since P = T L ∩ EF , S lies on the circumcircle of △P T F
. A simple angle chasing then gives:

∠AHS = ∠AF S = ∠T F S = ∠T P S

which means AH is tangent to the circumcircle of △SP H.


(Note that the proven tangency holds in a general situation: E and F can be arbitrary points
on AC and AB, P can be an arbitrary point on EF , and H can be an arbitrary point on the
circumcircle of △AEF .)

E
T
S P

C
B D

To prove the tangency of the circumcircles of ALT S and △SP H, we will show that the needed
condition is the equality of angles ∠ASH and ∠T P F , which in our situation is trivial (as they
are both right). The criterion for tangency is that ∠ASH equals to the sum of angle of arc (AS)
of the circumcircle of AT SL and the angle of arc (SH) of the circumcircle of △SP H. This sum
equals to:
∠ST F + ∠SP T = ∠ST F + ∠SF T = ∠T P F
Collinearity. First of all, we will redefine the tagency point S as the second intersection point
of KH and the circumcircle of AF HE. We will be finished when we prove ∠HSP = ∠AHP ,
which subsequently means that AH and the circumcircle of △SP H are tangent.
Let A′ be the point for which ABA′ C is a paralellogram. Then B, A′ , C, H are concyclic and lie
on the circle with a diameter A′ H.
Note that A′ , H, P are collinear. This is a simple angle chasing exercise, but can also be seen like
this: In triangle △HBC, HA′ is a line through the circumcenter, whereas in triangle △HEF
, HP is a line through the orthocenter. Since triangles △HBC and △HEF are isogonal, lines
HA′ and HP must be the same.
34 Advanced Level

E
S
P

F
H

K
B C
D

A′

Clearly A′ K ∥ AD, so we have ∠AHP = ∠DHA′ = ∠KA′ H = ∠KA′ P . To prove ∠AHP =


∠HSP , we just need to show that points S, P, K, A′ are concyclic. Notice A, S, D, K are concyclic
due to right angles ∠ASK and ∠ADK. Therefore, we need just to prove that A, P, D, A′ are
concyclic, then we would have

HS · HK = HA · HD = HP · HA′

whih would show that S, P, K, A′ are concyclic.


Notice we have got rid of point S. We now need to show that ∠AP A′ = ∠ADA′ . Since ∠EP H =
∠ADC = 90◦ , we only need to prove ∠AP E = ∠A′ DC. This will be easy: Triangles △HBC
and △HEF are similar and in this similarity, point D corresponds to P ,which gives BD : CD =
F P : P E . Also, triangles △AEF and △A′ CB are similar, and so in this similarity, points P
and D correspond, which gives ∠AP E = ∠A′ DC, which was needed to show.
Solutions 35

P
F
H

C
B
D

A′
36 Advanced Level

Problem 4. 2021 points on the plane in the convex position, no three collinear and no four
concyclic, are given. Prove that there exist two of them such that every circle passing through
these two points contains at least 673 of the other points in its interior.
(A finite set of points on the plane are in convex position if the points are the vertices of a convex
polygon.)
Proposed by Morteza Saghafian - Iran
------------------------------------------------------------------
Solution. Call the points P1 , P2 , . . . , P2021 . We need two lemmas for the statement.
Lemma 1. Call a triangle good if its circumcircle covers all the other points. All the good
triangles form a triangulation of the 2021-gon P1 P2 . . . P2021 .
Proof. Note that on every side of this polygon we can construct exactly one good triangle by
simply selecting the smallest angle formed by some other vertex. Now start from a good triangle
say Pi Pj Pk and pick one of its sides say Pi Pj . Note that all the vertices on the same side of Pi Pj
as Pk subtend a larger angle with Pi and Pj as the endpoints, while all the vertices X on the
other side fulfill ∠Pi XPj + ∠Pi Pk Pj > 180◦ . Taking point Pl so that ∠Pi Pl Pj is minimal and
Pl , Pk are on the different sides of Pi Pj will also create Pi Pl Pj as a good triangle, because:

• For all points X on the same side of Pi Pj as Pl , ∠Pi XPj is larger than or equal to ∠Pi Pl Pj .

• Among all of angles ∠Pi XPj so that X and Pl are on different sides of Pi Pj , ∠Pi Pk Pj is
the smallest angle and hence ∠Pi XPj + ∠Pi Pl Pj > 180◦ .

Continuing this process we reach a triangulation of the 2021-gon and call it T . Also note that any
triangle from the triangulation uniquely determines the rest. Suppose now there exists a triangle
not belonging to the previous triangulation but still being good. Go ahead and apply the same
procedure as in the previous case and suppose we reach a triangulation T ′ . Fix a certain side of
the 2021-gon say the side P1 P2 . Note that this side is part of only one good triangle and hence
T, T ′ share a triangle however then we have T = T ′ and hence we are done.

Lemma 2. In the above triangulation T of P1 P2 . . . P2021 , there exists a drawn diagonal with at
least 673 points on each side.
Proof. Consider a regular 2021-gon Q1 Q2 . . . Q2021 and draw the diagonals between Qi s of similar
indices as in T . Obviously, we get a triangulation T ′ of Q1 Q2 . . . Q2021 . Now let O, the circum-
center of Q1 Q2 . . . Q2021 , be in the one of the triangles in T ′ . Then this triangle, say Qi Qj Qk is an
acute-angled triangle (because its circumcenter lies in its interior). Let Qi Qj be its longest side.
So the angle ∠Qi Qk Qj is acute and at least 60◦ and this means there are at least 673 of other
Ql s on each side of Qi Qj .
Now moving back to P1 P2 . . . P2021 , the diagonal Pi Pj has the desired property.
Finally notice that the two points Pi , Pj in Lemma 2 satisfy the statement of the problem since
every circle passing through these two points covers all the points on either one side of Pi Pj .
Solutions 37

Problem 5. Given a triangle ABC with incenter I. The incircle of triangle ABC is tangent to BC
at D. Let P and Q be points on the side BC such that ∠P AB = ∠BCA and ∠QAC = ∠ABC,
respectively. Let K and L be the incenter of triangles ABP and ACQ, respectively. Prove that
AD is the Euler line of triangle IKL.
(The Euler line of a triangle is the line going through the circumcenter and orthocenter of that
triangle.)
Proposed by Le Viet An - Vietnam
------------------------------------------------------------------
Solution. Let Γ, O and H be the circumcircle, circumcenter and orthocenter of triangle IKL,
respectively.
Claim 1. A lies on OH.
Proof. Suppose that Γ meets HK, HL, OK, and OL at M , N , K ′ , and L′ , respectively. Also
M L′ intersects N K ′ at A′ . Applying Pascal’s theorem to the hexagon M KK ′ N LL′ implies that
the points H, O, and A′ are collinear.
It is easy to see that ∠AKI = ∠ABK+∠BAK
2
= ∠B+∠C
2
and
∠B + ∠C
∠IKN = ∠ILH = 90◦ − (180◦ − ∠BIC) = 90◦ − .
2
Hence ∠AKN = ∠AKI + ∠IKN = 90◦ . It yields that AK ∥ A′ K ′ . Similarly, one can show that
AL ∥ A′ L′ . Combining with KL ∥ K ′ L′ (by symmetry about center O), follows that two triangles
AKL and A′ K ′ L′ are homothetic. Therefore AA′ , KK ′ , and LL′ are concurrent and this proves
the claim.
H

I
L
K

B C
P Q
O
N
M

K′

L

A′

Claim 2. D lies on OH.


38 Advanced Level

Proof. Let HK and HL intersect BC at E and F . Point A′ is the symmetric point to A with
respect to BI. Then
1 1
∠KA′ B = ∠KAB = ∠BAP = ∠C = ∠ICB.
2 2
It follows that KA′ ∥ CI, so ∠A′ KE = 90◦ . Now notice that ∠A′ IK = ∠AIK = 90◦ + ∠C 2
. On the
other hand, note that KE ⊥ CI so ∠A′ EK = 90◦ −∠ICE = 90◦ − ∠C 2
. Hence ∠A ′
IK +∠A ′
EK =
◦ ′
180 that implies quadrilateral A IKE is inscribed.
Let X be the midpoint of A′ E. Since ∠A′ IE = 90◦ , X is the circumcenter of triangle EKI. It
yields that

∠IXD = ∠IXA′ = 2∠IKA′ = 2∠IKA = 2(∠KAB + ∠KBA) = ∠C + ∠B.

Similarly, if Y be the circumcenter of triangle ILF , one can show that Y lies on BC and ∠IY D =
∠B + ∠C. So the triangle IXY is isosceles at I. It yields that XE = XI = Y I = Y F . From
the other hand, ID ⊥ XY that implies D is the midpoint of XY and EF . Finally note that OX
and OY are perpendicular bisectors of IK and IL, so OX ∥ HF and OY ∥ HE. It implies that
triangles OXY and HF E are similar. Now note that DO and DH are medians of this triangles
so ∠HDE = ∠ODY and the conclusion follows.
H

I
L
K

A′
B E P X D Y Q F C

These two claims together yields the desired result.


9th Iranian Geometry Olympiad
October 14, 2022

Contest problems with solutions


(Draft Version)

Note That this is the first version of the booklet prepared for the use of local
organizers of IGO. It should be kept confidential until the final version is posted
on the official website of IGO.
Contents
Elementary Level 3
Problems . . . . . . . . . . . . . . . . . . . . . . . . . . . . . . . . . . . . . . . . . . . . . . . . . . . . . . . . . . . . . . . . . . . . . . 3

Solutions . . . . . . . . . . . . . . . . . . . . . . . . . . . . . . . . . . . . . . . . . . . . . . . . . . . . . . . . . . . . . . . . . . . . . . 5

Intermediate Level 13
Problems . . . . . . . . . . . . . . . . . . . . . . . . . . . . . . . . . . . . . . . . . . . . . . . . . . . . . . . . . . . . . . . . . . . . . . 13

Solutions . . . . . . . . . . . . . . . . . . . . . . . . . . . . . . . . . . . . . . . . . . . . . . . . . . . . . . . . . . . . . . . . . . . . . . 15

Advanced Level 23
Problems . . . . . . . . . . . . . . . . . . . . . . . . . . . . . . . . . . . . . . . . . . . . . . . . . . . . . . . . . . . . . . . . . . . . . . 23

Solutions . . . . . . . . . . . . . . . . . . . . . . . . . . . . . . . . . . . . . . . . . . . . . . . . . . . . . . . . . . . . . . . . . . . . . . 25
Elementary Level

1
Problems

Problem 1. Find the angles of the pentagon ABCDE in the figure below.

B D

X Y

Z
A E

(→ p.5)
Problem 2. An isosceles trapezoid ABCD (AB ∥ CD) is given. Points E and F lie on the
sides BC and AD, and the points M and N lie on the segment EF such that DF = BE and
F M = N E. Let K and L be the foot of perpendicular lines from M and N to AB and CD,
respectively. Prove that EKF L is a parallelogram.
(→ p.6)
Problem 3. Let ABCDE be a convex pentagon such that AB = BC = CD and ∠BDE =
∠EAC = 30◦ . Find the possible values of ∠BEC.
(→ p.7)
Problem 4. Let AD be the internal angle bisector of triangle ABC. The incircles of triangles
ABC and ACD touch each other externally. Prove that ∠ABC > 120◦ . (Recall that the incircle
of a triangle is a circle inside the triangle that is tangent to its three sides.)
(→ p.8)
Problem 5. a) Do there exist four equilateral triangles in the plane such that each two have
exactly one vertex in common, and every point in the plane lies on the boundary of at most two
of them?
b) Do there exist four squares in the plane such that each two have exactly one vertex in common,
and every point in the plane lies on the boundary of at most two of them?
(Note that in both parts, there is no assumption on the intersection of interior of polygons.)
(→ p.9)

3
4 Elementary Level
Solutions

Problem 1. Find the angles of the pentagon ABCDE in the figure below.
C

B D

X Y

Z
A E

Proposed by Morteza Saghafian - Iran


------------------------------------------------------------------
Solution.
C

B D

X
Y

A E

It’s easy to see that the triangles △CBX, △CXY, △CY D, △ABX, △AXZ, △EDY, △EY Z are
all congruent. So 360◦ = ∠CXY + ∠CXB + ∠BXA + ∠AXZ + ∠Y XZ = 4∠CXY + 60◦ .
So ∠CXY = 75. So ∠ABC = ∠CDE = 150◦ and ∠BCD = 90◦ . So ∠BAE + ∠DEA =
540◦ − ∠ABC − ∠BCD − ∠CDE = 150◦ .Because ∠BAE and ∠DEA are equal, then they are
each 75◦

5
6 Elementary Level

Problem 2. An isosceles trapezoid ABCD (AB ∥ CD) is given. Points E and F lie on the
sides BC and AD, and the points M and N lie on the segment EF such that DF = BE and
F M = N E. Let K and L be the foot of perpendicular lines from M and N to AB and CD,
respectively. Prove that EKF L is a parallelogram.
Proposed by Mahdi Etesamifard - Iran
------------------------------------------------------------------
Solution. Let the line at F parallel to BC intersect AB, CD in P and Q respectively.
Since BP ∥QC and P Q∥BC the quadrilateral P BCQ is a parallelogram.
We have F P = AF because ∡F P A = ∡F AP , hence F P = AF = EC
Letting O be the midpoint of M N :

OF = OE, F P = EC, ∡P F O = ∡CEO

in fact P F O ∼
= CEO. Now excluding the points A and D the figure is completely symmetric
with respect to O hence F K = EL, F K∥EL, therefore F KEL is a parallelogram.
P A K B

E
N
O

M
F

D Q L C
Solutions 7

Problem 3. Let ABCDE be a convex pentagon such that AB = BC = CD and ∠BDE =


∠EAC = 30◦ . Find the possible values of ∠BEC.
Proposed by Josef Tkadlec - Czech Republic
------------------------------------------------------------------
Solution. Answer: 60◦ .

B C

A 30 30 D
60

E′

Fix A, B, C, D such that AB = BC = CD and ABCD is convex. Note that the point E is
uniquely determined by the angle conditions.
Let E ′ be a point such that BCE ′ is equilateral (and A, E ′ , D all lie on the same side of BC).
By inscribed angles we have ∠BDE ′ = 21 ∠BCE ′ = 30◦ and likewise ∠E ′ AC = 30◦ . Hence
E ′ ≡ E and ∠BEC = ∠BE ′ C = 60◦ .
8 Elementary Level

Problem 4. Let AD be the internal angle bisector of triangle ABC. The incircles of triangles
ABC and ACD touch each other externally. Prove that ∠ABC > 120◦ . (Recall that the incircle
of a triangle is a circle inside the triangle that is tangent to its three sides.)
Proposed by Volodymyr Brayman - Ukraine
------------------------------------------------------------------
Solution. Denote by ω(I, r) and ω1 (I1 , r1 ) the incircles of triangles ABC and ACD, respectively.
Let T be the point where ω1 touches AD. Point I1 lies on the segment CI, so the distance from
I1 to BC is less than the distance from I to BC. Hence r1 < r. Consider right triangle II1 T
(∠IT I1 = 90◦ ). Since II1 = r + r1 > 2r1 = 2I1 T , we have ∠T I1 I > 60◦ . On the other hand,
∠T I1 I + 90◦ = ∠AIC = 12 ∠ABC + 90◦ . Thus 12 ∠ABC > 60◦ , or ∠ABC > 120◦ .

ω1
I
I1

T
B D C

Remark. It can be shown that triangle ABC satisfies condition of the problem if and only if
2 cos B2 = 1 − sin C2 .
Solutions 9

Problem 5. a) Do there exist four equilateral triangles in the plane such that each two have
exactly one vertex in common, and every point in the plane lies on the boundary of at most two
of them?
b) Do there exist four squares in the plane such that each two have exactly one vertex in common,
and every point in the plane lies on the boundary of at most two of them?
(Note that in both parts, there is no assumption on the intersection of interior of polygons.)
Proposed by Hesam Rajabzadeh - Iran
------------------------------------------------------------------
Solution. a) The answer is no. We begin with a lemma.
Lemma. Given two segments AB, CD of equal length, there is at most one point Z such that the
(clockwise or counter-clockwise) rotation of angle 60◦ with center Z maps AB to CD.
Proof. Consider Cartesian coordinate system on the plane and suppose that in this coordinate
−→
system the vector AB lies on x-axis and points to the right. Suppose that there is Z such that
the rotation of 60◦ with center Z maps exactly A to C and B to D (by symmetry, other cases
−→ −−→ −−→
are similar). Therefore, this rotation maps vector AB to CD. In particular, CD makes angle
60◦ with the positive direction of x-axis. We claim that such point Z is unique. Note that both
triangles ZAC, ZBD are equilateral, so Z lies on the perpendicular bisectors of AC, BD. This
intersection point is unique unless AC ∥ BD. In this case, Z is the intersection point of lines
AB, CD.
Now, if there is another point Z ′ such that the rotation of 60◦ with center Z ′ maps exactly A to
−−→
D and B to C, by similar arguments DC makes angle 60◦ with the positive direction of x-axis
−−→
which is a contradiction with the angle between CD and the positive direction of x-axis. The
proof of lemma is complete.
We return to the main problem. Assume to the contrary that there exist four equilateral triangles
△1 , △2 , △3 , △4 satisfying the conditions. The conditions imply that there are six points in the
plane; each is the vertex of exactly two of △i ’s.
First suppose that △i ’s are mutually non-congruent. Suppose that △1 = XAC and △2 = XBD,
and the labels of the vertices are so that the counter-clockwise rotation of 60◦ with center X
−→ −−→
maps A to C and B to D, respectively. So this rotation will map vector AB to CD (in particular
AB = CD).
Next, denote the common vertex of △3 , △4 by Y . We have several cases.

• △3 = Y AB, △4 = Y CD. In this case, we have

Y B = Y A = AB = CD = Y C = Y D.

So △3 ≡ △4 , contradicting our assumption.

• The remaining two triangles are △3 = Y AD, △4 = Y BC. In this case, either △Y BD ≡
△Y AC or △Y CD ≡ △Y BA. In the former case, we have AC = BD and so △1 ≡ △2 .
This leads to contradiction with the assumption of triangles not being congruent. In the
latter case, we get a rotation of angle 60◦ with center Y maps AB to CD. Note that the
point X has the same property so in view of lemma we must have X = Y . This leads to a
contradiction with the assumptions as X is a common vertex of all triangles.

Now suppose that two equilateral triangles △1 = XAC, △2 = XBD are congruent. This implies
that points A, B, C, D lie on the same circle with center X. So in particular, X lies on the
perpendicular bisectors of any pair from {A, B, C, D}. Assume that the labels of the points are
chosen so that the counter-clockwise rotation of 60◦ with center X maps A to C and B to D,
10 Elementary Level

respectively. By arguments similar to what we have done before, the angle between lines AB, CD
is 60◦ .
Denote the common vertex of the remaining two triangles by Y . We have several cases.

• The remaining two triangles are △3 = Y AB, △4 = Y CD. So, Y lies on the perpendicular
bisector of AB, CD. Since AB is not parallel to CD, their perpendicular bisectors meet at
the unique point X and so X = Y . This leads to contradiction as X = Y became a vertex
of all four triangles.

• The remaining two triangles are △3 = Y AD, △4 = Y BC. So, Y lies on the perpendicular
bisector of AD, BC. Now, if AD ̸∥ BC, similar to the previous item, we get X = Y ,
contradiction. But if AD ∥ BC, since ∠BY C = ∠DY A = 60◦ , Y should be the intersection
point of AC, BD and A, X, D are collinear. This also leads to a contradiction, as point X
is on the boundary of three triangles XBD, XAC, AY D.

So the proof is complete.


b) The answer is yes. First consider three squares Q1 , Q2 , Q3 , as figure below.

A D Q3 Q3
O

Q1 Q1
Q4

B Q2 Q2

Note, if we denote the vertices of the squares by A, B, C, D (as in the left figure above) and the
center of Q3 by O, then right-angled triangles ABD and AOC are congruent. So, AC = AB and

∠CAB = ∠CAO + ∠DAB = 90◦ .

Therefore, A, C, B are three vertices of a square, say Q4 . Then, Q1 , Q2 , Q3 , Q4 satisfy all required
conditions (see figure right above).
Intermediate Level

11
Problems

Problem 1. In the figure below we have AX = BY . Prove that ∠XDA = ∠CDY .

A X Y B

(→ p.15)
Problem 2. Two circles ω1 and ω2 with equal radius intersect at two points E and X. Arbitrary
points C, D lies on ω1 , ω2 . Parallel lines to XC, XD from E intersect ω2 , ω1 at A, B, respectively.
Suppose that CD intersect ω1 , ω2 again at P, Q, respectively. Prove that ABP Q is concyclic.
(→ p.16)
Problem 3. Let O be the circumcenter of triangle ABC. Arbitrary points M and N lie on
the sides AC and BC, respectively. Points P and Q lie in the same half-plane as point C with
respect to the line M N , and satisfy △CM N ∼ △P AN ∼ △QM B (in this exact order). Prove
that OP = OQ.
(→ p.17)
Problem 4. We call two simple polygons P , Q compatible if there exists a positive integer k such
that each of P, Q can be partitioned into k congruent polygons similar to the other one. Prove
that for every two even integers m, n ≥ 4, there are two compatible polygons with m and n sides.
(A simple polygon is a polygon that does not intersect itself.)
(→ p.18)
Problem 5. Let ABCD be a quadrilateral inscribed in a circle ω with center O. Let P be the
intersection of two diagonals AC and BD. Let Q be a point lying on the segment OP . Let E
and F be the orthogonal projections of Q on the lines AD and BC, respectively. The points M
and N lie on the circumcircle of triangle QEF such that QM ∥ AC and QN ∥ BD. Prove that
the two lines M E and N F meet on the perpendicular bisector of segment CD.
(→ p.19)

13
14 Intermediate Level
Solutions

Problem 1. In the figure below we have AX = BY . Prove that ∠XDA = ∠CDY .

A X Y B

Proposed by Iman Maghsoudi - Iran


------------------------------------------------------------------
Solution. Let O be the circumcenter of ω. Since OY ⊥ AB, OD ⊥ AD, we obtain that AY OD
is a square, therefore OY = AD, ∠OY B = ∠DAX = 90◦ , it implies that triangles DAX, OY B
are congruent. Hence ∠XDA = ∠BOY = ∠CDY .

A X Y B

D O

15
16 Intermediate Level

Problem 2. Two circles ω1 and ω2 with equal radius intersect at two points E and X. Arbitrary
points C, D lies on ω1 , ω2 . Parallel lines to XC, XD from E intersect ω2 , ω1 at A, B, respectively.
Suppose that CD intersect ω1 , ω2 again at P, Q, respectively. Prove that ABP Q is concyclic.
Proposed by Ali Zamani - Iran
- - - - - - - - - - - - - - - - - - - - - - - - - - - - - - - - - - - - - - - - - - - - - - - - - - - - - ->
------------
EX
Solution. Since two circles are equal and EA ∥ XC, we have ∠ECX = 2 = ∠EAX =
180◦ − ∠AXC, it shows that AX ∥ EC and AXCE is parallelogram. Similarly BEDX is
parallelogram. From these we get AE = XC, BE = XD, ∠AEB = ∠CXD, then two triangles
EAB, XCD are congruent and AB ∥ CD.
On the other hand

∠AQP = ∠AQD = ∠AQE + ∠EQX + ∠XQD


= ∠XP C + ∠EP X + ∠BP E
= ∠BP C = ∠BP Q

Hence ABP Q is isosceles trapezoid and we are done.

B A

ω1 ω2
E

Q
P C D
Solutions 17

Problem 3. Let O be the circumcenter of triangle ABC. Arbitrary points M and N lie on
the sides AC and BC, respectively. Points P and Q lie in the same half-plane as point C with
respect to the line M N , and satisfy △CM N ∼ △P AN ∼ △QM B (in this exact order). Prove
that OP = OQ.
Proposed by Medeubek Kungozhin - Kazakhstan
------------------------------------------------------------------
Solution. From the statement we get quadrilaterals AN CP and BM CQ are concyclic, so

∠ACP = ∠AN P = 180◦ − ∠AP N − ∠P AN


= 180◦ − ∠ACB − ∠BM Q
= 180◦ − ∠ACB − ∠BCQ

It implies that the points P, C, Q are collinear.


Let M N meet circumcircles of triangles AN C, BM C again at E, F . We have

∠AN P = ∠EN C, ∠CM F = ∠BM Q

O P

B C
N

F Q

It shows that quadrilaterals AEP C, BQF C are isosceles trapezoids, then O lies on perpendicular
bisectors of EP, F Q(since O lies on perpendicular bisectors of AC, BC). Also we have

∠N EP = ∠N CQ = 180 − ∠CQF = 180 − ∠P QF

Therefore the quadrilateral EP QF is inscribed in a circle centered at O and this completes the
proof.
18 Intermediate Level

Problem 4. We call two simple polygons P , Q compatible if there exists a positive integer k such
that each of P, Q can be partitioned into k congruent polygons similar to the other one. Prove
that for every two even integers m, n ≥ 4, there are two compatible polygons with m and n sides.
(A simple polygon is a polygon that does not intersect itself.)
Proposed by Hesam Rajabzadeh - Iran
------------------------------------------------------------------
Solution. We begin the solution with a lemma.

Lemma. Compatibility is an equivalence relation.

Proof. We only need to check transitivity, i.e. whenever a polygon Q is compatible to both
polygons P and R, then P is compatible with R. The other properties are trivial from the
definition.
Clearly, if P can be partitioned into m congruent polygons Q1 , . . . , Qm similar to Q, and Q can
be partitioned into n congruent polygons R1 , . . . , Rn similar to R, then one can partition each of
Qi ’s into n congruent polygons Ri,1 , . . . , Ri,n similar to R. Note that every pair of polygons of
the from Ri,j are congruent (because they have equal area), so {Ri,j : 1 ≤ i ≤ m, 1 ≤ j ≤ n} is a
partition of P into mn congruent polygons similar to R. Similarly, one can show that R can be
partitioned into mn congruent polygons similar to P and the proof is complete.
In view of above lemma, it suffices to introduce a polygon P such that for every even number d
there is simple d-gon compatible to P . We claim that one can take P to be a square.
For every k ≥ 1, consider the following staircase (2k + 2)-gon with two sides of length 2k and 2k
sides of length 2 and denote it by Pk .

2k

2
2k

Obviously, Pk can be partitioned into 4 k(k+1)


2
= 2k(k + 1) unit squares. On the other hand, one
can partition a rectangle with side lengths 2k, 2k + 2 into two polygons congruent to Pk , and a
square with side length 2k(k + 1) can be partitioned into k(k + 1) of such rectangles. Putting
these together, a square can be partitioned into 2k(k + 1) polygons congruent to Pk . This shows
that for every k ≥ 2, Pk is compatible with the unit square and so by the lemma, for every pair
k, k ′ ∈ N of even numbers, Pk and Pk′ are compatible and the proof is complete.
Solutions 19

Problem 5. Let ABCD be a quadrilateral inscribed in a circle ω with center O. Let P be the
intersection of two diagonals AC and BD. Let Q be a point lying on the segment OP . Let E
and F be the orthogonal projections of Q on the lines AD and BC, respectively. The points M
and N lie on the circumcircle of triangle QEF such that QM ∥ AC and QN ∥ BD. Prove that
the two lines M E and N F meet on the perpendicular bisector of segment CD.
Proposed by Tran Quang Hung - Vietnam
------------------------------------------------------------------
Solution. We reformulate the problem into a triangle version as follows
Problem. Let ABC be a triangle. The circle (K) passing through B, C cuts the segments
CA, AB again at E, F , respectively. BE meets CF at H. Let P be a point lying on the segment
KH. The circle with diameter P A meets CA, AB again at M, N , respectively. Points S, T lie on
the circle with diameter P A such that P S ∥ BE, P T ∥ CF . Prove that M S and N T intersect
on the perpendicular bisector of BC.

A
G

E S
T F L
H M
N
P

B C

Two circumcircles of triangles AEF and ABC meet again at G. Easily seen AG ⊥ GK (from the
property of Miquel point) so G lies on the circle with diameter AP . Let GK meet the circumcircle
of triangle AEF again at L, then AL is the diameter of circumcircle of triangle AEF , therefore
AL ⊥ BC. Let M S meet N T at Q, we need to prove that KQ ⊥ BC. Indeed, we see that
P S ∥ BE and P M ⊥ AC, angles chasing give us

∠N M Q = ∠N M P + ∠M SP + ∠M P S
= 180◦ − ∠M P N + 90◦ − ∠BEC
= ∠BAC − ∠BEC + 90◦
= 90◦ − ∠EBA
= ∠F EK.
20 Intermediate Level

Similarly, ∠EF K = ∠M N Q. From these, triangles QM N and KEF are isosceles. Hence
△QM N ∼ △KEF . We have △GEF ∼ △GM N (since G is the center of spiral similarity which
transforms M N 7→ EF ), so △GF K ∼ △GN Q, this implies that △GF N ∼ △GKQ. Hence, we
have angle chasing again

∠GKQ = ∠GF N = 180◦ − ∠GF A = 180◦ − ∠GLA = ∠ALP.

This leads to KQ ∥ AL. We are done.


Advanced Level

21
Problems

Problem 1. Four points A, B, C, and D lie on a circle ω such that AB = BC = CD. The
tangent line to ω at point C intersects the tangent line to ω at point A and the line AD at points
K and L. The circle ω and the circumcircle of triangle KLA intersect again at M . Prove that
MA = ML
(→ p.25)
Problem 2. We are given an acute triangle ABC with AB ̸= AC. Let D be a point on BC
such that DA is tangent to the circumcircle of triangle ABC. Let E and F be the circumcenters
of triangles ABD and ACD, respectively, and let M be the midpoint of EF . Prove that the line
tangent to the circumcircle of AM D through D is also tangent to the circumcircle of ABC.
(→ p.26)
Problem 3. In triangle ABC (∠A ̸= 90◦ ), let O, H be the circumcenter and the foot of the
altitude from A respectively. Suppose M, N are midpoints of BC, AH respectively. Let D be
the intersection of AO and BC and let H ′ be the reflection of H about M . Suppose that the
circumcircle of OH ′ D intersects the circumcircle of BOC at E. Prove that N O and AE are
concurrent on the circumcircle of BOC.
(→ p.28)
Problem 4. Let ABCD be a trapezoid with AB ∥ CD. Its diagonals intersect at a point P . The
line passing through P parallel to AB intersects AD and BC at Q and R, respectively. Exterior
angle bisectors of angles DBA, DCA intersect at X. Let S be the foot of X onto BC. Prove
that if quadrilaterals ABP Q, CDQP are circumscribed, then P R = P S.
(→ p.31)
Problem 5. Let ABC be an acute triangle inscribed in a circle ω with center O. Points E, F
lie on its sides AC, AB, respectively, such that O lies on EF and BCEF is cyclic. Let R, S be
the intersections of EF with the shorter arcs AB, AC of ω, respectively. Suppose K, L are the
reflection of R about C and the reflection of S about B, respectively. Suppose that points P and
Q lie on the lines BS and RC, respectively, such that P K and QL are perpendicular to BC.
Prove that the circle with center P and radius P K is tangent to the circumcircle of RCE if and
only if the circle with center Q and radius QL is tangent to the circumcircle of BF S.
(→ p.33)

23
24 Advanced Level
Solutions

Problem 1. Four points A, B, C, and D lie on a circle ω such that AB = BC = CD. The
tangent line to ω at point C intersects the tangent line to ω at point A and the line AD at points
K and L. The circle ω and the circumcircle of triangle KLA intersect again at M . Prove that
MA = ML
Proposed by Mahdi Etesamifard - Iran
------------------------------------------------------------------
Solution. Observe that BC∥AD, an easy angle chase gives :
> >
BC AB
∡KM A = ∡KLA = ∡KCB = = = ∡BM A
2 2
thus points K, B and M are collinear. But the two triangles KBC and KBA are congruent
hence ∡AKB = ∡CKB which implies that M A = M L as desired.

B C

A L
D

25
26 Advanced Level

Problem 2. We are given an acute triangle ABC with AB ̸= AC. Let D be a point on BC
such that DA is tangent to the circumcircle of triangle ABC. Let E and F be the circumcenters
of triangles ABD and ACD, respectively, and let M be the midpoint of EF . Prove that the line
tangent to the circumcircle of AM D through D is also tangent to the circumcircle of ABC.
Proposed by Patrik Bak - Slovakia
------------------------------------------------------------------
Solution. Without loss of generality assume AB < AC. This means that B lies between D and
C. Triangle DBA is obtuse with the obtuse angle at B. Therefore, E lies in the half-plane DBC
containing A. Similarly, angle DAC, equal to 180◦ − ∠CBA, is obtuse, therefore F lies in the
opposite half-plane. Simple angle chasing gives
1
∠CDE = ∠BDE = 90◦ − ∠DEB = 90◦ − ∠DAB = 90◦ − ∠ACB
2
and
1
∠F DC = ∠F DB = 90◦ − ∠DF C
2
= 90◦ − (180◦ − ∠DAC)
= ∠DAC − 90◦
= ∠DAB + ∠BAC − 90◦
= ∠ACB + ∠BAC − 90◦
= 90◦ − ∠CBA

These two equalities show that DE ⊥ AC and DF ⊥ AB, respectively.

E
A

O
D
M
B C

Denote by O the circumcenter of triangle ABC. Since F O is the perpendicular bisector of AC,
we have F O ⊥ AC, therefore DE ∥ F O. Analogously, EO is the perpendicular bisector of AB,
Solutions 27

and so DF ∥ EO. Together, we have that DF OE is a parallelogram, therefore M is the midpoint


of DO. Since DA ⊥ AO, we have M D = M O = M A.
Let P be the reflection of A in line DM O. Clearly, P lies on the circumcircle of ABC. Since
DA ⊥ AO, then OP ⊥ DP , and so line DP is tangent to the circumcircle of ABC. Finally,
simple angle chasing shows that ∠P DM = ∠M DA = ∠DAM , therefore DP is also tangent to
the circumcircle of AM D, which concludes the proof.
28 Advanced Level

Problem 3. In triangle ABC (∠A ̸= 90◦ ), let O, H be the circumcenter and the foot of the
altitude from A respectively. Suppose M, N are midpoints of BC, AH respectively. Let D be
the intersection of AO and BC and let H ′ be the reflection of H about M . Suppose that the
circumcircle of OH ′ D intersects the circumcircle of BOC at E. Prove that N O and AE are
concurrent on the circumcircle of BOC.
Proposed by Mehran Talaei - Iran
------------------------------------------------------------------
Solution 1. Let the line OH intersect the circumcircle BOC in F
Claim 1. F, D, E are collinear
Proof. Since OM ⊥ BC and M H = M H ′ we have ∡OHH ′ = ∡OH ′ H. But OH ′ ED is cyclic
and ∡OH ′ D = ∡OED. The claim is easily proven considering the fact that OB = OC
Now let A′ be the A − antipode in the circumcircle ABC.It’s easy to see that
OH.OF = OB 2 = OC 2 = OA2
since F H is the bisector of ∡BF C, so OA is tangent to the circumcircle AHF , thus
∡HAO = ∡HF A or
∡A′ AF = ∡OAF = ∡OHA (1)
D lies on the radical axis of the two circumcircles ABC and BOC :
DF.DE = DB.DC = DA′ .DA (2)
by (2) the quadrilateral AEA′ F is cyclic :
∡A′ EF = ∡A′ AF (3)
letting OM intersect the circumcircle BOC in P we have OP ∥AH
∡P OF = ∡OHA (4)
Combining (1) , (3) , (4) points E, A′ , P are collinear
A

O
E

B C
H M DH ′

A′

P
Solutions 29

Let AO intersect the circumcircle BOC for the second time in K

Claim 2. (AA′ , DK) = −1

Proof. We have ∡P BC = ∡P CB = ∡A, P B and P C are tangent to the circumcircle ABC.


Hence the line BC is the polar of P wrt the circumcircle ABC and by La Hire’s Theorem the
polar of D wrt the circumcircle ABC passes through P . Since ∡P KO = 90, the line P K is this
polar and the claim is proved!

Now projecting from O onto the line AH we have:

−1 = (∞N, AH) = (P X, KF )

where X is the intersection point of N O and the circumcircle BOC. Taking a look at claim (2)
and projecting from E
−1 = (AA′ , DK) = (Y P, F K)

Where Y is the intersection point of AE and the circumcircle BOC Considering the last two
projections X ≡ Y .
Solution 2. Let OD and OH ′ intersect the circumcircle BOC in K and L respectively.
Easy to see that ∡OKL = ∡OH ′ D = ∡OHD.
Define P = KL ∩ BC. Then the quadrilateral HP OK is cyclic.

E
B M D H′ C P
H

Apply the inversion about the circumcircle ABC. The following problem is the figure after
mapping :

Problem 1. In triangle ABC the line AO intersects side BC and circumcircle BOC in D, K
respectively. H is the foot of the A − altitude. The line BC intersects the circumcircle OHK in
P and Q is the second intersection of this line with the circumcircle AOP . Prove that OQ bisects
AH.
30 Advanced Level

B P
H D Q C

A′

Proof.
Let A′ be the A − antipode in the circumcircle ABC.
D is the radical center of the three circumcircles OHK, ABC, BOC, hence

DH.DP = DO.DK = DA′ .DA

implying that the quadrilateral AHA′ P is cyclic

∡HA′ A = ∡HP A = ∡QOD

therefore the lines HA′ and OQ are parallel.


But O is the midpoint of AA′ thus QO bisects AH as desired !
Solutions 31

Problem 4. Let ABCD be a trapezoid with AB ∥ CD. Its diagonals intersect at a point P . The
line passing through P parallel to AB intersects AD and BC at Q and R, respectively. Exterior
angle bisectors of angles DBA, DCA intersect at X. Let S be the foot of X onto BC. Prove
that if quadrilaterals ABP Q, CDQP are circumscribed, then P R = P S.
Proposed by Dominik Burek - Poland
------------------------------------------------------------------
Solution. We start with the following simple lemma.
Lemma 1. Let KLM N be a circumscribed trapezoid with KL ∥ M N . Suppose that rays
−−→ −−→
KN , LM , intersect at O. Let K ′ be the projection of K onto M N . Then KO = OM + K ′ M .

Proof. Let X, Y, Z, T be the points of tangency of the incircle with KL, LM, M N, N K, respec-
tively. Then

KO = KT + T O = KX + Y O = K ′ Z + OM + M Y = K ′ Z + OM + M Z = K ′ M + OM

K′ N Z M

T Y

K X L

Let A′ , D′ be the feet of A, D onto P Q. Applying lemma for circumscribed trapezoids ABP Q
and DCP Q we obtain DP + A′ P = AD = AP + D′ P .
Without loss of generality assume that ∠BAD ≤ 90◦ . Denote the foot of D onto AB by D′′ .
Then AD′′ = A′ D′ = A′ P − D′ P = AP − DP .
Choose D′′′ on AP such that P D′′′ = P D. Then AD′′′ = AP − D′′′ P = AP − DP = AD′′ . Hence
internal angle bisectors of D′′ AD′′′ and DP D′′′ coincide with perpendicular bisectors of D′′ D′′′
and DD′′′ . It follows that the circumcenter X ′ of DD′′ D′′′ coincides with the A-excenter of ABP .
But it lies on perpendicular bisector of DD′′ as well, which is the line parallel to AB and CD
equidistant from them. It follows that the A-excircle ω of ABP is tangent to CD. It is easy
to see now that X ′ lies on the exterior angle bisectors of DBA and DCA, hence X ′ = X. Let
K, L, M, N be the points of tangency of ω with AB, CD, AC, BD, respectively. Let X∞ be the
point at infinity of lines AB, CD, P R. Brianchon theorem for quadrilateral BX∞ CP shows that
M K and N L intersect at R.
32 Advanced Level

D C L

A′ Q ω
P R
D′ X′ ≡ X
S

N
D ′′′

A D ′′ B K

Since P R ∥ AK and AK = AM , we have P R = P M . Similarly, P N = P R, and, obviously,


DP
P Q = AB · DB = AB · CPCA
= P R. Therefore M and N lie on the circle with diameter QR, so
π
∠QM R = 2 and ∠RN Q = π2 . Since KL is a diameter of ω, we have ∠KM L = π2 = ∠KN L. It
follows that Q, M, L are collinear and Q, N, K are collinear.
Since M L is polar of C with respect to ω and N K is polar of B with respect to ω, we obtain
that BC is polar of Q. In particular QX ⊥ BC, so the intersection of QX with BC is the foot
of X onto BC, i.e. the point S. Hence ∠QSR = π2 , thus S lies on the circle with diameter QR,
so P S = P R.
Solutions 33

Problem 5. Let ABC be an acute triangle inscribed in a circle ω with center O. Points E, F
lie on its sides AC, AB, respectively, such that O lies on EF and BCEF is cyclic. Let R, S be
the intersections of EF with the shorter arcs AB, AC of ω, respectively. Suppose K, L are the
reflection of R about C and the reflection of S about B, respectively. Suppose that points P and
Q lie on the lines BS and RC, respectively, such that P K and QL are perpendicular to BC.
Prove that the circle with center P and radius P K is tangent to the circumcircle of RCE if and
only if the circle with center Q and radius QL is tangent to the circumcircle of BF S.
Proposed by Mehran Talaei - Iran
------------------------------------------------------------------
Solution. Let ω1 , ω2 be the circumcircles of RCE and SBF and let Ω1 , Ω2 be the circles with
centers P, Q and radius P K, QL respectively. Notice that AO ⊥ EF because ∠OAE = 90◦ −
> >
∠ABC and ∠OEA = ∠ABC. So AR = AS = 90◦

P
A

45

R O E
F

B C

G K

Claim 1. ω1 and Ω1 are tangent iff ∠BAC = 45◦ .


First of all it is trivial to see this concludes the problem. Now to the proof:
Proof. First of all notice that ∠RBS = 90◦ . Let G be the reflection of R about B. It is obvious
that BS is the perpendicular bisector of RG and GK is parallel to BC so by P K ⊥ BC, we
can conclude that GK is tangent to Ω1 . Now if T is the reflection of K about BS, it is clear
that T R is also tangent to Ω1 . So the circumcircle of ABC maps to the circumcircle of RT KG
with homothety with its center at R and 2 as its scale. Note that with a little angle chasing the
angle between the circumcircle of ABC and circumcircle of REC is 135◦ , thus the angle between
circumcircle of REC and circumcircle of RT K is also 135◦ . Now we can rewrite the problem from
the perspective of the triangle RT K:
34 Advanced Level

Problem 1. In triangle RT K, let Ω1 be a circle passing through K and tangent to RT at T . Also


let ω1 be a circle passing through R and C, midpoint of RK, which angle with the circumcircle
of RKT is 135◦ . Prove that ω1 is tangent to Ω1 iff RKT = 45◦ .

T
135
Ω1

ω1 C
K

We prove the problem by performing an inversion centered at R with radius RK.RT and then
reflecting about the angle bisector of K ′ RT ′ . The following is the resulted problem: (The images
of the points are denoted by primes)

Problem 2. In triangle RK ′ T ′ , let C ′ be the reflection of R about K ′ and Z the reflection


of R about T ′ . Let Ω′1 be a a circle passing through K ′ and tangent to RT ′ at T ′ and let ω1′ be a
line passing through C ′ which the angle between ω1′ and C ′ Z is 45◦ . Then ω1′ is tangent to Ω′1 iff
∠RT ′ K ′ = 45◦ .

K′ T′

O
Ω1 ′

C′ Z
45
H
Solutions 35

Part 1. ∠RT ′ K ′ = 45◦


Let O be the circumcenter of Ω′1 . Notice that K ′ O ∥ RZ ∥ ω1′ . Then K ′ O is the midline of C ′
in triangle RC ′ Z. Let E be the intersection of ω1′ and K ′ T ′ thus K ′ is the midpoint of T ′ E. So
the reflection of T ′ about O ,H, lies on EC ′ . So H is the tangent point of ω1′ and Ω′1 because
∠OHC = 90◦ .

Part 2. ω1′ is tangent to Ω′1

E K′ T′

C′ Z
45
H

>′
>′
>′
>′
Let α be the angle ∠RZC ′ . It’s easy to see that 45◦ = ∠K ′ EH = T H−2 K H and T H+2 K H =
>′ ′ > >
180◦ − K2T = 180◦ − α. So T ′ H = 225◦ − α and K ′ H = 135◦ − α. Now we split the problem into
two the cases α > 45◦ and α < 45◦ .
Case 1. α > 45◦
With easy angle chasing we conclude that ∠K ′ C ′ E > ∠K ′ RT ′ . Therefor K ′ E > K ′ T ′ . So
′ ′
( HK
HT ′
)2 = EK
ET ′
> 12 . Thus:

sin( 1352 −α ) 1
225◦ −α >

sin( 2 ) 2
◦ √ √ √
So if θ = 1352 −α , then 2 sin θ > sin(θ + 45) = 22 sin θ + 22 cos θ, so sin θ > cos θ hence θ > 45◦
therefor α < 45◦ , which is a contradiction.
Case 2. α < 45◦
It’s similar to the first case.
As shown above both cases cause a contradiction so α must be equal to 45◦ , as requested.

You might also like